EXAM 2 (CH 9,12,13,14,15)

Pataasin ang iyong marka sa homework at exams ngayon gamit ang Quizwiz!

An infertile woman has been diagnosed with endometriosis. She asks the nurse why that diagnosis has made her infertile. Which of the following explanations is appropriate for the nurse to make? A. "Scarring surrounds the ends of your tubes, preventing your eggs from being fertilized by your partner's sperm." B. "You are producing insufficient quantities of follicle-stimulating hormone that is needed to mature an egg every month." C. "Inside your uterus is a benign tumor that makes it impossible for the fertilized egg to implant." D. "You have a chronic infection of the vaginal tract that makes the secretions hostile to your partner's sperm."

A Endometriosis is characterized by the presence of endometrial tissue outside the uterine cavity. The tissue may be on, for example, the tubes, ovaries, or colon. Adhesions develop from the monthly bleeding at the site of the misplaced endometrial tissue, often resulting in infertility.

A nurse working in an infertility clinic should include which of the following in her discussions with the couple? A. Adoption as an alternative to infertility treatments B. The legal controversy surrounding intrauterine insemination C. The need to seek marriage counseling before undergoing infertility treatments D. Statistics regarding the number of couple who never learn why they are infertile

A It is important for the couple to be provided with al relevant information. Adoption is a viable alternative to infertility treatments.

A couple who has sought fertility counseling has been told that the man's sperm count is very low. The nurse advises the couple that spermatogenesis is impaired when which of the following occurs? A. The testes are overheated B. The vas deferens is ligated C. The prostate gland is enlarged D. The flagella are segmented

A Spermatogenesis occurs in the testes. High temperatures harm the development of the sperm

A couple is seeking infertility counseling. The practitioner has identified the factors listed below in the woman's health history. Which of these findings may be contributing to the couple's infertility? A. The client is 36 years old B. The client was 13 years old when she started to menstruate C. The client works as a dental hygienist 3 days a week D. The client jogs 2 miles every day

A The eggs of an older woman (for reproductive purposes considered age 35 years +) age and fertility is reduced.

A client asks the nurse about the gamete intrafallopian transfer (GIFT) procedure. Which of the following responses would be appropriate for the nurse to make? A. Fertilization takes place in the woman's body B. Zygotes are placed in the fallopian tubes C. Donor sperm are placed in a medium with donor eggs D. A surrogate carries the infertile woman's fetus

A This statement is true. Although the gametes are placed in the fallopian tubes artificially, fertilization does occur within the woman's body.

An infertility specialist is evaluating whether a woman's cervical mucus contains enough estrogen to support sperm motility. Which of the following tests is the physician conducting? A. Ferning capacity B. Basal body temperature C. Colposcopy D. Hysterotomy

A When a woman's cervical mucus is estrogen rich, it is slippery and elastic (thread-like), and when assessed under a microscope, the practitioner will observe "ferning" - that is, an image that looks like a fern. The woman is not in her fertile period, the mucus is thick and gluey.

A woman is 8 months pregnant. She tells the nurse that she knows her baby listens to her, but her husband thinks she is imagining things. Which response by the nurse is most appropriate? "Many women imagine what their baby is like." "A baby in utero does respond to the mother's voice." "You'll need to ask the doctor if the baby can hear yet." "Thinking that your baby hears will help you bond with the baby."

"A baby in utero does respond to the mother's voice." Fetuses respond to sound by 24 weeks. The fetus can be soothed by the sound of the mother's voice, and the nurse should instruct the mother so. Although statement A is accurate, it is not the most appropriate response. Statement D is not appropriate because it gives the mother impression that her baby cannot hear her and belittles her interpretation of her fetus's behaviors.

"______, ______, ______, _____, and ______ Developmental tasks that are accomplished by parents as they adapt to the changes of pregnancy and impending parenthood"

"Accepting the pregnancy; identifying with role of par- ent; reordering personal relationships; establishing relationship with the fetus; preparing for childbirth"

A woman who is 14 weeks pregnant tells the nurse that she always had a glass of wine with dinner before she became pregnant. She has abstained during her first trimester and would like to know whether it is safe for her to have a drink with dinner now. The nurse tells her: "Because you're in your second trimester, there's no problem with having one drink with dinner." "One drink every night is too much. One drink three times a week should be fine." "Because you're in your second trimester, you can drink as much as you like." "Because no one knows how much or how little alcohol it takes to cause fetal problems, the best course is to abstain throughout your pregnancy."

"Because no one knows how much or how little alcohol it takes to cause fetal problems, the best course is to abstain throughout your pregnancy." Although the consumption of occasional alcoholic beverages may not be harmful to the mother or her developing fetus, complete abstinence is strongly advised. A safe level of alcohol consumption during pregnancy has not yet been established. The first trimester is a crucial period of fetal development, but pregnant women at all gestations are counseled to eliminate all alcohol from their diet.

Which statement made by a patient having an amniocentesis indicates an understanding of the risks associated with the diagnostic test? "I will not need RhoGAM after this even though I am Rh-negative." "There is a moderate risk I will get an infection as a result of this test." "Because the needle is guided by ultrasound, the risk of injury to the baby is small." "Because I am 14 weeks gestation, there is a higher chance of the baby having a foot deformity after the procedure."

"Because the needle is guided by ultrasound, the risk of injury to the baby is small." Risk of injury to the fetus or umbilical cord is low when ultrasound is used to guide needle insertion. This statement indicates that the patient understands the risks of amniocentesis

As a pregnant woman establishes a relationship with her fetus and emotional attachment begins, she progresses through three phases. In phase one she accepts the ______ and needs to be able to state _______. In phase two the woman accepts the _______. She can now say ________. Finally, in phase three the woman prepares realistically for the _______ and _____. She expresses the thought _________.

"Biologic fact of pregnancy; "I am pregnant"; grow- ing fetus as distinct from herself; "I am going to have a baby"; birth; parenting of the child; "I am going to be a mother"" As a pregnant woman establishes a relationship with her fetus and emotional attachment begins, she progresses through three phases. In phase one she accepts the Biologic fact of pregnancy and needs to be able to state "I am pregnant". In phase two the woman accepts the grow- ing fetus as distinct from herself. She can now say "I am going to have a baby". Finally, in phase three the woman prepares realistically for the birth and parenting of the child. She expresses the thought "I am going to be a mother".

A pregnant woman asks "How does my baby protect itself from amniotic fluid?" Which response by the nurse explains how the fetus protects itself during development? "Protection from amniotic fluid is not necessary." "Both vernix and lanugo shield your baby in utero." "Lanugo secretes vernix that protects your baby's skin." "A cheese-like substance is secreted from your sebaceous glands to protect the baby."

"Both vernix and lanugo shield your baby in utero." Vernix covers the skin to protect it from constant exposure to amniotic fluid. Lanugo is fine, downy hair that covers the fetal body to help the vernix adhere to the skin. Both structures play a role in protecting the fetus from the amniotic fluid.

A woman says, "My husband and I will be trying to have a baby soon." Which statement by the woman demonstrates an understanding of the growth that occurs during the embryonic period? "If I eat more, then the baby's structures will grow larger." "By week 8, all organs are formed, but none are functioning." "By the time I am aware of my pregnancy, my baby's structure is formed." "In the beginning, the fetal brain and heart are the only organs that grow rapidly."

"By the time I am aware of my pregnancy, my baby's structure is formed." The woman has an accurate understanding that basic structures of all major body organs are developed during the embryonic period. By the end of the 8th week, all major organ systems are in place, and many are functioning in a simple way.

Which statement, made by the nurse, demonstrates appropriate patient teaching regarding the effect of cervical mucus abnormalities on conception? "Cervical mucus may cause an obstruction, preventing sperm from entering the uterus." "Cervical mucus abnormalities can be caused by sexually transmitted diseases (STDs)." "Cervical mucus deficiency keeps sperm from penetrating the uterus." "Cervical mucus deficiency causes painful periods and painful intercourse."

"Cervical mucus deficiency keeps sperm from penetrating the uterus." Abnormal cervical mucus caused by estrogen deficiency, surgical destruction of the mucus-secreting glands, or cervical damage secondary to infection or other factors can prevent normal capacitation and movement of the sperm into the uterus and fallopian tubes for fertilization.

"Supine hypotension; vena cava; aorta"

"Change in blood pressure that can occur when a pregnant woman lies on her back and the ______ and the _________ are compressed by abdominal contents, including the uterus."

A woman's last normal menstrual period (LNMP) began on November 9, 2014, and ended on November 14, 2014. Using Nägele's rule, the estimated date of birth is: A. July 2, 2015 B. July 7, 2015 C. August 16, 2015 D. August 21, 2015

"Choice C is correct; use Nägele's rule by subtracting 3 calendar months and adding 7 days and 1 year (if appropriate) to the first day of the last normal menstrual period, which in this case is November 9, 2014." C. August 16, 2015

Doulas are becoming important members of a laboring woman's health care team. Which activity should be expected as part of the doula's responsibilities? A. Monitoring hydration of the laboring woman, including adjusting intravenous (IV) flow rates B. Interpreting electronic fetal monitoring tracings to determine the well-being of the maternal-fetal unit C. Eliminating the need for the husband or partner to be present during labor and birth D. Providing continuous support throughout labor and birth, including explanations of labor progress

"Choice D is correct; continuous support is critical and involves praise, encouragement, reassurance, comfort measures, physical contact, and explanations; the doula does not get involved in clinical tasks; she is not a substitute for the father but rather encourages his participation as a partner in support- ing the laboring woman." D. Providing continuous support throughout labor and birth, including explanations of labor progress

A woman at 30 weeks of gestation assumes a supine position for a fundal measurement and Leopold maneuvers. She begins to complain about feeling dizzy and nauseated. Her skin feels damp and cool. The nurse's first action is to: A. Assess the woman's blood pressure and pulse B. Provide the woman with an emesis basin C. Elevate the woman's legs 20 degrees from her hips D. Turn the woman onto her left side

"Choice D is correct; supine hypotension related to compression of the aorta and vena cava is being ex- perienced; the first action is to remove the cause of the problem by turning the woman onto her side; this should alleviate the symptoms being experienced, including nausea; assessment of vital signs can occur after the woman's position is changed." D. Turn the woman onto her left side

During an early-bird prenatal class, a nurse teaches a group of newly diagnosed pregnant women about their emo- tional reactions during pregnancy. What topics should the nurse discuss with the women? (Circle all that apply.) A. Sexual desire (libido) usually increases during the second trimester of pregnancy. B. A referral for counseling should be sought if a woman experiences conflicting feelings about her pregnancy in the first trimester. C. Rapid, unpredictable mood swings reflect gestational bipolar disorder. D. A quiet period of introspection is often experienced around the time a woman feels her baby move for the first time. E. A woman's own mother is usually her greatest source of emotional support during pregnancy. F. Attachment to her baby begins late in the third trimester, when she begins attending childbirth preparation classes and realizes that the baby will arrive soon.

"Choices A and D are correct; during this normal quiet period a woman focuses on her fantasy child as the attachment process is occurring; sexual desire is decreased during the first and third trimesters and increased in the second; ambivalence and mood swings are common responses when preparing for a new role and reacting to hormonal changes; safe pas- sage and birth preparation are primary concerns during the third trimester; a woman's partner is usually the most important person." A. Sexual desire (libido) usually increases during the second trimester of pregnancy. D. A quiet period of introspection is often experienced around the time a woman feels her baby move for the first time.

The nurse evaluates a pregnant woman's knowledge about prevention of urinary tract infections (UTIs) at the pre- natal visit following a class on infection prevention that the woman attended. The nurse recognizes that the woman needs further instruction when she tells the nurse about which measures that she now uses to prevent urinary tract infections. (Circle all that apply.) A. I drink about one quart of fluid a day. B. I have stopped using bubble baths and bath oils. C. I have started wearing panty hose and underpants with a cotton crotch. D. I have intercourse with my husband only once a week now because it could lead to bladder infections. E. If I drink cranberry juice at least twice a day, I will not get an infection.

"Choices A, D, and E are correct; intake of at least 2 to 3 L per day is recommended; she does not have to reduce frequency of intercourse but rather void before and after intercourse and then drink a large glass of water; the effectiveness of cranberry juice has not been proven; additional methods of preventing UTIs include frequent, regular urination; good genital hygiene; and avoiding wearing tight-fitting jeans for long periods." A. I drink about one quart of fluid a day. D. I have intercourse with my husband only once a week now because it could lead to bladder infections. E. If I drink cranberry juice at least twice a day, I will not get an infection.

"Ambivalence"

"Emotional state of having conflicting feelings about a pregnancy simultane- ously; it is considered a normal response to pregnancy."

A patient has diagnosed endometriosis. Which statement by the patient demonstrates an understanding of the relationship between infertility and endometriosis? "Endometriosis can affect the patency of the ovaries." "Endometriosis can scar and block the fallopian tubes." "Endometriosis can increase the frequency of shedding of the endometrium." "Endometriosis can cause the uterine lining to overproduce in the uterus."

"Endometriosis can scar and block the fallopian tubes." Endometriosis causes tubal adhesions, painful menstrual periods, and painful intercourse. Large lesions can distort fallopian tube anatomy and lead to infertility.

A woman at 12 weeks' gestation asks "When will I begin to feel my baby move?" Which statement by the nurse provides information on fetal movement at this gestational age? "Don't worry; I am sure your baby is fine." "This is concerning. Let's complete an ultrasound." "Even though you can't feel it yet, your baby is moving." "You may never feel your baby move."

"Even though you can't feel it yet, your baby is moving." During the first trimester, the first fetal movements begin but are often too slight for the mother to detect.

A student nurse is reviewing the conception process with a classmate. Which statement demonstrates an understanding of the fertilization process? Select all that apply. "Fertilization causes the ovum to undergo mitosis." "Fertilization causes changes to the zona pellucida." "It only takes 1 day for fertilization to be complete." "Cell division occurs after the sperm and ovum merge." "Fertilization describes the union between the ovum and the sperm." "Sperm penetrate the ovum by enlarging and lengthening the tail."

"Fertilization causes changes to the zona pellucida." The zona pellucida is a mechanical protection of the egg as it travels from the ovary to the uterus. Once the egg is fertilized, the zona pellucida keeps other sperm from entering. "It only takes 1 day for fertilization to be complete." Fertilization is completed within 24 hours. "Cell division occurs after the sperm and ovum merge." Cell division begins when the nuclei of the sperm and ovum unite. "Fertilization describes the union between the ovum and the sperm." Fertilization consists of the entry of one spermatozoon into the ovum and fusion of the nuclei of sperm and ovum.

A woman at 8½ weeks' gestation presents for a prenatal check-up. She is eager to find out the baby's sex. Which response by the nurse best explains genitourinary changes at this stage? "Gender structure should be visible in 2 weeks." "The baby's DNA will assign the sex after week 10." "At this phase, nothing changes in regard to the genitalia." "External genitalia begin differentiating at week 8, so let's take a look."

"Gender structure should be visible in 2 weeks." Although in week 8 the external genitalia begin to differentiate, male and female characteristics are not distinct until after the 10th week.

"Noninvasive prenatal testing"

"Genetic testing technology used to identify fetuses at risk for trisomies 21, 18, and 13 and sex chromosome abnormalities."

A couple that is infertile has just been made aware that their infertility is related to hypospadias. Which statements by the couple demonstrate that more teaching is needed? Select all that apply. "Sperm cannot reach the cervix." "Hypospadias affects ejaculation." "Hypospadias affects sperm structure." "Hypospadias is an anatomic problem." "Hypospadias is possible in both partners."

"Hypospadias affects sperm structure." The couple needs further teaching to understand that hypospadias affects how sperm are deposited, not the structure of the sperm themselves. "Hypospadias is possible in both partners." The couple needs further teaching to understand that hypospadias occurs in men only. In men with hypospadias, the urethral opening is on the underside of the penis.

A patient is seeking family planning advice at her annual wellness visit. The nurse provides teaching for the patient about conception and ovulation. Which statement by the patient on the timing of conception indicates that further teaching is necessary? "I can get pregnant up to 7 days after ovulation." "Sperm can survive up to 2 days after sexual intercourse." "When trying to conceive, we should plan to have sex when I am ovulating." "Even if I am ovulating, my husband still has to produce enough healthy sperm for me to conceive."

"I can get pregnant up to 7 days after ovulation." More teaching is required so that the patient understands that the ovum may have the capacity to be fertilized for 24 hours after ovulation, not for 7 days.

A nursing student is reviewing information relative to cultural beliefs about infertility. Which statement, if made by a patient, would require intervention? A patient states that she wants to consult her Rabbi before going any further with an infertility workup. "I will not be able to use IVF therapies as it is against my religion; I am a Roman Catholic." "I have to take full responsibility for my failure to conceive." "I may consider having infertility treatments but I have to talk this over with my husband first."

"I have to take full responsibility for my failure to conceive." In certain cultures, women are viewed as being responsible for all fertility problems, but the fact that this statement acknowledges complete fault would require the nursing student to intervene in order to provide compassion and decrease the psychologic stress of the patient. Statements about consulting the partner are expected, because both partners should discuss and explore available fertility options. Consultation about infertility treatments with a spiritual leader, regardless of religion, requires no intervention. Certain religions, such as Roman Catholicism, do not allow IVF therapy.

Which patient statement would signal a contraindication for a transcervical chorionic villus sampling (CVS) procedure? "My doctor told me I have preeclampsia." "My last ultrasound said I'm 10 weeks pregnant." "I had a baby before this one, but it had neurological defects." "I took fluconazole (Diflucan) yesterday for a yeast infection."

"I took fluconazole (Diflucan) yesterday for a yeast infection." Vaginal infection is a contraindication for transcervical CVS due to increased risk of uterine infection. The patient would require transabdominal CVS instead.

A woman at 26 weeks' gestation visits the clinic for a prenatal check-up. The baby is in a breech position as revealed by a sonogram. The woman asks "Shouldn't he already be head down by now?" Which response by the nurse explains fetal position changes at this gestational age? "Don't worry; babies typically flip after 30 weeks." "I understand your concern, but babies usually do not flip until about week 28." "We need to plan for a C-section as babies assume their birthing position now." "The baby's head is bigger than the feet but not heavy enough to cause him to turn yet."

"I understand your concern, but babies usually do not flip until about week 28." The fetus floats freely but begins to be positioned head down between weeks 25 and 28; therefore, the baby may still flip.

When scheduling a diagnostic procedure, a patient tells the nurse that she plans to terminate the pregnancy if an abnormality is found. Which is an appropriate response from the nurse? "What made you come to that decision?" "That isn't what I would do in your situation, but the decision is yours to make." "I will give you contact information for support groups for parents of children with disabilities in case you change your mind." "I will give you printed information on how to schedule pregnancy termination so you know what to do if the test finds an abnormality."

"I will give you printed information on how to schedule pregnancy termination so you know what to do if the test finds an abnormality." This response supports the patient's decision and provides follow-up care to assist the patient.

A patient pregnant with monozygotic twins shares with the nurse, "During my last labor with my son, it was so uncomfortable passing the placenta. Now I have to pass two!" Which are the nurse's best responses to the patient? Select all that apply. "Don't worry, it will be easier the second time around." "I'm sorry that it was an uncomfortable experience for you." "In this type of twin pregnancy, your babies may share a placenta." "Yes, but with twin pregnancies, the placenta often passes much quicker." "To assist with the discomfort, we can talk to the health care provider about providing pain medication during your labor."

"I'm sorry that it was an uncomfortable experience for you." This is a positive therapeutic response acknowledging the patient's experience. Correct "In this type of twin pregnancy, your babies may share a placenta." Some monozygotic twins share a placenta, making this an appropriate response by the nurse.

A patient whose ultrasound resulted in normal findings expresses relief that the fetus is "completely healthy." Which response from the nurse is appropriate? "Follow-up ultrasound scans will be necessary but only to measure fetal growth." "It can take more than one ultrasound to determine if your baby is completely healthy." "I'm happy that this test can give you peace of mind; most ultrasounds come back normal." "I'm very happy that you feel relieved; however, additional tests may still be indicated to detect other possible abnormalities."

"I'm very happy that you feel relieved; however, additional tests may still be indicated to detect other possible abnormalities." Ultrasound cannot detect every structural abnormality, and it cannot detect abnormalities that do not affect the structure of the fetus. This response is appropriate because it addresses the patient's misconception regarding ultrasound testing.

A patient who is 10 weeks pregnant feels uncomfortable having a transvaginal ultrasound and asks the nurse if there is any way she could have a transabdominal ultrasound instead. Which is the appropriate response from the nurse? Select all that apply. "We can perform a transabdominal ultrasound if your bladder is full." "If it makes you feel more comfortable, you can insert the probe yourself." "We would not be able to perform a transabdominal ultrasound until next week." "I will have the health care provider change the order to a transabdominal ultrasound." "Because you are in your first trimester, the uterus and embryo are difficult to visualize with a transabdominal ultrasound."

"If it makes you feel more comfortable, you can insert the probe yourself." Patients may feel more comfortable inserting the probe themselves for a transvaginal ultrasound. "Because you are in your first trimester, the uterus and embryo are difficult to visualize with a transabdominal ultrasound." The uterus, ovaries, and embryo are deep within the pelvis during the first trimester; therefore, transvaginal ultrasound is often used.

"Probable"

"Indicators of pregnancy that are detected by an examiner and are related mainly to physical changes in the uterus; they strongly suggest pregnancy but are not conclusive."

A couple returns to the clinic for prenatal counseling. The woman states "If we don't get pregnant in the next year, can we assume the problem is with one of our reproductive organs?" Which statement by the nurse explains the infertility factors for men and women? "Infertility is typically only associated with one of the partners." "Let's not worry about that now. I am sure you will conceive before then." "Let's not assume so soon. Infertility cannot be determined after 1 year." "Infertility can be influenced by factors including social behaviors and your environment."

"Infertility can be influenced by factors including social behaviors and your environment." Several factors, including anatomic factors, psychosocial factors, and the function of reproductive organs, contribute to infertility in men and in women.

A couple presents to the clinic for infertility evaluation. The man states "A low sperm count is the only cause of infertility in men, right?" Which statement by the nurse best teaches the couple about male infertility factors? "Male infertility is usually a matter of sperm count." "Male infertility can also be related to low testosterone levels." "Male infertility can be associated with the structure and function of sperm or the penis." "Male infertility can be related to obesity."

"Male infertility can be associated with the structure and function of sperm or the penis." Infertility in males can be caused by an inadequate number of sperm or by problems with sperm structure and function, erection, or ejaculation.

"Sequential Integrated Screening (SIS)"

"Multiple marker screen involving two blood tests and one ultrasound offered to women who start prenatal care before 14 weeks of gestation; it is used to identify such conditions as Down syndrome, neural tube defects, and abdominal wall abnormalities."

A woman has had three lost pregnancies in 18 months. The nurse is discussing possible causes with the patient. Which patient statement demonstrates that the woman understands this type of infertility? Select all that apply. "My pregnancy losses could be linked to my age." "My pregnancy losses could be the result of chromosomal errors." "My pregnancy losses could be caused by an incompetent cervix." "My pregnancy losses could be linked to the high stress I feel at my job." "My pregnancy losses could be associated with a hormonal imbalance."

"My pregnancy losses could be linked to my age." This statement shows an understanding by the patient that maternal age can increase the risk for spontaneous abortion. Correct "My pregnancy losses could be the result of chromosomal errors." This statement shows an understanding by the patient that errors in the fetal chromosomes may result in spontaneous abortion, usually in the first trimester. Chromosome abnormalities can severely disrupt development. Correct "My pregnancy losses could be caused by an incompetent cervix." This statement shows understanding by the patient; incompetent cervix is painless and premature cervical dilation, which often occurs early in the second trimester. Although conception occurs, the woman may be unable to carry the pregnancy long enough for the fetus to survive if born. "My pregnancy losses could be associated with a hormonal imbalance." This statement shows an understanding by the patient that pregnancy losses can occur as a result of inadequate progesterone secretion; menstrual irregularities and anovulation associated with increased luteinizing hormone and reduced follicle-stimulating hormone impair ovum maturation; and other hormonal conditions such as hypothyroidism, hyperthyroidism, and poorly controlled diabetes.

"Trimester"

"One of three periods of pregnancy, each of which is approximately 3 months in length"

"Couvade syndrome"

"Phenomenon of an expectant father experiencing pregnancy-like symptoms such as nausea, weight gain, and other physical symptoms."

"Emotional lability (mood swings and changes)"

"Rapid unpredictable changes in mood related to profound hormonal changes and concerns about finances and changed lifestyle."

A woman in a non-monogamous relationship expresses the desire to stop using condoms after the placement of an intrauterine device (IUD). Which is the appropriate nursing education on first-time IUD use? "IUD strings can puncture the condom, so there is no point in using them once it has been inserted." "Since you are not in a monogamous relationship, you should still use condoms to protect against STDs." "You should wait until after your first menstrual period with the IUD inserted before you stop using condoms." "IUDs should only be used by women in long-term monogamous relationships, so it wouldn't be a good fit for you."

"Since you are not in a monogamous relationship, you should still use condoms to protect against STDs." An IUD will not protect against STDs, and a woman in a non-monogamous relationship should still use measures to prevent STD infection. This education is appropriate for this woman.

A woman at 30 weeks' gestation is admitted to the labor and delivery unit for preterm labor. The clinical team has just discussed with the woman their concerns about delivering early because of fetal lung development. The woman states to the nurse: "Well, how important is surfactant, and when is it produced?" Which statement by the nurse best explains when and why surfactant is secreted? "Surfactant is first produced in the third trimester to help the baby breathe in utero." "Surfactant is produced for alveoli development when the lungs start developing in the first trimester." "Surfactant is initially produced at the beginning of the second trimester to keep the lungs from collapsing." "Surfactant is produced near the end of the second trimester to help the baby survive, but it isn't used in utero by the lungs."

"Surfactant is produced near the end of the second trimester to help the baby survive, but it isn't used in utero by the lungs." Surfactant is produced in the lungs to reduce the surface tension of fluid and help make the lung air sac more stable.

"Pinch"

"Test used to determine whether the nipple is everted or inverted, achieved by placing thumb and forefinger on the areola and gently pressing inward."

A patient at the OB/GYN clinic states "Once we figure out why I haven't gotten pregnant this past year, then we can fix it." Which response by the nurse best explains the success of identifying infertility causes? "That is our hope, but there is no guarantee." "Infertility usually resolves with time, not treatment." "Successful treatment typically means you will get pregnant afterward." "Most likely, when the cause of infertility is identified, a solution can be achieved."

"That is our hope, but there is no guarantee." Many factors contributing to infertility remain unknown. Although treatment does exist for some identified problems, it does not always lead to a successful pregnancy.

The student nurse knows that during the latter part of the first trimester, the intestines descend into the abdomen. Which statement provides the best explanation for this physiologic change? "The intestines shrink, allowing descent to occur." "The umbilical cord expels the intestines into the abdominal cavity." "The kidneys and the liver must shrink to allow room for the intestines." "The abdominal cavity increases in size, allowing the intestines to descend."

"The abdominal cavity increases in size, allowing the intestines to descend." By week 10, the abdominal cavity is proportional in size, compared with the rest of the body, and can accommodate the intestines.

A woman at 35 weeks' gestation wants to deliver as soon as possible. The nurse provides teaching on normal fetal growth patterns during the third trimester. Which statement by the woman shows further teaching is needed? "My baby's lungs are still maturing until birth." "My baby is growing at a slower pace now." "The baby's weight increases until delivery." "The good news is that my baby will stop growing at 38 weeks."

"The good news is that my baby will stop growing at 38 weeks." The baby will not stop growing at 38 weeks, as the fetus continues growing in length and gaining weight until birth.

Students are completing a clinical rotation in the prenatal clinic. The nurse is reviewing the stages of heart development in utero. Which statement by a student indicates an understanding of the timing of cardiovascular development during the embryonic period? "The heart chambers are formed in week 5." "Cardiac development occurs the most rapidly during weeks 7 and 8." "The cardiac system develops faster than the central nervous system." "The heart is beating, but the heart chambers don't form during the embryonic period."

"The heart chambers are formed in week 5." The heart reaches its final four-chambered form in week 5.

A patient has not felt fetal movement in several days and is scheduled for an ultrasound. Which statement by the nurse helps explain the necessity of the ultrasound? "The ultrasound will let us see your baby's heartbeat." "The ultrasound can show us where your baby is located." "The ultrasound will help us be certain of your baby's gestational age." "With the ultrasound, we can measure how much your baby has grown since your last appointment."

"The ultrasound will let us see your baby's heartbeat." Absence of fetal movement for several days may indicate multiple adverse events, including fetal death. Ultrasound is used to confirm fetal viability, which is the indication that applies to this patient.

A woman's cousin gave birth to an infant with a congenital heart anomaly. The woman asks the nurse when such anomalies occur during development. Which response by the nurse is most accurate? "We don't really know when such defects occur." "It depends on what caused the defect." "They occur between the third and fifth weeks of development." "They usually occur in the first 2 weeks of development."

"They occur between the third and fifth weeks of development." The nurse would be aware of when such defects occur. Regardless of the cause, the heart is vulnerable during its period of development, the third to fifth weeks. The cardiovascular system is the first organ system to function in the developing human. Blood vessel and blood formation begins in the third week, and the heart is developmentally complete in the fifth week.

An expectant father confides in the nurse that his pregnant wife, at 10 weeks of gestation, is driving him crazy. "One minute she seems happy, and the next minute she is crying over nothing at all. Is there something wrong with her?" The nurse's best response is: "This is normal behavior and should begin to subside by the second trimester." "She may be having difficulty adjusting to pregnancy; I will refer her to a counselor I know." "This is called emotional lability and is related to hormone changes and anxiety during pregnancy. The mood swings will eventually subside as she adjusts to being pregnant." "You seem impatient with her. Perhaps this is precipitating her behavior."

"This is called emotional lability and is related to hormone changes and anxiety during pregnancy. The mood swings will eventually subside as she adjusts to being pregnant." The statement in C is the most appropriate response because it gives an explanation and a time frame for when the mood swings may stop. The statement in A is an appropriate response but it does not answer the father's question. Mood swings are a normal finding in the first trimester; the woman does not need counseling. The statement in D is judgmental and not appropriate.

A nurse is meeting with a woman to confirm her pregnancy at week 8; a sonogram technician is also present to confirm the fetal heartbeat by ultrasound. The woman says "I'm so excited to see a little human." Which response should the nurse make to clarify development expectations for this woman? "This must be so exciting!" "You should know the sex too!" "Your baby should look like a ball of cells." "Your baby will still look like a little tadpole."

"This must be so exciting!" This is an appropriate response, as the embryo has a definite human form by week 8.

"Fundal height"

"Uterine measurement that is performed beginning in the second trimester as one indicator of the progress of fetal growth."

A perimenopausal woman wishes to discontinue all contraception. The woman has not had a menstrual period in six months. Which is the appropriate nursing education on contraception during perimenopause? "You should consider tubal sterilization, just to be sure." "We would need to taper off your oral contraceptives to reduce the risk of blood clots." "You can stop your oral contraceptives, but you should still use a barrier method to prevent STDs." "We can change your method of contraception, but you should not stop it completely for another six months."

"We can change your method of contraception, but you should not stop it completely for another six months." Perimenopausal women should continue to use an effective method of contraception until one year after the last menstrual period. This woman's contraception should not be stopped at this time, so this education is appropriate for a perimenopause woman.

"Doula"

"Woman professionally trained to provide physical, emotional, and informa- tional support to women and their partners during labor and birth."

A nurse is discussing with a 25-year-old patient the likelihood of becoming pregnant with monozygotic twins. Which statements by the nurse are included in teaching? Select all that apply. "Your young age does not play a role." "You have no control over having twins." "Your family history does not play a role." "Your partner's family history plays a role." "You are at a higher risk of twins if taking fertility drugs."

"You have no control over having twins." This is appropriate teaching by the nurse as monozygotic twinning occurs at random. Correct "Your family history does not play a role." This is appropriate teaching by the nurse as monozygotic twinning has no genetic links.

A pregnant woman at 10 weeks of gestation jogs three or four times per week. She is concerned about the effect of exercise on the fetus. The nurse should tell her: "You don't need to modify your exercising any time during your pregnancy." "Stop exercising, because it will harm the fetus." "You may find that you need to modify your exercise to walking later in your pregnancy, around the seventh month." "Jogging is too hard on your joints; switch to walking now."

"You may find that you need to modify your exercise to walking later in your pregnancy, around the seventh month." The nurse should inform the woman that she may need to reduce her exercise level as the pregnancy progresses. Typically, running should be replaced with walking around the seventh month of pregnancy. Physical activity promotes a feeling of well-being in pregnant women. It improves circulation, promotes relaxation and rest, and counteracts boredom. Simple measures should be initiated to prevent injuries, such as warm-up and stretching exercises, to prepare the joints for more strenuous exercise.

A woman inquires about herbal alternative methods for improving fertility. Which statement by the nurse is the most appropriate for instructing the woman about which herbal preparations to avoid while trying to conceive? "You should avoid nettle leaf, dong quai, and vitamin E while you are trying to get pregnant." "You may want to avoid licorice root, lavender, fennel, sage, and thyme while you are trying to conceive." "You should not take anything with vitamin E, calcium, or magnesium. They will make you infertile." "Herbs have no bearing on fertility."

"You may want to avoid licorice root, lavender, fennel, sage, and thyme while you are trying to conceive." Although most herbal remedies have not been proven clinically to promote fertility, herbs that a woman should avoid while trying to conceive include licorice root, yarrow, wormwood, ephedra, fennel, goldenseal, lavender, juniper, flaxseed, pennyroyal, passionflower, wild cherry, cascara, sage, thyme, and periwinkle. Nettle leaf, dong quai, and vitamin E promote fertility; calcium and magnesium may promote fertility and conception.

A woman arrives at a prenatal clinic to confirm her pregnancy. Assessment indicates that she is 23 days gestation. She states "Oh, so my baby is just beginning to grow." Which statement by the nurse is the best initial response? "Your baby has a beating heart now." "Your baby's face is already defined." "Your baby's major organs are already in place." "Your baby's central nervous system is developed now."

"Your baby has a beating heart now." At the end of the 3rd week, the neural tube is fused in the middle but is still open at each end. The early heart begins beating at 21 to 22 days.

Some factors affecting cervical mucus:

- Diabetes - antibiotic therapy - IUD - Stress - pathogenic bacteria - polyp presence of blood

What can cause ovulatory dysfunction?

- OBESITY - polycystic ovary syndrome - STRENUOUS EXERCISE - thyroid dysfunction - hyperprolactinemia - vegetables - high in animal protein - cancer -mradiation - chemo therapy - smoking, even passive exposurefactors

nurses must___ in order to deal with couple's sexuality

- be comfortable with own sexuality - possess factual knowledge about human practice - accepting, nonjudgemental - skills in interviewing - knowledge of sociocultural beliefs and religious - gender neuteral language - explore system of support of the couple - explore reaction of couple to pregnancy

mullerian malformations of the uterine cavity

- bicornuate or septate uterus - tumors of the endometrium and myometrium ( polyps, myomas)

ovarian factors affecting female fertility

- developmental anomalies - anovulation, primary or secondary - pituitary or hypothalamic hormone disorder - adrenal gland disorder - congenital adrenal hyperplasia - disruption of hypothalamic-pituitary-ovarian axis - amenorrhea after discontinuing oral contraceptive pills - premature ovarian failure - increased prolactine levels

uterine, tubal, and peritoneal factors

- developmental anomalies - tubal motility reduced - inflammation within the tubes - tubal adhesions - endometrial and myometrial tumors - asherman syndrome ( uterine adhesions or scar tissues)- characterized by hypomenorrhea, because adhesions prevent normal cyclic endometrial proliferation - endometriosis - chronic cervicitis - hortile or inadequate cervical mucus

in fetuses with mother with uncontrolled diabetes

- fetal hyperglycemia - hyperinsulinamia - islet cell hyperplasia - large fetus - blocks lung maturation-> fetal risk of respiratory distress - hypoglycemia whn maternal glucose source is lost at birth

changes of amniotic fluid

- fluids,secreted by the respiratory and GI of the fetus also enters the amniotic cavity - fetus swallows fluid - fluid flows in and out of fetus lungs - week 11 - fetus urinates into the fluid

leg cramps interventions

- getting regular exercise - dorsiflexing the foot ofthe affected leg - increasing calcium intake

non medical therapy of infetility

- lifestyle changes - avoid ht-tub bathing or saunas ( men) - men should wear loose clothing to improve sperm count - remmber that lubricants may diminish sperm motility - cell phones on belt may decrease sperm quality - nutritional changes - avoid alcohol and smoking - overweight and obese may have decreased chance of pregnancy - counseling on optimal time of intercourse

Therapeutic donor insemination (TDI)/ indications

- male parter is azoospermic or has a very low sperm count - couple has a genetic defect - male partner has antisperm antibodies - lesbian couple

varicose veins interventions

- wearing supportive stockings - elevating the feet when sitting - lying with the feet and hips elevated - avoiding long periods of standing or sitting - moving about while standing to improve circulation - avoidijg leg crossing - avoiding constricting articles of clothing

implantation

- zona pellucida degenerates - blastocyst embeds into the endometrium ( anterior, posterior fundal region) - 6-10 days after conception, trophoblast secretes enzymes, which enable it to borrow into the endometrium until entire blastocyst is covered

Multifetal Pregnancy

-A multifetal pregnancy, or pregnancy with more than one fetus, places the mother and fetuses at increased risk for adverse outcomes. -The maternal blood volume is increased, resulting in an increased strain on the maternal cardiovascular system. -Placenta previa develops more commonly in multifetal pregnancies because of the large size or placement of the placentas. -Twin pregnancies often end prematurely. -The mother needs nutrition counseling so that she gains more weight than that needed for a singleton birth. -Fetal growth is carefully monitored throughout pregnancy using regular ultrasound evaluations. -Iron and vitamin supplements are desirable. -Every multifetal pregnancy is at risk for preterm labor; thus the women receive frequent ultrasound examinations, FHR monitoring, and nonstress tests. Routine bedrest is not recommended in twin pregnancies at low risk for preterm labor. -If bedrest is recommended, the mother needs to assume the lateral position to promote increased placental perfusion. If birth is delayed until after the thirty-sixth week, the risk for morbidity and mortality decreases for the neonates.

History of Drugs and Herbal Preparations

-A woman's past and present use of drugs, both legal OTC, prescriptions, and herbal drugs; caffeine; alcohol; nicotine and illegal (marijuana, cocaine, heroin), must be assesed because many substances cross the placenta and can harm the developing fetus. -It is important for health care providers to question pregnant women regarding the use of herbal preparations and document their responses.

Air Travel

-Air travel in large commercial jets usually poses little risk to the pregnant woman, but policies vary from airline to airline. -Most health care providers allow air travel up to 36 weeks of gestation in women without medical or pregnancy complications. -Women at risk for preterm labor should avoid air travel. - Magnetometers (metal detectors) used at airport security checkpoints are not harmful to the fetus. -The 8% humidity at which cabins are maintained in commercial airlines may result in some water loss; hydration (with water) should be maintained under these conditions. -A pregnant woman is encouraged to take a 15-minute walk around the aircraft during each hour of travel to minimize this risk.

History of Physical Abuse

-All women should be assessed for a history of or risk for physical abuse, particularly because the likelihood of intimate partner violence (IPV) increases during pregnancy. -It is essential that the screening is done in a safe, private setting with the woman alone. -Although visual cues from the woman's appearance or behavior may suggest the possibility of abuse, no one profile of the battered woman exists. -Women report physical blows directed to the head, breasts, abdomen, and genitalia. Sexual assault is common. -Routine screening for abuse and sexual assault is recommended for pregnant adolescents. -Nurses should be aware that victims of human trafficking may be seen in prenatal settings because of unintended pregnancy.

Kegel Exercises

-Alternate contracting and relaxing of the pelvic floor muscles to strengthen the muscles around the reproductive organs and improve muscle tone. -The pelvic floor muscles encircle the vaginal outlet, and they need to be exercised. -Practice of pelvic muscle exercise during pregnancy also results in fewer complaints of urinary incontinence in late pregnancy and postpartum.

Medications and Herbal preparations

-Although much has been learned in recent years about fetal drug toxicity, the possible teratogenicity of many prescription and OTC drugs is still unknown. -The greatest danger of drug-caused developmental defects in the fetus extends from the time of fertilization through the first trimester, a time when the woman may not realize she is pregnant. -The use of all drugs, including OTC medications, herbs, and vitamins, should be limited; and a careful record should be kept of all therapeutic and nontherapeutic agents used. -There is limited research evidence about the safety of herbal preparations, especially during pregnancy.

Group Prenatal Care

-An alternative model to traditional care during pregnancy. -Authority is shifted from the provider to the woman and other women who have similar due dates. -The model creates an atmosphere that facilitates learning, encourages discussion, and develops mutual support. -Centering Pregnancy, is a well-known model of group prenatal care that involves three components: health care assessment, education, and peer support. -Families and partners are encouraged to participate. -Benefits associated with group prenatal care include improved birth outcomes such as lower rates of preterm birth, increased knowledge, improved satisfaction, and higher breastfeeding initiation rates

Drugs

-Any drug or environmental agent that enters the pregnant woman's bloodstream has the potential to cross the placenta and harm the fetus. -Although substance abuse in pregnancy is a major public health concern and comprehensive care of drug-addicted women improves maternal and neonatal outcomes, few facilities are available for treatment of these women.

Physical parameters measured at each visit

-BP is taken at every visit, using the same arm and with the woman seated. -Weight is measured, and the appropriateness of the weight gain is evaluated in relation to her BMI. -Urine may be checked by dipstick. -The presence and degree of edema are noted. -Abdominal inspection is followed by measurement of the height of the fundus.

Prevention of Urinary Tract Infection

-Because of physiologic changes that occur in the renal system during pregnancy (see Chapter 7), infections of the lower urinary tract (acute urethritis, acute cystitis) are common. -Women should be instructed to inform their health care provider promptly if they experience these symptoms. -Urinary tract infections pose a risk to the mother and fetus; thus their prevention or early treatment is essential. -Advise her to drink at least 2 liters (L) (eight glasses) of liquid a day to maintain an adequate fluid intake that ensures frequent urination. -The consumption of yogurt and acidophilus milk can help prevent urinary tract and vaginal infections. -Women should be told not to ignore the urge to urinate, because holding urine lengthens the time bacteria are in the bladder and allows them to multiply. -Bacteria also can be introduced during intercourse.

Second Trimester Screenings

-Between 15 and 20 weeks, maternal serum alpha-fetoprotein (MSAFP) screening, the QUAD test (alpha-fetoprotein, hCG, estriol, and inhibin A), or Penta Screen (components of QUAD test plus invasive trophoblast antigen [ITA]) can be done to screen for neural tube defects (NTDs) and other chromosomal abnormalities. -An ultrasound is often done at 18 to 24 weeks to survey fetal anatomy. -If the cervix is confirmed to be short, further assessment for preterm birth risk factors should occur and management options should be considered.

Harmful affects of Cigarette Smoking

-Cigarette smoking or continued exposure to secondhand smoke (even if the mother does not smoke) is associated with IUGR and an increase in perinatal and infant morbidity and mortality. -Smoking can affect fertility and may lead to a greater risk for ectopic pregnancy. -Smoking is associated with an increased incidence of spontaneous abortion, ectopic pregnancy, preterm birth, PROM, abruptio placentae, placenta previa, and fetal death. -All women who smoke should be strongly encouraged to quit or at least reduce the number of cigarettes they smoke. -Pregnant women should be told about the negative effects of secondhand smoke on the fetus and encouraged to avoid such environments.

Reordering Personal Relationships

-Close relationships of the pregnant woman undergo change as she prepares emotionally for the new role of mother. -Promoting effective communication patterns between the expectant mother and her own mother and between the expectant mother and her partner are common nursing interventions provided during the prenatal visits. -The woman's relationship with her mother is significant in adapting to pregnancy and motherhood. -Although the woman's relationship with her mother is significant in considering her adaptation in pregnancy, the most important person to the pregnant woman is usually the father of her child.

Human Trafficking

-Coercion or deception of individuals (usually women and children) to perform commercial sex acts or forced labor. -They are under strict control by their traffickers. -Similar to victims of IPV, these women are likely to exhibit signs of physical abuse or neglect such as scars, bruises, burns, unusual bald patches, or tattoos that may be a sign of branding.

Cultural Prescriptions and Proscriptions

-Cultural Prescriptions; Tell women what to do. -Cultural Proscriptions; Establish taboos. -The purposes of these practices are to prevent maternal illness caused by a pregnancy-induced imbalanced state and to protect the vulnerable fetus. - Prescriptions and proscriptions regulate the woman's emotional response, clothing, physical activity and rest, sexual activity, and dietary practices.

Childbearing and female Reproductive System History

-Data are gathered on the woman's age at menarche; menstrual history; contraceptive history; history of infertility or gynecologic conditions; history of any STIs; her sexual history; and a detailed history of all her pregnancies, including the present one, and their outcomes. -The date of the last Papanicolaou (Pap) test and the result are noted. -The date of her LMP is obtained to establish the EDB.

Dental Health

-Dental care during pregnancy is especially important because nausea during pregnancy may lead to poor oral hygiene and allow dental caries to develop. -Inflammation and infection of the gingival an periodontal tissues may occur. -Diagnosis and treatment of oral health problems, including necessary dental x-rays, are safe during pregnancy. -Dental care and nonemergent procedures are best scheduled during the second trimester when the woman is past the stage of feeling nauseous and can sit comfortably in the dental chair.

Personal Hygiene

-During pregnancy, the sebaceous (sweat) glands are highly active because of hormonal influences and women often perspire freely. -Baths and warm showers can be therapeutic because they relax tense, tired muscles; help counter insomnia; and make the pregnant woman feel fresh. -ub bathing is permitted even in late pregnancy because little water enters the vagina unless under pressure. -Tub bathing is contraindicated after rupture of the membranes.

Review of Systems

-During the review of systems, ask the woman to identify and describe pre-existing or concurrent problems in any of the body systems and assess her mental status. -Pregnancy affects and is affected by all body systems; therefore information on the present status of body systems is important in planning care

Fundal Height

-During the second trimester, the uterus becomes an abdominal organ. -The fundal height, or measurement of the height of the uterus above the symphysis pubis, is used as one indicator of fetal growth. -The measurement also provides a gross estimate of the duration of pregnancy. -The fundal height measurement may aid in the identification of risk factors. -A stable or decreased fundal height may indicate the presence of intrauterine growth restriction (IUGR); an excessive increase could indicate the presence of multifetal gestation (more than one

Establishing a Relationship with the Fetus

-Emotional attachment—feelings of being tied by affection or love—begins during the prenatal period as women use fantasizing and daydreaming to prepare themselves for motherhood. -They try to anticipate changes that the child will bring in their lives and wonder how they will react to noise, disorder, reduced freedom, and caregiving activities -Providing an accepting environment for parental reactions facilitates the parents' ability to move beyond disappointment to acceptance.

Employment

-Employment of pregnant women usually has no adverse effects on pregnancy outcomes. -Pregnant women should not perform any job that subjects them to severe physical strain or exposes them to harmful substances. -Job discrimination that is based solely on pregnancy is illegal. -Women in sedentary jobs need to walk around at intervals to counter the sluggish circulation in the legs. -They should avoid crossing their legs at the knees because all of these activities can foster the development of varices and thrombophlebitis. -Standing for long periods also increases the risk for preterm labor

Psychosocial Support

-Esteem, affection, trust, concern, consideration of cultural and religious responses, and listening are all components of the emotional support given to the pregnant woman and her family. -A discussion of fetal responses to stimuli such as sound, light, maternal posture, and tension, as well as patterns of sleeping and waking, can be helpful. -Other issues of concern that can arise for the pregnant woman and couple include fear of pain, loss of control, and possible birth of the infant before reaching the hospital. -The father's or partner's commitment to the pregnancy, the couple's relationship, and their concerns about sexuality and sexual expression can emerge as issues for many expectant parents. -Providing the prospective mother and father with opportunities to discuss their concerns and validating the normality of their responses can meet their needs to some degree.

Harmful affects of Alcohol

-Ethanol (alcohol) is a powerful teratogen that can have devastating effects on the developing fetus (e.g., fetal alcohol syndrome). -Women who are pregnant or considering pregnancy should totally abstain from alcohol.

Grandparent Adaptation

-Every pregnancy affects all family relationships. -In some family units, expectant grandparents are nonsupportive and may inadvertently decrease the self-esteem of the parents-to-be. -Most grandparents are delighted with the prospect of a new baby in the family. -Their satisfaction and that of the parents come with the realization that continuity between past and present is guaranteed. -The grandparent's presence and support can strengthen family systems by widening the circle of support and nurturance

First Trimester Screenings

-First-trimester screening for chromosomal abnormalities is offered as an option between 11 and 14 weeks. This multiple marker screen includes ultrasound evaluation of nuchal translucency (NT) and biochemical markers—pregnancy-associated placental protein (PAPP-A) and free beta-human chorionic gonadotropin (β-hCG).

Follow-up Interviews

-Follow-up visits are less intensive than the initial prenatal visit. -She is asked about her general emotional and physical well-being, complaints or problems, and questions she may have. -During the third trimester, the nurse needs to assess the parents' understanding of the following: the warning signs that indicate emergencies such as bleeding and abdominal pain, the signs of preterm and term labor, the labor process and anxieties about labor, fetal development, and methods to assess fetal well-being. -A review of the woman's physical systems is appropriate at each visit, and any suggestive signs or symptoms are assessed in depth.

Nutrition

-Good nutrition is important in the maintenance of maternal health during pregnancy and in the provision of adequate nutrients for embryonic and fetal development. -Assessing a woman's nutritional status and weight gain and providing information on nutrition are part of the nurse's responsibilities in providing prenatal care. -Teaching may include discussion about foods high in iron, encouragement to take prenatal vitamins, and recommendations to limit caffeine intake. -Nurses can refer women to a registered dietitian if a need is identified during the nursing assessment.

Things to remember to help prevent UTI

-Harsh, scented or printed toilet paper may cause irritation. -Bubble baths or other bath oils should be avoided because these can irritate the Urethra. -Women should wear underpants and panty hose with a cotton crotch and avoid wearing tight fighting slacks or jeans for long periods. -Anything that allows buildup of heat and moisture in the genital area can foster the growth of bacteria.

Ambivalence

-Having conflicting feelings at the same time—is considered a normal response for people preparing for a new role. -Body sensations, feelings of dependence, or the realization of the responsibilities of child care also can generate such feelings.

Third Trimester Screenings

-If not done earlier in pregnancy, a glucose screen is obtained between 24 and 28 weeks of gestation for women at high risk for gestational diabetes. -Group B strep (GBS) testing is done between 35 and 37 weeks of gestation; cultures collected earlier will not accurately predict GBS status at time of birth. -Tests that are often repeated at 28 weeks include hemoglobin and hematocrit, serologic test for syphilis, and HIV testing. -At 28 weeks, an Rh type and screen for antibodies is performed. -Hematocrit testing may be repeated at 36 weeks in women with anemia and those at risk for peripartum hemorrhage

Gestational Age

-In an uncomplicated pregnancy, fetal gestational age is estimated after the duration of pregnancy and the EDB are determined. -Fetal gestational age is determined from the menstrual history, contraceptive history, pregnancy test results.

Paternal Adaptation Accepting the Pregnancy

-In older societies, the man enacted the ritual couvade; that is, he behaved in specific ways and respected taboos associated with pregnancy and giving birth. -Now some men experience pregnancy-like symptoms, such as nausea, weight gain, and other physical symptoms.

Cultural Influences

-Many cultural variations are found in prenatal care. -A concern for modesty is a deterrent to many women seeking prenatal care. -For many cultural groups, a physician is deemed appropriate only in times of illness. -Western medicine's view of problems in pregnancy may differ from that of members of other cultural groups. -Although pregnancy is considered normal by many, certain practices are expected of women of all cultures to ensure a good outcome.

Preparing for Childbirth

-Many women actively prepare for birth. -Anxiety can arise from concern about safe passage for herself and her child during the birth process. -These feelings persist despite statistical evidence about the safe outcome of pregnancy for mothers and their infants. -Toward the end of the third trimester, breathing is difficult and fetal movements become vigorous enough to disturb the mother's sleep. -Backaches, frequency and urgency of urination, constipation, and varicose veins can become troublesome. -A strong desire to see the end of pregnancy, to be over and done with it, makes women at this stage ready to move on to childbirth.

Car Travel

-Maternal death as a result of injury is the most common cause of fetal death. -The next most common cause is placental separation (abruptio placentae) that occurs because body contours change in reaction to the force of a motor vehicle collision. -A combination lap belt and shoulder harness is the most effective automobile restraint, and both should be used. - The lap belt should be worn low across the hip bones and as snug as is comfortable (Fig. 8-17). The shoulder harness should be worn above the gravid uterus and below the neck to prevent chafing.

Contraindicated Live Vaccines

-Measles (Rubeola and Rubella) -Chickenpox (Varicella) -Sabin (oral) poliomyelitis (No longer used in the U.S.) -Human Papilloma Virus (HPV)

Nägele's rule

-Method for calculating the estimated date of birth (EDB) or "due date". Also called Naegele's rule. -Because the exact date of conception is usually unknown, several formulas have been suggested for calculating the EDB. -After determining the first day of the LMP, subtract 3 months, add 7 days and 1 year; or add 7 days to the LMP and count forward 9 months. -Assumes that the woman has a 28-day menstrual cycle and that the pregnancy occurred on the fourteenth day of the cycle.

Follow-up Visits

-Monthly visits are scheduled routinely during the first and second trimesters, although additional appointments may be made as the need arises. -Starting with week 28, visits are scheduled every 2 weeks until week 36; then visits are scheduled every week until birth unless the health care provider individualizes the schedule.

Education About Maternal and Fetal Changes

-Mothers may be more tolerant of the discomforts related to the continuing pregnancy if they understand the underlying causes. -To be most effective, the materila should be appropriate for the pregnant woman's or couple's ethnicity, culture, and literacy level and the agency's resources.

Physical Activity and Rest

-Norms that regulate physical activity of mothers during pregnancy vary tremendously. -It is important for the nurse to find out the way each pregnant woman views activity and rest.

Initial Visit

-Once the pregnancy is confirmed and the woman's desire to continue the pregnancy has been validated, prenatal care is begun. -Assessment techniques include the interview, physical examination, and laboratory tests.

Women older than 35

-One group consists of women who have many children or who have an additional child during the menopausal period. -The other group consists of women who have deliberately delayed childbearing until their late 30s or early 40s. -The U.S. birth rate for women who are 40 to 44 years of age was 10.2 births per 1000 women, which is the highest rate since 1967. -The birth rate for women 45 to 49 years of age is stable at 0.7 births per 1000 women

Paternal Adaptation

-One man may engage in nurturing behavior; another may feel lonely and alienated as the woman becomes physically and emotionally engrossed in the unborn child. -Some men view pregnancy as a proof of their masculinity and their dominant role. -For most men, pregnancy is a time of preparation for the parental role, fantasy, great pleasure, and intense learning.

Recognizing potential complications

-One of the most important responsibilities of care providers is to alert the pregnant woman to signs and symptoms that indicate a potential complication of pregnancy. -Pregnant women often have difficulty deciding when to report signs and symptoms.

Mother-Child Relationship through pregnancy Developmental Process in 3 phases.

-Phase 1: Accepts the biologic fact of pregnancy. Can state, "I am pregnant." -Phase 2: Accepts the growing fetus as distinct from herself and as a person to nurture. Can state, " I am going to have a baby." -Phase 3: The woman prepares realisticallly for the birth and parenting of the child. Can state, " I am going to be a mother."

Phases characterizing the developmental tasks experienced by the expectant father

-Phase 1: The announcement phase may last from a few hours to a few weeks. Men react to the confirmation of pregnancy with joy or dismay, depending on whether the pregnancy is desired, unplanned, or unwanted. -Phase 2: The moratorium phase, is the period when he adjusts to the reality of pregnancy. They become more introspective and engage in many discussions about their philosophy of life, religion, childbearing, and childrearing practices and their relationships with family members, particularly with their father. -Phase 3: The focusing phase, begins in the last trimester and is characterized by the father's active involvement in both the pregnancy and his relationship with his child. In this phase, the man concentrates on his experience of the pregnancy and begins to think of himself as a father.

Physical Activity

-Physical activity promotes a feeling of well-being and can help reduce anxiety in the pregnant woman. -The U.S. Department of Health and Human Services (USDHHS) recommends 150 minutes of moderate exercise each week during pregnancy for women who are not already active or engaging in moderate exercise.

Adaptation To Pregnancy

-Pregnancy affects all family members, and each family member must adapt to the pregnancy and interpret its meaning in light of his or her own needs. -Additional research is needed on a variety of families to determine if study findings generated in traditional families are applicable to others.

Pregnancy

-Pregnancy spans 9 calendar months. -Health care providers use the concept of lunar months, which last 28 days (or 4 weeks), to describe the duration of pregnancy or gestational age. -Thus normal pregnancy lasts about 10 lunar months, that is, 40 weeks, or 280 days.

Family History

-Provides information about the woman's family, including parents, grandparents, siblings, and children. -These data help identify familial or genetic disorders or conditions that could affect the present health status of the woman or her fetus

Emotional Lability

-Rapid and unpredictable changes in mood. -Increased irritability, explosions of tears and anger, and feelings of great joy and cheerfulness alternate, apparently with little or no provocation. -Profound hormonal changes that are part of the maternal response to pregnancy may be responsible for mood changes.

Follow-up Physical Examination

-Reevaluation is a constant aspect of a pregnant woman's care. -Physiologic changes are documented as the pregnancy progresses and reviewed for possible deviations. -The information provided during the interview and physical examination reflects the status of maternal adaptations. -When any of the findings is suspicious, an in-depth examination is performed.

Caffeine

-Research findings suggest that caffeine intake less than 200 mg per day during pregnancy does not appear to be a major contributing factor to miscarriage or preterm birth. -Women should be aware of the caffeine content of foods, drinks, and certain OTC medications and should intentionally limit their intake as recommended.

Suggesting Alternative Behaviors

-Research has not demonstrated that coitus and orgasm are contraindicated at any time during pregnancy for the obstetrically and medically healthy woman. -However, a history of more than one miscarriage; a threatened miscarriage in the first trimester; impending miscarriage in the second trimester; and PROM, bleeding, or abdominal pain during the third trimester warrant caution regarding coitus and orgasm. -Showing the woman or couple illustrations of the possible variations of coital position is helpful. -During the third trimester, the side-by-side position or any position that places less pressure on the pregnant abdomen and requires less energy may be preferred.

Sexual Counseling

-Sexual counseling of expectant couples includes countering misinformation, providing reassurance of normality, and suggesting alternative behaviors. -Many women merely need permission to be sexually active during pregnancy. -Many other women, however, need to be given information about the physiologic changes that occur during pregnancy, have the myths that are associated with sex during pregnancy dispelled, and participate in open discussions of positions for intercourse that decrease pressure on the gravid abdomen. -Couples whose long-standing problems with sexual dysfunction are intensified by pregnancy are good candidates for sex therapy.

Sibling Adaptation

-Sharing the spotlight with a new brother or sister may be the first major crisis for a child. -A mother with other children must devote time and energy to reorganizing her relationships with these children. -A mother with other children must devote time and energy to reorganizing her relationships with these children. -Sibling responses to pregnancy vary with age and dependency needs.

Social, Experimental, and Occupational History

-Situational factors such as the family's ethnic and cultural background and socioeconomic status are assessed while the history is obtained. -The woman's perception of this pregnancy is explored. -The family support system is determined. -During interviews throughout the pregnancy, the nurse should remain alert for the appearance of potential parenting problems such as depression, lack of family support, and inadequate living conditions. -Coping mechanisms and patterns of interacting are identified. -Before planning for nursing care, the nurse needs information about the woman's decision-making abilities and living habits (e.g., exercise, sleep, diet, diversional interests, personal hygiene, clothing). -Explore attitudes concerning the range of acceptable sexual behavior during pregnancy. -Women should be questioned regarding their occupation, past and present, since this may adversely affect maternal and fetal health.

Posture and Body Mechanics

-Skeletal, musculature, and hormonal changes in pregnancy can predispose the woman to backache and possible injury. -Poor posture and body mechanics contribute to the discomfort and potential for injury.

Immunizations

-Some concern has been raised over the safety of various immunization practices during pregnancy. -Immunization with live or attenuated live virus or live bacterial vaccines is generally contraindicated during pregnancy.

Identifying with the Father Role

-Some men are highly motivated to nurture and love a child. -Others may be more detached or even hostile to the idea of fatherhood.

Clothing

-Some women continue to wear their usual clothes during pregnancy as long as they fit and feel comfortable. -Comfortable, loose clothing is best. Tight bras and belts, stretch pants, garters, tight-top knee socks, body shapers, and other constrictive clothing should be avoided, because tight clothing over the perineum encourages vaginitis and miliaria (heat rash) and impaired circulation in the legs can cause varicosities. -Maternity bras are constructed to accommodate the increased breast weight, chest circumference, and size of breast tail tissue (under the arm). -Maternity support (compression) hose give considerable comfort and promote greater venous emptying in women with large varicose veins. -Comfortable shoes that provide firm support and promote good posture and balance are advisable.

Recognizing Preterm Labor

-Teaching each expectant mother to recognize preterm labor is necessary for early diagnosis and treatment. -Preterm labor occurs after the twentieth week but before the thirty-seventh week of pregnancy. -It consists of uterine contractions that, if untreated, cause the cervix to dilate and efface earlier than normal, resulting in preterm birth.

Adolescents

-Teenage pregnancy is a worldwide problem. The United States has one of the highest teen birth rates among industrialized nations. -Characteristics of programs that make a difference are those that have sustained commitment to adolescents over a long period, involve the parents and other adults in the community, promote abstinence and personal responsibility, and assist adolescents to develop a clear strategy for reaching future goals such as a college education or a career. -These young women also are more likely to smoke and less likely to gain adequate weight during pregnancy. -As a result of these and other factors, babies born to adolescents are at greatly increased risk for LBW, neglect and abuse, serious and long-term disability, and dying during the first year of life.

Killed Virus Vaccines okay during Pregnancy

-Tetanus -Diptheria -Recombinant Hepatitis B -Influenza vaccines -The optimal timing for the vaccine is between 27 and 36 weeks of gestation.

Sibling Responses to Pregnancy

-The 1-year-old infant seems largely unaware of the process. -The 2-year-old child notices the change in the mother's appearance. -Toddlers may exhibit more clinging behavior and revert to dependent behaviors in toilet training or eating. -By age 3 or 4 years, children like to be told the story of their own beginning and accept its being compared to the present pregnancy. -They like to listen to heartbeats and feel the baby moving in utero -School-age children take a more clinical interest in their mother's pregnancy. Tthey still think in concrete terms and base judgments on the here and now, they respond positively to their mother's current good health. -Early and middle adolescents preoccupied with the establishment of their own sexual identity may have difficulty accepting the overwhelming evidence of the sexual activity of their parents. -Late adolescents do not appear to be unduly disturbed. They realize that they soon will be gone from home. -Parents usually report that late adolescents are comforting and act more like other adults than children.

Preparation for Breastfeeding the Newborn

-The American Academy of Pediatrics (AAP) Recommends exclusive breastfeeding for the first 6 months, continued breastfeeding as complementary feedings are introduced, and breastfeeding for at least 1 year and beyond as desired by the mother and infant. -Most women who chose to breastfeed do so because they are aware of the numerous benefits. -Modesty issues, lack of support by the partner, and family, incompatibility with lifestyle, and lack of confidence are among the reasons cited by women who decide to formula feed their infants. -Assessment of breasts during the prenatal period may reveal potential concerns related to breastfeeding. -Breast stimulation is contraindicated in women at risk for preterm labor. -The woman is taught to cleanse the nipples with warm water to prevent blocking of the ducts with dried colostrum. -Soap, ointments, alcohol, and tinctures should not be applied because they remove protective oils that keep nipples supple.

Fetal Health Status

-The assessment of fetal health status includes consideration of fetal movement. -The fetal heart rate (FHR) is checked on routine visits once it has been heard. -Early in the second trimester, the heartbeat may be heard with the Doppler stethoscope. -The heartbeat is counted for 1 minute, and the quality and rhythm are noted. -Later in the second trimester, the FHR can be determined with the fetoscope or Pinard fetoscope. -A normal rate and rhythm are other good indicators of fetal health. -Intensive investigation of fetal health status is initiated if any maternal or fetal complications arise (e.g., maternal hypertension, IUGR, premature rupture of membranes [PROM], irregular or absent FHR, absence of fetal movements after quickening).

Laboratory Tests

-The data yielded by laboratory examination of specimens obtained during the examination add important information concerning the symptoms of pregnancy and the woman's health status. -Specimens are collected at the initial visit so that any abnormal findings can be treated. -Testing for antibody to the human immunodeficiency virus (HIV) is strongly recommended for all pregnant women; this testing must be voluntary and without coercion. -The folate level is measured when indicated. -Cystic fibrosis (CF) carrier screening tests should be offered to all pregnant women; if the woman is a CF carrier, the father of the baby should be tested. -During the pelvic examination, cervical and vaginal smears can be obtained for cytologic studies and for diagnosis of infection. -Recognition of risk factors during pregnancy may indicate the need to repeat some tests at other times. -STIs are common in pregnancy and may have negative effects on mother and fetus. -Careful assessment and thorough screening are essential.

Paternal preparation for Childbirth

-The days and weeks immediately before the expected day of birth are characterized by anticipation and anxiety. -They are able to channel the anxiety and other feelings experienced during the final weeks before birth into productive activities. -Major concerns for the man are getting the mother to the birthing facility in time for the birth and not appearing ignorant. -With the exception of childbirth preparation classes, a man has few opportunities to learn ways to be an involved and active partner in this rite of passage into parenthood. -Birth partners need to be kept informed, supported, and included in all activities in which the mother desires their participation.

Accepting the Pregnancy

-The degree of acceptance is reflected in the woman's emotional responses. -Eventual acceptance of pregnancy parallels the growing acceptance of the reality of a child. -Women who are happy and pleased about their pregnancy have high self-esteem and tend to be confident about outcomes for themselves, their babies, and other family members.

Education for self-Management

-The expectant mother needs information on many topics. -Pregnant women who receive conflicting advice or instruction are likely to grow increasingly frustrated with members of the health care team and the care provided.

Establishing a Paternal Relationship with the Fetus

-The father-child attachment can be as strong as the mother-child relationship, and fathers can be as competent as mothers in nurturing their infants. -Men prepare for fatherhood in many of the same ways that women prepare for motherhood. -Some men become involved by choosing the child's name and anticipating the child's sex if it is not already known.

Quickening

-The first movements of the fetus in the uterus felt by the mother. -It usually occurs between weeks 16 and 20 of gestation and is initially experienced as a fluttering sensation.

Cognitive Restructuring

-The first step in adapting to the maternal role is accepting the idea of pregnancy and assimilating the pregnant state into the woman's way of life. -Intense feelings of ambivalence that persist through the third trimester can indicate an unresolved conflict with the motherhood role

Care Management

-The goal of prenatal care is to promote the health and well-being of the pregnant woman, her fetus, the newborn, and the family. -Although women of middle or high socioeconomic status routinely seek prenatal care, women living in poverty or those who lack health insurance are not always able to use public health care services or gain access to private care. -Lack of culturally sensitive care providers and barriers in communication caused by differences in language also interfere with access to care. -Barriers to obtaining health care during pregnancy include a lack of motivation to seek care, especially for unintended pregnancies; inadequate finances; lack of transportation; unpleasant clinic personnel, facilities, or procedures; inconvenient clinic hours; child care problems; and personal attitudes. -The initial visit usually occurs in the first trimester, with monthly visits through week 28 of pregnancy. Thereafter, visits are scheduled every 2 weeks until week 36 and then every week until birth. -Prenatal care is ideally a multidisciplinary activity in which nurses work with nurse-midwives, nutritionists, physicians, social workers, and others. -If women can be taught healthy lifestyle behaviors and then practice them before conception—specifically, good nutrition, entering pregnancy with as healthy a weight as possible, adequate intake of folic acid, avoidance of alcohol and tobacco use, prevention of sexually transmitted infections (STIs) and other health hazards—a healthier pregnancy may result.

Health History

-The health history includes those physical or surgical procedures that can affect the pregnancy or that can be affected by the pregnancy. -The woman should also describe any previous surgical procedures. -Note any injury involving the pelvis. -Observations are a vital component of the interview process because they prompt the nurse and the woman to focus on the specific needs of the woman and her family.

Initial Visit Interview

-The initial evaluation includes a comprehensive health history emphasizing the current pregnancy, previous pregnancies, the family, a psychosocial profile, a physical assessment, diagnostic testing, and an overall risk assessment. -Two types of data are collected: the woman's subjective appraisal of her health status and the nurse's objective observations. -Observations and information about the woman's partner and/or family are then included in the database.

Initial Physical Examination

-The initial physical examination provides the baseline for assessing subsequent changes. -The interaction requires an unhurried, sensitive, and gentle approach with a matter-of-fact attitude. -The physical examination begins with assessment of vital signs, including blood pressure (BP), height, and weight (for calculation of body mass index [BMI]) -Heart and lung sounds are evaluated, and extremities are examined. -The skin is assessed for changes in pigmentation, rashes, and edema. -Distribution, amount, and quality of body hair are of particular importance because the findings reflect nutritional status, endocrine function, and attention to hygiene. -Particular attention is paid to the size of the uterus because this is an indication of the duration of gestation. -One vaginal examination during pregnancy is recommended; another is usually not done unless indicated for medical reasons.

Primiparous Women

-The number of first-time pregnancies in U.S. women between ages 35 and 40 years has increased significantly over the past three decades. -Reasons for delaying pregnancy include a desire to obtain advanced education, career priorities, and use of better contraceptive measures. -These women choose parenthood. -First-time mothers older than 35 years select the "right time" for pregnancy; this time is influenced by their awareness of the increasing possibility of infertility or of genetic defects in the infants of older women. -Women 35 years of age and older are more likely than younger primiparas to have infants with chromosomal abnormalities, LBW infants, preterm birth, abruptio placentae, and multiple gestation. -In addition, in women ages 35 years or older, there is an increased risk for maternal mortality from hemorrhage, infection, embolisms, hypertensive disorders of pregnancy, cardiomyopathy, and strokes.

Fetal Laboratory Tests

-The number of routine laboratory tests done during follow-up visits in pregnancy is limited. -Other diagnostic tests, such as amniocentesis, are available to assess the health status of both the pregnant woman and the fetus

Nutritional History

-The nutritional status of a pregnant woman has a direct effect on the growth and development of the fetus. -It is essential that obese women receive counseling about weight gain, nutrition, and food choices. -Women with a history of bariatric surgery are nutritionally at risk and should be followed closely throughout pregnancy to promote maternal and fetal well-being

Reordering Paternal Personal Relationships

-The partner's main role in pregnancy is to nurture the pregnant woman and to respond supportively to her feelings of vulnerability. -The partner's support indicates involvement in the pregnancy and preparation for attachment to the child. -Feelings of rivalry may be unconscious and not verbalized but expressed in subtle behaviors.

Current Pregnancy

-The presumptive signs of pregnancy, such as nausea and vomiting, may be of great concern to the woman. -A review of symptoms she is experiencing and how she is coping with them helps establish a database to develop a plan of care.

Identifying with the Mother Role

-The process of identifying with the mother role begins early in each woman's life when she is being mothered as a child. -Their high motivation to become a parent promotes acceptance of pregnancy and eventual prenatal and parental adaptation.

Sexual Expression during Pregnancy

-The sexual relationship is affected by physical, emotional, and interactional factors, including misinformation about sex during pregnancy, sexual dysfunction, and physical changes in the woman. -As pregnancy progresses, changes in body shape, body image, and levels of discomfort influence both partners' desire for sexual expression. -Partners need to feel free to discuss their sexual responses during pregnancy with each other and with their health care provider.

Fetal Assesment

-Toward the end of the first trimester, before the uterus is an abdominal organ, the fetal heart tones (FHTs) can be heard with an ultrasound fetoscope or an ultrasound stethoscope. -The health status of the fetus is assessed at each visit for the remainder of the pregnancy.

Travel

-Travel is not contraindicated for low risk pregnant women. -Travel to areas where medical care is poor, water is untreated, or malaria is prevalent should be avoided if possible. -Women who contemplate foreign travel should be aware that many health insurance carriers do not cover birth in a foreign setting or even hospitalization for preterm labor. -Vaccinations for foreign travel may be contraindicated during pregnancy.

Emotional Response

-Virtually all cultures emphasize the importance of maintaining a socially harmonious and agreeable environment for the pregnant woman. -A lifestyle with minimal stress is important in ensuring a successful outcome for the mother and baby.

Estimating Date of Birth

-When pregnancy is confirmed, the woman's first question usually concerns when she will give birth. -To promote a more positive perception of both pregnancy and birth, the term estimated date of birth (EDB) is now used. -Accurate dating of pregnancy and calculation of the EDB have implications for timing of specific prenatal screening tests, assessing fetal growth, and making critical decisions for managing pregnancy complications. -Nägele's rule assumes that the woman has a 28-day menstrual cycle and that the pregnancy occurred on the fourteenth day of the cycle. -Only about 5% of pregnant women give birth spontaneously on the EDB as determined by Nagele's rule. -Most women give birth during the period extending from 7 days before to 7 days after the EDB.

Maternal Adaptation

-Women of all ages use the months of pregnancy to adapt to the maternal role—a complex process of social and cognitive learning. -Pregnancy is a maturational milestone that can be stressful but also rewarding as the woman prepares for a new level of caring and responsibility. -She moves gradually from being self-contained and independent to being committed to a lifelong concern for another human being. -This growth requires mastery of certain developmental tasks: accepting the pregnancy, identifying with the role of mother, reordering the relationships between herself and her mother and between herself and her partner, establishing a relationship with the unborn child, and preparing for the birth experience.

Gestational carrier ( embryo host)/ surrogate mother/ indications

-congenital absence or surgical removal of uterus -reproductive impaired uterus -myomas -uterine adhesions -other congenital abnormalities -medical condition that might me life threatening during pregnancy, such as severe heart, kidney, liver disease - gay male couple seeking genetic offspring

urinary urgency and frequency interventions

-drinking no less than 2000 ml of fluid during the day - limiting fluid intake in the evening - voiding at regular intervals - sleeping side-lying at night - wearing perineal pads - performing kegels

Donor oocyte/ indications

-early menopause -surgical removal of ovaries -congenitally absent ovaries -autosomal or sex-linked disorders -lack of fertilization in repeated IVF attempts because of subtle oocyte abnormalities or defects on oocyte-spermatozoa interaction

nausea/vomiting interventions

-eating dry crackers before arising -avoiding brushing teeth immediately after arising -eating small, frequent, low-fat meals during the day - drinking liquids between meals rather than at meals - avoiding fried foods and spicy foods - asking the health care providing about acupuncture, herbal remedy - taking antiemetic medications

Potential Complications that need attention immediately

-f she senses that something is wrong, she should call her care provider immediately. -Several signs and symptoms must be discussed more extensively. -These include vaginal bleeding, alteration in fetal movements, symptoms of preeclampsia, rupture of membranes, and preterm labor.

what should be the folic acid intake

0.4 mg in fortified foods

What is the estimated due date (EDD) of a pregnant woman with last menstrual period onset of May 10th? Record date formatted mm/dd.

02/17 The due date (EDD) is calculated by subtracting three months from the first day of the last menstrual period and adding seven days. This is known as Naegele's rule and is based on the woman having a normal 28-day menstrual cycle.

A gravida GI P0000, is having her first prenatal physical examination. Which of the following assessments should the nurse inform the client that she will have that day? 1. Pap Smear 2. Mammogram 3. Glucose Challenge Test 4. Biophysical Profile 5. Complete Blood Count

1 & 5. A glucose test and biophysical profile won't be done until later in the pregnancy.

What does each homologous pair of chromosomes contain?

1 chromosome from MOM and 1 chromosome from DAD

Serving Size of Diary:

1 cup = 1 cup (8 oz) milk or yogurt; 1-1/2 oz natural cheese; 2 oz processed cheese; 1 cup frozen yogurt or 1-1/2 cups ice cream; 1 cup Calcium-Fortified Soy Milk

Serving Size of Fruits:

1 cup = 1 cup raw, frozen, or canned fruit; 1 cup 100% juice; or 1/2 cup dried fruit

Serving Size of Vegetables:

1 cup = 2 cups raw leafy greens; 1 cup of other vegetables, raw or cooked; or 1 cup of vegetable juice

Serving Size of Grains:

1 ounce = 1 slice bread; 3/4 to 1 cup ready-to-eat cereal; 1/2 cup cooked rice, pasta, or cooked cereal; 1 small muffin; or 1 small tortilla

Serving Size of Protein

1 oz = 1 oz (30 g) cooked meat, poultry, or fish; 1/4 cup cooked dried beans or peas; 1 egg; 1 tablespoon (15 ml) peanut butter, 1/2 oz nuts or seeds

A nurse is discussing the serving sizes in the grain food group with a new prenatal client. Which of the following foods equals 1 oz serving size from the grain group? Select all that apply. 1. 1 bagel. 2. 1 slice of bread. 3. 1 cup cooked pasta. 4. 1 tortilla. 5. 1 cup dry cereal.

1 slice of bread. 4. 1 tortilla. 5. 1 cup dry cereal.

Serving Size of Oils:

1 teaspoon = 1 teaspoon liquid oil (e.g., olive, canola, sunflower, safflower, peanut, soybean, cottonseed) or Soft Margarine; 1 tablespoon Mayonnaise-type or Italian Salad Dressing; 3/4 tablespoon Thousand Island Salad Dressing; 1/2 oz dry-roasted nut or peanuts

What are the types of primary germ layers?

1) Ectoderm - Upper layer 2) Mesoderm - Middle layer 3) Endoderm - Outer layer

What are the 4 steps of conception?

1) Gametogenesis 2) Ovulation 3) Fertilization 4) Implantation

What are the 3 stages of Intrauterine development?

1) Ovum or Preembryonic 2) Embryo 3) Fetus

What are the 2 proactive layers of the ovum?

1) Zona Pellucida - Thick inner layer 2) Corona Radiata - Outer layer

Which of the following findings in a 8-week gestation client, G2 P1001, should be nursing highlight for the nurse midwife? (select all) 1. Body mass index of 17 kg/mm2 2. Rubella titer of 1:8 3. Blood pressure of 100/60 mm Hg 4. Hematocrit of 30% 5. Hemoglobin of 13.2 g/dL

1, 2, & 4. The BMI of 17 is a concern. This client is entering her pregnancy underweight. The rubella titer results need to be reported. The hematocrit result is below normal

A antenatal client is informing the nurse of her prenatal signs and symptoms. Which of the following findings would the nurse determine are presumptive signs of pregnancy? (select all) 1. Amenorrhea 2. Breast tenderness 3. Quickening 4. Frequent urination 5. Uterine growth

1, 2, 3, & 4. The correct answers are signs of presumptive pregnancy. Uterine growth is a probable sign of pregnancy. Probable = Signs that are objective but not absolute and include alterations in uterine shape and size and softening of the cervix.

A nurse is advising a pregnant woman about the danger signs of pregnancy. The nurse should teach the mother that she should notify the physician immedi ately if she experiences which of the following signs/symptoms? Select all that apply. 1. Convulsions. 2. Double vision. 3. Epigastric pain. 4. Persistent vomiting. 5. Polyuria.

1, 2, 3, and 4 are correct. 1. Convulsions are a danger sign of pregnancy. 2. Double vision is a danger sign of pregnancy. 3. Epigastric pain is a danger sign of pregnancy. 4. Persistent vomiting is a danger sign of pregnancy. 5. Polyuria is not highlighted as a danger sign of pregnancy.

A nurse is working in the prenatal clinic. Which of the following findings seen in third-trimester pregnant women would the nurse consider to be within normal limits? Select all that apply. 1. Leg cramps. 2. Varicose veins. 3. Hemorrhoids. 4. Fainting spells. 5. Lordosis.

1, 2, 3, and 5 are correct. 1. Leg cramps are normal, although the client's diet should be assessed. 2. Varicose veins are normal, although client teaching may be needed. 3. Hemorrhoids are normal, although client teaching may be needed. 4. Fainting spells are not normal, although the client may feel faint when rising quickly from a lying position. 5. Lordosis, or change in the curvature of the spine, is normal, although pa tient teaching may be needed.

A woman has just completed her first trimester. Which of the following fetal struc tures can the nurse tell the woman are well formed at this time? Select all that apply. 1. Genitals. 2. Heart. 3. Fingers. 4. Alveoli. 5. Kidneys.

1, 2, 3, and 5 are correct. 1. The genitalia are formed by the end of the first trimester. 2. The heart is formed by the end of the first trimester. 3. The fingers are formed by the end of the first trimester. 4. The alveoli will not be formed until well into the second trimester. 5. The kidneys are formed by the end of the first trimester.

A woman is planning to become pregnant. Which of the following actions should she be counseled to take before stopping birth control? Select all that apply. 1. Take a daily multivitamin. 2. See a medical doctor. 3. Drink beer instead of vodka. 4. Stop all over-the-counter medications. 5. Stop smoking cigarettes.

1, 2, and 5 are correct. 1. It is very important that women, be fore attempting to become pregnant, begin taking daily multivitamin tablets. 2. Women who wish to become preg nant should first see a medical doctor for a complete check-up 3. Women who wish to become pregnant should refrain from drinking any alcoho 4. Women who wish to become pregnant should ask an obstetrician/gynecologist which over-the-counter medications should be avoided. Some—for example, acetaminophen—are safe to take, while others are not. 5. Women who wish to become preg nant should be counseled to stop smoking.

The nurse is providing counseling to a group of sexually active single women. Most of the women have expressed a desire to have children in the future but not within the next few years. Which of the following actions should the nurse suggest the women take to protect their fertility for the future? (Select all that apply) A. Use condoms during intercourse B. Refrain from smoking cigarettes C. Maintain an appropriate weight for height D. Exercise in moderation E. Refrain from drinking carbonated beverages

1,2,3,4 - Condoms should be worn during sexual contacts to prevent infection with a sexually transmitted disease, which can affect the long-term health of the woman's reproductive system. - Women who smoke have a higher incidence of infertility than those who do not smoke. - Women who are either overweight or underweight have increased incidence of infertility. - Body mass index (BMI) is related to the amount of exercise a woman engages in. Those who exercise excessively are more likely to have a very low BMI and those who rarely exercise, to be obese. Since fertility is related to body weight, it is recommended that women exercise in moderation

A Roman Catholic couple is infertile. Their healthcare practitioner advises them that their best chance of getting pregnant is via in-vitro fertilization with a mixture of the man's sperm and donor sperm. Which of the following issues, related to this procedure, should the nurse realize may be in conflict with the couple's religious beliefs? (Select all that apply) A. The man will ejaculate by masturbation into a specially designed condom. B. The woman may become pregnant with donor sperm. C. Fertilization is occurring in the artificial environment of the laboratory. D. More embryos may be created than will be used to inseminate the woman. E. The woman will receive medications to facilitate the ripening of her ova.

1,2,3,4 - Masturbation and the use of a condom, even for the express purpose of creating life, are considered sins in the Catholic tradition. - Procreation with the man's sperm alone is unlikely. The addition of the donor sperm makes this unacceptable in the eyes of the Catholic Church because a woman should become pregnant only by her husband. - According to the precepts of the Catholic Church, fertilization may take place only within the body of the woman. - It is immoral in the Catholic tradition to create more embryos than are needed to conceive.

A couple who has been attempting to become pregnant for 5 years is seeking assistance from an infertility clinic. The nurse assesses the clients' emotional responses to their infertility. Which of the following responses would the nurse expect to find? (Select all that apply) A. Anger at others who have babies B. Feeling of failure because they can't make a baby C. Sexual excitement because they want so desperately to conceive a baby D. Sadness because of the perceived loss of being a parent E. Guilt on the part of one partner because he or she is not able to give the other a baby

1,2,4,5 - Infertility clients often express anger at others who are able to conceive. - Infertility clients often express a feeling of personal failure. - Sadness is another common feeling expressed by infertility clients. - Guilt is commonly expressed by infertility clients

When assessing the psychological adjustment of an 8-week gravida, which of the following would the nurse expect to see signs of? 1. Ambivalence 2. Depression 3. Anxiety 4. Ecstasy

1. Ambivalence should be expected.

The nurse notes each of the following findings in a 10-week gestation client. Which of the findings would enable the nurse to tell the client that she is positively pregnant? 1. Fetal heart rate via Doppler 2. Positive pregnancy test 3. Positive Chadwick's sign 4. Montgomery gland enlargements

1. Hearing a fetal heart rate is a positive sign of pregnancy.

The nurse has taken a health history on four primigravid clients at their first prenatal visits. It is high priority that which of the clients receives nutrition counseling? 1. The woman diagnosed with phenylketonuria 2. The woman who has Grave's disease 3. The woman with Cushing's syndrome 4. The woman has Myasthenia Gravis.

1. The client with PKU must receive counseling from a registered dietician.

Because nausea and vomiting are such common complaints of pregnant women, the nurse provides anticipatory guidance to a 6-week gestation client by telling her to do which of the following? 1. Avoid eating greasy foods 2. Drink some orange juice before rising 3. Consume 1 teaspoon of nutmeg each morning 4. Eat 3 large meals plus a bedtime snack

1. Greasy foods should be avoided. Saltine crackers should be eaten before rising. Drinking orange juice has not been recommended. Ginger can help alleviate nausea and vomiting. It is also recommended that mothers eat small, frequent meals throughout the day.

A client enters the prenatal clinic. She states that she believes she is pregnant. Which of the following hormone elevations will indicate a high probability that the client is pregnant? 1. Chorionic gondotropic 2. Oxytocin 3. Prolactin 4. Luteinizing hormone

1. High levels of the hormone chorionic gonadotropin in the blood stream and urine of the woman is a probably sign of pregnancy.

The nurse is providing anticipatory guidance to a woman in her second trimester regarding signs/symptoms that are within normal limits during the latter half of the pregnancy. Which of the following comments by the client indicates that the teaching was successful? 1. "During the third trimester I may experience frequent urination." 2. "During the third trimester I may experience heartburn." 3. "During the third trimester I may experience nagging back pain." 4. "During the third trimester I may experience persistent headache." 5. "During the third trimester I may experience blurred vision."

1. "During the third trimester I may experience frequent urination." 2. "During the third trimester I may experience heartburn." 3. "During the third trimester I may experience nagging back pain."

meiotic division

1. (replicated DNA) during fetal life but remained suspended at this stage until puberty 2. begins at ovulation ( 2nd division), which ovum doesnt complete unless fetilization occurs

Which finding would the nurse view as normal when evaluating the laboratory reports of a 34-week gestation client? 1. Anemia. 2. Thrombocytopenia. 3. Polycythemia. 4. Hyperbilirubinemia.

1. Anemia is an expected finding.

During a preconception counseling session, the nurse encourages a couple to pre pare a birth plan. Which of the following is the most important goal for this action? 1. Promote communication between the couple and health care professionals. 2. Enable the couple to learn about the types of medicine used in labor. 3. Provide the couple with a list of items that they should put in a bag for labor. 4. Give the high-risk couple a sense of control over having to have a cesarean.

1. Birth plans help to facilitate communication between couples and their health care providers.

A nurse is discussing diet with a pregnant woman. Which of the following foods should the nurse advise the client to avoid consuming during her pregnancy? 1. Bologna. 2. Cantaloupe. 3. Asparagus. 4. Popcorn.

1. Bologna should not be consumed during pregnancy unless it is thor oughly cooked.

During a prenatal visit, a gravid client is complaining of ptyalism. Which of the following nursing interventions is appropriate? 1. Encourage the woman to brush her teeth carefully. 2. Advise the woman to have her blood pressure checked regularly. 3. Encourage the woman to wear supportive hosiery. 4. Advise the woman to avoid eating rare meat.

1. Clients who experience ptyalism have an excess of saliva. They should be advised to be vigilant in the care for their teeth and gums. Ptyalism is often accompanied by gingivitis and nausea and vomiting.

A nurse, who is providing nutrition counseling to a new gravid client, advises the woman that a serving of meat is approximately equal in size to which of the following items? 1. Deck of cards. 2.paper book 3. clenched fist 4. large tomato

1. Deck of cards.

A 38-week gestation client, Bishop score 1, is advised by her nurse midwife to take evening primrose daily. The office nurse advises the client to report which of the following side effects that has been attributed to the oil? 1. Skin rash. 2. Pedal edema. 3. Blurred vision. 4. Tinnitus.

1. Evening primrose has been shown to cause skin rash in some women.

A pregnant woman must have a glucose challenge test (GCT). Which of the following should be included in the preprocedure teaching? 1. Fast 8-16 hours prior to the test 2. Bring urine specimen to the laboratory on the day of the test 3. consume a solid sugar cube immediately upon akening 4.. drink 16 oz of water 1 hr prior to the test 5. smoke no sigarettes the day ofthe test

1. Fast 8-16 hours prior to the test 5. smoke no sigarettes the day ofthe test

A mother is experiencing nausea and vomiting every afternoon. The ingestion of which of the following spices has been shown to be a safe complementary therapy for this complaint? 1. Ginger. 2. Sage. 3. Cloves. 4. Nutmeg.

1. Ginger has been shown to be a safe antiemetic agent for pregnant women.

A father experiencing couvade syndrome is likely to exhibit which of the following symptoms/behaviors? 1. Heartburn. 2. Promiscuity. 3. Hypertension. 4. bloating 5. abdominal pain

1. Heartburn. 4. bloating 5. abdominal pain

When should a couple seek specialized fertility treatment?

1. If the couple is unable to achieve pregnancy after one year of unprotected sex 2. at 6 months unable to get pregnant if age >35 years or have a known risk factor such as endometriosis, or male subfertility

Presumptive proof of ovulation

1. OTC ovulation kits (detects luteinizing hormone surge 24-36 hours prior to ovulation) 2. PROGESTERONE level 1 week prior to expected menses elevated 3. Assessment of basal temp. (occurs in response to progesterone) 4. Hormone analysis (hypothalamic-pituitary-ovarian axis) 5. cervical mucus analysis 6. ultrasound mitelschmerz and midcycle spotting - unreliable presumptive evidence of ovulation

A gravida's fundal height is noted to be at the xiphoid process. The nurse is aware that which of the following fetal changes is likely to be occurring at the same time in the pregnancy? 1. Surfactant is formed in the fetal lungs. 2. Eyes begin to open and close. 3. Respiratory movements begin. 4. The spinal column is completely formed.

1. Surfactant is usually formed in the fetal lungs by the 36th week.

Which of the following vital sign changes should the nurse highlight for a pregnant woman's obstetrician? 1. Prepregnancy blood pressure (BP) 100/60 and third trimester BP 140/90. 2. Prepregnancy respiratory rate (RR) 16 rpm and third trimester RR 22 rpm. 3. Prepregnancy heart rate (HR) 76 bpm and third trimester HR 88 bpm. 4. Prepregnancy temperature (T) 98.6ºF and third trimester T 99.2ºF.

1. The blood pressure should not ele vate during pregnancy. This change should be reported to the health care practitioner.

The nurse is assisting a couple to develop decisions for their birth plan. Which of the following decisions should be considered nonnegotiable by the parents? 1. Whether or not the father will be present during labor. 2. Whether or not the woman will have an episiotomy. 3. Whether or not the woman will be able to have an epidural. 4. Whether or not the father will be able to take pictures of the delivery.

1. The presence of the father at delivery should be nonnegotiable.

The nurse asks a woman about how the woman's husband is dealing with the preg nancy. The nurse concludes that counseling is needed when the woman makes which of the following statements? 1. "My husband is ready for the pregnancy to end so that we can have sex again." 2. "My husband has gained quite a bit of weight during this pregnancy." 3. "My husband seems more worried about our finances now than before the pregnancy." 4. "My husband plays his favorite music for my belly so the baby will learn to like it."

1. The woman implies that she and her husband are not having sex. There is no need to refrain from sexual inter course during a normal pregnancy— so the woman and her husband need further counseling.

A gravid woman who recently emigrated from mainland China is being seen at her first prenatal visit. She was never vaccinated in her home country. An injection to prevent which of the following communicable diseases should be administered to the woman during her pregnancy? 1. Influenza. 2. Mumps. 3. Rubella. 4. Varicella.

1. The woman should receive the in fluenza injection. The nasal spray, however, should not be administered to a pregnant woman.

A gravid woman and her husband inform the nurse that they have just moved into a three-story home that was built in the 1930s. Which of the following is critical for the nurse to advise the woman in order to protect the unborn child? 1. Stay out of any rooms that are being renovated. 2. Drink water only from the hot water tap. 3. Refrain from entering the basement. 4. Climb the stairs only once per day.

1. The woman should stay out of rooms that are being renovated.

The nurse is reading an article that states that the maternal mortality rate in the United States in the year 2014 was 14. Which of the following statements would be an accurate interpretation of the statement? 1. There were 14 maternal deaths in the United States in 2014 per 100,000 live births. 2. There were 14 maternal deaths in the United States in 2014 per 100,000 women of childbearing age. 3. There were 14 maternal deaths in the United States in 2014 per 100,000 pregnancies. 4. There were 14 maternal deaths in the United States in 2014 per 100,000 women in the country.

1. There were 14 maternal deaths in the United States in 2014 per 100,000 live births.

The nurse is teaching a couple about fetal development. Which statement by the nurse is correct about the morula stage of development? 1. "The fertilized egg has yet to implant into the uterus." 2. "The lung fields are finally completely formed." 3. "The sex of the fetus can be clearly identified." 4. "The eye lids are unfused and begin to open and close."

1. This is a true statement. In the morula stage, about 2 to 4 days after fertilization, the fertilized egg has not yet implanted in the uterus.

Why is it essential that women of childbearing age be counseled to plan their pregnancies? 1. Much of the organogenesis occurs before the missed menstrual period. 2. Insurance companies must preapprove many prenatal care expenditures. 3. It is recommended that women be pregnant no more than 3 times during their lifetime. 4. The cardiovascular system is stressed when pregnancies are less than 2 years apart.

1. This statement is true. Organogenesis begins prior to the missed menstrual period.

The nurse is caring for a pregnant client who is a vegan. Which of the following foods should the nurse suggest the client consume as substitutes for restricted foods? 1. Tofu, legumes, broccoli. 2. Corn, yams, green beans. 3. Potatoes, parsnips, turnips. 4. Cheese, yogurt, fish.

1. Tofu, legumes, and broccoli are excel lent substitutes for the restricted foods.

A pregnant client is lactose intolerant. Which of the following alternative calcium rich foods could this woman consume? 1. Turnip greens. 2. Green beans. 3. Cantaloupe. 4. Nectarines.

1. Turnip greens are calcium-rich.

a non stress test is performed on a client who is pregnant, and the results of the test indicate non-reactive findings. the health care provider prescribes a contraction stress test, and the results are documented as negative. how should the nurse document the finding? 1. a normal stress test result 2. an abnormal test result 3. a high risk for fetal demise 4. the need for a cesarean section

1. a normal stress test result

the nurse is performing an assessment on a client who suspects that she is pregnant and is checking the client for probable signs of pregnancy. the nurse should assess for which probable signs of pregnancy? select all that apply 1. ballottement 2. Chadwick's sign 3. uterine enlargement 4. positive pregnancy test 5. fetal heart rate detected by a non electronic device 6. outline of fetus via radiography or ultrasonography

1. ballottement 2. Chadwick's sign 3. uterine enlargement 4. positive pregnancy test

A woman states that she frequently awakens with "painful leg cramps" during the night. Which of the following assessments should the nurse make? 1. exercise schedule. 2. Goodell's sign. 3. Hegar's sign. 4. dietary intake

1. exercise schedule.

characteristics enabling fetus to obtain sufficient oxygen from the maternal blood

1. fetal Hg has a high affinity for O2 and carries 20-30% more than mothers 2. Hg concentration of fetus is 50% greater than mothers 3. fetal HR 110-160, making cardiac output per unit of body weight higher

other factors affecting male fertility

1. genetic disorders (Kleinefekter syndrome) 2. decrease in libido ( heroine, mathadone, SSRI) and barbiturates 3. impotence - alcohol, antihypertensive medication

testa for impaired fertility

1. hysterosalpingogram 2. sonohysterogram 3. basal body temperature 4. ovulation detection kit 5. assessment of cervical mucus 6. ultrasound diagnosis of follicular collapse 7. serum assay of plasma progesterone 8. endometrial biopsy 9. hysteroscopy 10. laparascopy

assisted reproductive technology

1. intrauterine insemination 2. in vitro fertilization 3. gamete intrafallopian transfer and zygote intrafallopian transfer 4. oocyte donation 5. embryo donation 6. surrogate mothers and embryo hosts 7. therapeutic donor insemination

risk factors for endometriosis

1. low body mass index 2. alcohol and tobacco use 3. caucasian race

factors decreasing uterine flow

1. low maternal arterial pressure 2. contractions of the uterus 3. maternal supine position

key components of nutritional care

1. nutritional assessment -weight for height, adequacy of dietary intake, preexisting issues 2. diagnosis of nutritional related problems or risk factors such as diabetes 3. interventions based on individual's dietary goals and plan to promote adequate weight gain 4. evaluation during preconception

factors affecting male infertility/ poor sperm quality

1. substance abuse, especially tobacco 2. age 3. STI 4. exposure to workplace hazards such as radiation ot toxic substances 5. exposure of scrotum to high temperatures 6. nutritional deficiencies 7. obesity 8. antisperm antibodies

medical therpy of infectility/ correcting preexisting factors

1. treat infections with antimicrobial formulations 2. surgery or hysterosalpingogram to correct tubal blockage or pelvic distortion 3. laparoscopic removal of endometrial adhesions and implants 4. draining of hydrosalpinges ( endometrial fluid pockets) 5. weight normalization 6. drug therapy for male infertility 7. correction of thyroid and adrenal glands problems with meds 8. surgery to correct varicocele, blockages or tumors in males 9. FSH, gonadotropins, clomiphene to stimulate spermatogenesis in men with hypogonadism

factors affecting male fertility/structural or hormonal disorders

1. undescended testes 2. hypospadias 3. varicocele 4. obstructive lesions of the vas deferens or epididymis 5. low testosterone levels 6. hypopituitarism 7. endocrine disorders 8. testicular damage caused by mumps 9. retrograde ejaculation

What is the size of the fetal brain at term?

1/4 the size of an adult head

Which time span delineates the appropriate length for a normal pregnancy? 9 lunar months, 8.5 calendar months, 39 weeks, 272 days 10 lunar months, 9 calendar months, 40 weeks, 280 days 9 calendar months, 10 lunar months, 42 weeks, 294 days 9 calendar months, 38 weeks, 266 days

10 lunar months, 9 calendar months, 40 weeks, 280 days Pregnancy lasts approximately 10 lunar months, 9 calendar months, 40 weeks, 280 days. Length of pregnancy is computed from the first day of the last menstrual period (LMP) until the day of birth. Nine lunar months is just short of a term pregnancy, and 294 days is longer than the average length of a pregnancy and would be considered postterm. Because conception occurs approximately 2 weeks after the first day of the LMP, the length described in D represents the postconception age of 266 days or 38 weeks. Postconception age is used in the discussion of fetal development.

doppler heart beatingq

10-12 weeks

women with single fetus and normal BMI gain

11.3-15.9 kg during pregnancy

islats of langherhan

12 week

What week is the structure of the placenta complete?

12th week

urine secreted

12th weeks

When does MOM start perceiving fetal moments?

16 to 20 WEEKS

primitive alveoli and vascular structure are formed

16-24 weeks

A woman typically consumes 1300 calories per day prior to becoming pregnant. How many calories should the patient consume during her pregnancy?

1600 A pregnant woman needs an additional 300 calories during pregnancy and 500 additional calories for lactation postpartum each day. Therefore, a woman who consumed 1300 calories per day prior to pregnancy would need to consume 1600 calories per day during her pregnancy and 1800 calories per day during postpartum lactation.

Protein (g) Recommendation for Pregnancy:

1st Trimester- same as non pregnant 2nd & 3rd Trimester- 71 g

Calorie Recommendation during Pregnancy:

1st Trimester- same as non-pregnant 2nd Trimester- +340 calories (kcals) 3rd Trimester- nonpregnant needs plus +452 calories (kcals)

A client is in the 10th week of her pregnancy. Which of the following symptoms would the nurse expect the client to exhibit? 1. Backache 2. Urinary Frequency 3. Dyspnea on exertion 4. Fatigue 5. Diarrhea

2 & 4. Backaches usually do not develop until the second trimester of pregnancy and dyspnea is associated with the third trimester of pregnancy. Diarrhea is not normally heard in prenatal clients.

The nurse working in an outpatient obstetric office assesses four primigravid clients. Which of the client findings should the nurse highlight for the physician? 1. 17 weeks gestation, denies feeling fetal movement 2. 24 weeks gestation, fundal height of the umbilicus 3. 27 weeks gestation, salivates excessively 4. 34 weeks gestation, experiences uterine cramping 5. 37 weeks gestation, complains of hemorrhoidal pain

2 & 4. The fundal height at 24 weeks should be 4 cm above the umbilicus. The fundal height at the level of the umbilicus is expected at 20 weeks gestation. The woman with uterine cramping may be going into uterine labor.

The nurse is assessing the laboratory report of a 40-week gestation client. Which of the following values would the nurse expect to find elevated about prepregnancy levels? (select all) 1. Glucose 2. Fibrinogen 3. Hematocrit 4. Bilirubin 5. White blood cells

2 & 5. Fibrinogen levels will be elevated slightly in a 40-week pregnant woman because coagulation factors like fibrinogen increase to help prevent excessive blood loss during delivery. A 40-week pregnant woman's white blood cell count will be elevated above normal as a means of protecting her body from infection.

Daily Amount if Fruits

2 cups

What happens at the time of implementation?

2 fetal membranes that will surround the developing embryo will begin to form

dizigotic twins

2 mature ovas are produced in the ovarian cycle, both have potential to be fertilized by separate sperm rezults in 2 zygotes they have 2 amnions, 2 chorions, 2 placentas fraternal twins

A third-trimester client is being seen for routine prenatal care. Which of the fol lowing assessments will the nurse perform during the visit? Select all that apply. 1. Blood glucose. 2. Blood pressure. 3. Fetal heart rate. 4. Urine protein. 5. Pelvic ultrasound.

2, 3, and 4 are correct. 1. Urine glucose is performed at each visit, not the blood glucose. 2. The blood pressure is assessed at each prenatal visit. 3. The fetal heart rate is assessed at each prenatal visit. Depending on the equipment available, it will be assessed mechanically via Doppler or manually via fetoscope. The fetal heart is audible via Doppler many weeks before it is audible via fetoscope. 4. Urine protein is performed at each prenatal visit.

Nurses working in a midwifery office have attended a conference to learn about factors that increase a woman's risk of becoming infertile. To evaluate the nurses' learning, the conference coordinator tests the nurses' knowledge at the conclusion of the seminar. Which of the following problems should the nurses state increase a client's risk of developing infertility problems? (Select all that apply) A. Women who have menstrual cycles that are up to 30 days long B. Women who experience pain during intercourse C. Women who have had pelvic inflammatory disease D. Women who have excess facial hair E. Women who have menstrual periods that are over 5 days long

2,3,4 - Dyspareunia, or pain during intercourse, may be a symptom of a sexually transmitted infection (STI) or of endometriosis. Both STIs or of endometriosis can adversely affect a woman's fertility. - A woman who has had pelvic inflammatory disease (PID) is much more likely to have blocked fallopian tubes than a woman who has never had PID. - Women who have facial hair (hirsutism) often have polycystic ovarian syndrome (PCOS). PCOS patients frequently have irregular menses, elevated serum cholesterol, and insulin resistance. Women with PCOS are often infertile.

A female client seeks care at an infertility clinic. Which of the following tests may the client undergo to determine what, if any, infertility problem she may have? (Select all that apply) A. Chorionic villus sampling B. Endometrial biopsy C. Hysterosalpingogram D. Serum progesterone assay E. Postcoital test

2,3,4,5 - Endometrial biopsy is performed about 1 week following ovulation to detect the endometrium's response to progesterone. - Hysterosalpingogram is performed after menstruation to detect whether or not the fallopian tubes are patent. - Serum progesterone assay is performed about 1 week following ovulation to determine whether or not the woman's corpus luteum produces enough progesterone to sustain a pregnancy. - Postcoital tests are performed about 1 to 2 days before ovulation to determine whether healthy sperm are able to survive in the cervical mucus.

Daily Amount of Vegetables:

2-1/2 to 3 cups Dark Green and Red/Orange vegetables should be included often

A client enters the prenatal clinic. She states that she missed her period yesterday and used a home pregnancy test this morning. She states that the results were negative but "I still think I am pregnant." Which of the following statements would be appropriate for the nurse to ask at this time? 1. "Your period is probably just irregular." 2. "We could do a blood test to check." 3. "Home pregnancy test results are very accurate." 4. "My recommendation would be to repeat the test in one week."

2. Serum pregnancy tests are more sensitive than urine tests.

A 16-year old, GI P000, is being seen at her 10-week gestation visit. She tells the nurse that she felt the baby move in the morning. which of the following responses by the nurse is appropriate? 1. "That is very exciting. The baby must be very healthy." 2. "Would you please describe what you felt for me?" 3. "That is impossible. The baby is not big enough yet." 4. "Would you please let me see if I can feel the baby?"

2. The other statements are inappropriate.

The nurse plans to provide anticipatory guidance to a 10 week gravid client who is being seen in the prenatal clinic. Which of the following information should be a priority for the nurse to provide? 1. Pain management during labor 2. Methods to relieve backaches 3. Breastfeeding positions 4. Characteristics of a newborn

2. It is too early in the pregnancy to educate the patient on the other options.

A woman, 6 weeks pregnant, is having a vaginal examination. Which of the following would the nurse practitioner expect to find? 1. Thin cervical muscle 2. An enlarged ovary 3. Thick cervical muscles 4. Pale pink vaginal wall

2. The practitioner should expect to palpate an enlarged ovary.

The nurse midwife tells a client that the baby is growing and that ballottement was evident during the vaginal examination. How should the nurse explain what the nurse midwife means by ballottement? 1. The nurse midwife saw that the mucous plug was intact 2. The nurse midwife felt that the baby rebound after being pushed. 3. The nurse midwife palpated the fetal parts through the uterine wall 4. The nurse midwife assessed that the baby is head down

2. This is the definition of ballottement.

A woman, 26-weeks' gestation, calls the triage nurse stating, "I'm really scared. I tried not to but I had an orgasm when we were making love. I just know that I will go into preterm labor now." Which of the following responses by the nurse is appropriate? 1. "Lie down and drink a quart of water. If you feel any back pressure at all call me back right away." 2. "Although oxytocin was responsible for your orgasm, it is very unlikely that it will stimulate preterm labor." 3. "I will inform the doctor for you. What I want you to do is to come to the hos pital right now to be checked." 4. "The best thing for you to do right now is to take a warm shower, and then do a fetal kick count assessment."

2. "Although oxytocin was responsible for your orgasm, it is very unlikely that it will stimulate preterm labor."

the nurse in a health care clinic is instructing a pregnant client on how to perform "kick counts" which statement by the client indicates a need for further instruction? 1. "I will record the number of movements or kicks" 2. "I need to lie flat on my back to perform the procedure" 3. "if I count fewer than 10 in a 2 hour period, I should count the kicks again over the next 2 hours" 4. " I should place my hands on the largest part of my abdomen an concentrate on the fetal movements to count the kicks"

2. "I need to lie flat on my back to perform the procedure"

A woman who is seen in the prenatal clinic is found to be 8 weeks pregnant. She confides to the nurse that she is afraid her baby may be "permanently damaged be cause I had at least 5 beers the night I had sex." Which of the following responses by the nurse would be appropriate? 1. "I would let the doctor know that if I were you." 2. "It is unlikely that the baby was affected." 3. "Abortions during the first trimester are very safe." 4. "An ultrasound will tell you if the baby was affected."

2. "It is unlikely that the baby was affected."

A client, in her third trimester, is concerned that she will not know the difference between labor contractions and normal aches and pains of pregnancy. How should the nurse respond? 1. "Don't worry. You'll know the difference when the contractions start." 2. "The contractions may feel just like a backache, but they will come and go." 3. "Contractions are a lot worse than your pregnancy aches and pains." 4. "I understand. You don't want to come to the hospital before you are in labor."

2. "The contractions may feel just like a backache, but they will come and go."

A preganant client in the first trimester calls the nurse at the health care clinic and reports that she has noticed a thin, colorless vaginal drainage. The nurse should make which statement to the client? 1. "come to the clinic immediately" 2. "the vaginal discharge may be bothersome, but is a normal occurrence" 3. report to the emergency department at the maternity center immediately" 4. "use tampons if the discharge is bothersome, nut be sure to change the tampons every 2 hours"

2. "the vaginal discharge may be bothersome, but is a normal occurrence"

The nurse is evaluating the 24-hour dairy intake of four gravid clients. Which of the following clients consumed the highest number of dairy servings during 1 day? The client who consumed: 1. 4 oz whole milk, 2 oz hard cheese, 1 cup of pudding made with milk and 2 oz cream cheese. 2. 1 cup yogurt, 8 oz chocolate milk, 1 cup cottage cheese, and 11⁄2 oz hard cheese. 3. 1 cup cottage cheese, 8 oz whole milk, 1 cup of buttermilk, and 1⁄2 oz hard cheese. 4. 1⁄2 cup frozen yogurt, 8 oz skim milk, 4 oz cream cheese, and 11⁄2 cup of cottage cheese.

2. 1 cup yogurt, 8 oz chocolate milk, 1 cup cottage cheese, and 11⁄2 oz hard cheese.

Which of the following choices can the nurse teach a prenatal client is equivalent to one oz nonmeat serving of protein? 1. 2 tbsp peanut butter. 2. 1 eggs. 3. 1 cup cooked lima beans. 4. 3 ounces mixed nuts.

2. 1 eggs.

A client who was seen in the prenatal clinic at 20 weeks' gestation weighed 128 lb at that time. Approximately how many pounds would the nurse expect the client to weigh at her next visit at 24 weeks' gestation? 1. 129-130 lb. 2. 131-132 lb. 3. 133-134 lb. 4. 135-136 lb.

2. 131-132 lb.

A 36-week gestation gravid lies flat on her back. Which of the following maternal signs/symptoms would the nurse expect to observe? 1. Hypertension. 2. Dizziness. 3. Rales. 4. Chloasma.

2. Dizziness is an expected finding.

the nurse is collecting data during an admission assessment of a client who is pregnant with twins. the client has a healthy 5-year old child who was delivered at 38 weeks and tells the nurse that she does not have a history of any type of abortion or fetal demise. using GTPAL, what should the nurse document in the clients chart? 1. G=3, T=2, P=0, A=0,L=1 2. G=2, T=1, P=0, A=0, L=1 3. G=1, T=1, P=1, A=0, L=1 4. G=2, T=0, P=0, A=0, L=1

2. G=2, T=1, P=0, A=0, L=1

The nurse is caring for a prenatal client who states she is prone to developing ane mia. Which of the following foods should the nurse advise the gravida is the best source of iron? 1. Raisins. 2. Hamburger. 3. Broccoli. 4. Molasses.

2. Hamburger contains the most iron.

It is discovered that a pregnant woman practices pica. Which of the following com plications is most often associated with this behavior? 1. Hypothyroidism. 2. Iron deficiency anemia. 3. Hypercalcemia. 4. Overexposure to zinc.

2. Iron deficiency anemia is often seen in clients who engage in pica.

A couple is preparing to interview obstetric primary care providers in order to deter mine who they will go to for care during their pregnancy and delivery. In order to make the best choice, which of the following actions should the couple perform first? 1. Take a tour of hospital delivery areas. 2. Develop a preliminary birth plan. 3. Make appointments with three or four obstetric care providers. 4. Search the internet for the malpractice histories of the providers.

2. It is best that a couple first develop a birth plan.

The nurse is interviewing a 38-week gestation Muslim woman. Which of the fol lowing questions would be inappropriate for the nurse to ask? 1. "Do you plan to breastfeed your baby?" 2. "What do you plan to name the baby?" 3. "Which pediatrician do you plan to use?" 4. "How do you feel about having an episiotomy?"

2. It is inappropriate to ask the Muslim client about the name for the baby.

a client arrives at the clinic for the first prenatal assessment. she tells the nurse that the first day of her last normal menstrual period was October 19, 2018. using nageles rule, which expected date of delivery should the nurse document in the clients chart? 1. July 12, 2019 2. July 26, 2019 3. august 12, 2019 4. august 26, 2019

2. July 26, 2019

A mother has just experienced quickening. Which of the following developmental changes would the nurse expect to occur at the same time in the woman's pregnancy? 1. Fetal heart begins to beat. 2. Lanugo covers the fetal body. 3. Kidneys secrete urine. 4. Fingernails begin to form

2. Lanugo does cover the fetal body at approximately 20 weeks' gestation.

A Chinese immigrant is being seen in the prenatal clinic. When providing nutrition counseling, which of the following factors should the nurse keep in mind? 1. Many Chinese eat very little protein. 2. Many Chinese believe pregnant women should eat cold foods. 3. Many Chinese are prone to anemia. 4. Many Chinese believe strawberries can cause birth defects.

2. Many Chinese believe pregnant women should eat cold foods.

An 18-week gestation client telephones the obstetrician's office stating, "I'm really scared. I think I have breast cancer. My breasts are filled with tumors." The nurse should base the response on which of the following? 1. Breast cancer is often triggered by pregnancy. 2. Nodular breast tissue is normal during pregnancy. 3. The woman is exhibiting signs of a psychotic break. 4. Anxiety attacks are especially common in the second trimester

2. Nodular breast tissue is normal in pregnancy.

A woman tells the nurse that she would like suggestions for alternate vitamin C sources because she isn't very fond of citrus fruits. Which of the following sugges tions is appropriate? 1. Barley and brown rice. 2. Strawberries and potatoes. 3. Buckwheat and lentils. 4. Wheat flour and figs.

2. Strawberries and potatoes are excellent sources of vitamin C, as are zucchini, blueberries, kiwi, green beans, green peas, and the like.

A woman in her third trimester advises the nurse that she wishes to breastfeed her baby, "but I don't think my nipples are right." Upon examination, the nurse notes that the client has inverted nipples. Which of the following actions should the nurse take at this time? 1. Advise the client that it is unlikely that she will be able to breastfeed. 2. Refer the client to a lactation consultant for advice. 3. Call the labor room and notify them that a client with inverted nipples will be admitted. 4. Teach the woman exercises in order to evert her nipples.

2. The client should be referred to a lac tation consultant.

A woman asks the nurse about the function of amniotic fluid. Which of the follow ing statements by the woman indicates that additional teaching is needed? 1. The fluid provides fetal nutrition. 2. The fluid cushions the fetus from injury. 3. The fluid enables the fetus to grow. 4. The fluid provides a stable thermal environment. 5. the fluid enables the fetus to practice swallowing

2. The fluid cushions the fetus from injury. 3. The fluid enables the fetus to grow. 4. The fluid provides a stable thermal environment. 5. the fluid enables the fetus to practice swallowing

A 36-week gestation gravid client is complaining of dyspnea when lying flat. Which of the following is the likely clinical reason for this complaint? 1. Maternal hypertension. 2. Fundal height. 3. Hydramnios. 4. Congestive heart failure.

2. The fundal height is the likely cause of the woman's dyspnea.

The blood of a pregnant client was initially assessed at 10 weeks' gestation and re assessed at 38 weeks' gestation. Which of the following results would the nurse ex pect to see? 1. Rise in hematocrit from 34% to 38%. 2. Rise in white blood cells from 5,000 cells/mm3 to 15,000 cells/mm3. 3. Rise in potassium from 3.9 mEq/ L to 5.2 mEq/ L. 4. Rise in sodium from 137 mEq/ L to 150 mEq/ L.

2. The nurse would expect to see an elevated white blood cell count.

Which of the following exercises should be taught to a pregnant woman who com plains of backaches? 1. Kegeling. 2. Pelvic tilting. 3. Leg lifting. 4. Crunching.

2. The pelvic tilt is an exercise that can reduce backache pain.

A woman delivers a fetal demise that has lanugo covering the entire body, nails that are present on the fingers and toes, but eyes that are still fused. Prior to the death, the mother stated that she had felt quickening. Based on this information, the nurse knows that the baby is about how many weeks' gestation? 1. 15 weeks. 2. 22 weeks. 3. 29 weeks. 4. 36 weeks.

2. This fetus is about 22 weeks' gesta tion. Nails start to develop in the fi trimester, and lanugo starts to de velop at about 20 weeks, but eyes r main fused until about 29 weeks.

A woman is carrying dizygotic twins. She asks the nurse about the babies. Which of the following explanations is accurate? 1. During a period of rapid growth, the fertilized egg divided completely. 2. When the woman ovulated, she expelled two mature ova. 3. The babies share one placenta and a common chorion. 4. The babies will definitely be the same sex and have the same blood type.

2. This is a true statement. Dizygotic twins result from two mature ova that are fertilized.

A woman confides in the nurse that she practices pica. Which of the following alternatives could the nurse suggest to the woman? 1. Replace laundry starch with salt. 2. Replace ice with frozen fruit juice. 3. Replace soap with cream cheese. 4. Replace soil with uncooked pie crust.

2. This is an excellent suggestion. Fruit juice, although high in sugar, does contain vitamins.

A client states that she is a strong believer in vitamin supplements to maintain her health. The nurse advises the woman that it is recommended to refrain from con suming excess quantities of which of the following vitamins during pregnancy? 1. Vitamin C. 2. Vitamin D. 3. Vitamin B2 (niacin). 4. Vitamin B12 (cobalamin).

2. Vitamin D supplementation can be harmful during pregnancy.

a gravid client is counseled by her primary healthcare provider to have a vaccination during her third trimester of pregnancy. The client questions the healthcare provider's recommendations. Which of the following statements would be appropriate for the nurse to give the client? 1. when received during pregnancy, the rotavirus vaccine helps to prevent dehydration in newborns 2. if you eeceive the tetanus, diphtheria, and acellular pertussis vaccine, your baby will be protected against whooping cough 3. when received during pregnancy, the human papillomavirus helps to prevent newborns from acquiring the sexually transmitted infection 4. if you receive the varicella vaccine, your baby will be protected from the chicken pox virus

2. if you receive the tetanus, diphtheria, and acellular pertussis vaccine, your baby will be protected against whooping cough

a rubella titer result of a 1-day postpartum client is less than 1:8, and a rubella virus vaccine is prescribed to be administered before discharge. the nurse provides which information to the client about the vaccine? select all that apply 1. breast-feeding needs to be stopped for 3 months 2. pregnancy needs to be avoided for 1 to 3 months 3. the vaccine is administered by the subcutaneous route 4. exposure to immunosuppressed individuals needs to be avoided 5. a hypersensitivity reaction can occur if the client has an allergy to eggs 6. the area of the injection needs to be covered with a sterile gauze for 1 week

2. pregnancy needs to be avoided for 1 to 3 months 3. the vaccine is administered by the subcutaneous route 4. exposure to immunosuppressed individuals needs to be avoided 5. a hypersensitivity reaction can occur if the client has an allergy to eggs

a nurse is providing health education to a group of women who are planning to become pregnant. which of the following actions should the nurse advise the women to atke throughout their pregnancyies? SATA 1. the women should avoid consuming well done meat 2. the women should avoid travelling to locations where the zika virus is endemic 3. the women should engage in a daily exercise porgramm 4. the women should consume one serving of seafood each day 5. the women should drink beer and wine instead of spirits like whiskey and vodka

2. the women should avoid travelling to locations where the zika virus is endemic 3. the women should engage in a daily exercise porgramm

Water (L) in food & beverages recommendation for nonpregnant woman:

2.7 l

insulin produces

20th week

When doe fetuses start responding to sound?

24 WEEKS

ova remain viable

24 hrs

Fiber (g) Recommendation for Nonpregnant Woman:

25 g

Iron Recommendation for Pregnancy:

27 g

The nurse enters a woman's room as the health care provider is talking about the features appearing in the embryo. "The face and upper respiratory tract are beginning to take shape. Also the neural tube should be closed." The nurse should suspect the embryo is at least days old.

28

The expected fundal height for a pregnant woman at 28 weeks gestation is cm, give or take 2 cm.

28 From 18 to 30 weeks' gestation, the fundal height in centimeters is approximately equal to the number of weeks of gestation. Therefore, a woman who is 28 weeks' gestation should have a fundal height of 28 cm, give or take 2 cm

Fiber Recommendation for Pregnancy

28 g

scalp hair

28th week

testes begin descending

28th week

Fiber Recommendation for Lactation:

29 g

Water Recommendation for Pregnancy:

3 L

Daily Amount of Dairy

3 cups

A nurse is teaching an infertile couple about how the sperm travel through the man's body during ejaculation. Please put the following five major structures in order, beginning with the place where spermatogenesis occurs and continuing through the path that the sperm and semen travel until ejaculation. 1. Epididymis 2. Prostate 3. Testes 4. Urethra 5. Vas deferens

3,1,5,2,4 The sperm are produced in the testes (3). They then proceed to the epididymis (1) where they mature. The vas deferens (5) is the conduit through which the sperm first travel during ejaculation. The prostate (2), encircling the neck of the urethra, produces a fluid that protects the sperm, and, finally, the sperm exit the male body via the urethra (4).

A woman is 6 weeks pregnant. She has had a previous spontaneous abortion at 14 weeks of gestation and a pregnancy that ended at 38 weeks with the birth of a stillborn girl. What is her gravidity and parity according to the GTPAL system? 2-0-0-1-1 2-1-0-1-0 3-1-0-1-0 3-0-1-1-0

3-1-0-1-0 According to the GPTAL system, this woman's gravidity and parity information is calculated as follows:G: Total number of times the woman has been pregnant (she is pregnant for the third time) T: Number of pregnancies carried to term (only one pregnancy resulted in a fetus at term)P: Number of pregnancies that resulted in a preterm birth (none)A: Abortions or miscarriages before the period of viability (she has had one)L: Number of children born who are currently living (she has no living children)3-1-0-1-0 is the correct calculation of this woman's gravidity and parity.

A client asks the nurse what was mean why the physician told her she had a positive Chadwick's sign. Which of the following information about the finding would be appropriate for the nurse to convey at this time? 1. "It is a purplish stretch mark on your abdomen." 2. "It means that you are having heart problems." 3. "It is a bluish coloration of your cervix and vagina." 4. "It means the doctor heard abnormal sounds when you breathed in."

3. A positive Chadwick's sign means that the client's cervix and vagina are a bluish color. This is a probably sign of pregnancy.

The following for changes occur in the preganancy. Which of them usually increases the father's interest and involvement in the pregnancy? 1. Learning the results of the pregnancy test 2. Attending childbirth education classes 3. Hearing the fetal heartbeat 4. Meeting the obstetrician or midwife

3. Hearing the fetal heartbeat often increases father's interests in their partner's pregnancies.

The glucose challenge screening test is performed at or after 24 weeks' gestation to assess for the maternal physiological response to which of the following pregnancy hormones? 1. Estrogen 2. Progesterone 3. Human placental lactogen 4. Human chorionic gonadotropin

3. Human placental lactogen is an insulin antagonist.

A client makes the following statement after finding out that her pregnancy test is positive. "This is not a good time. I am in college and the baby will be due during final exams." Which of the following responses by the nurse would be most appropriate at this time? 1. "I'm absolutely positive that everything will turn out all right." 2. "I suggest that you e-mail your professors to set up an alternate plan." 3. "It sounds like you are feeling a little overwhelmed right now." 4. "You and the baby's father will find a way to get through the pregnancy."

3. This is the most appropriate response

A 20-year old client states that the at-home pregnancy test that she took this morning was positive. Which of the following comments by the nurse is appropriate at this time? 1. "Congratulations, you and your family must be so happy." 2. "Have you told the baby's father yet?" 3. "How to you feel about the results." 4. "Please tell me when your last menstrual period was."

3. It is important for the nurse to ask the young woman how to feels about being pregnant. She may decide on now to continue with the pregnancy.

When analyzing the need for health teaching of prenatal multigravida, the nurse should ask which of the following questions? 1. "What are the ages of your children?" 2. "What is your marital status?" 3. "Do you ever drink alcohol?" 4. "Do you have any allergies?"

3. This question is important to ask to determine a prenatal client's health teaching needs

When assessing the fruit intake of a pregnant client, the nurse notes that the client usually eats 1 piece of fruit per day and drinks 12 oz glass of fruit juice per day. Which of the following is the most important communication for the nurse to make? 1. "You are effectively meeting your daily fruit requirements." 2. "Fruit juices are excellent sources of folic acid." 3. "It would be even better if you were to consume more whole fruits and less fruit juice." 4. "Your fruit intake far exceeds the recommended daily fruit intake."

3. "It would be even better if you were to consume more whole fruits and less fruit juice."

Which of the following developmental features would the nurse expect to be absent in a 41-week gestation fetus? 1. Fingernails. 2. Eye lashes. 3. Lanugo. 4. Milia.

3. Because this baby is postterm, lanugo would likely not be present.

Which of the following skin changes should the nurse highlight for a pregnant woman's health care practitioner? 1. Linea nigra. 2. Melasma. 3. Petechiae. 4. Spider nevi.

3. Petechiae are pinpoint red or purple spots on the skin. They are seen in hemorrhagic conditions.

A pregnant woman informs the nurse that her last normal menstrual period was on September 20, 2006. Using Nagele's rule, the nurse calculates the client's estimated date of delivery as: 1. May 30, 2007. 2. June 20, 2007. 3. June 27, 2007. 4. July 3, 2007.

3. The estimated date of delivery is June 27, 2007.

A nurse is providing diet counseling to a new prenatal client. Which of the following dairy products should the client be advised to avoid eating during the pregnancy? 1. Vanilla yogurt. 2. Parmesan cheese. 3. Gorgonzola cheese. 4. Chocolate milk.

3. The intake of gorgonzola cheese should be discouraged during preg nancy.

An ultrasound of a fetus' heart shows that normal fetal circulation is occurring. Which of the following statements should the nurse interpret as correct in relation to the fetal circulation? 1. The foramen ovale is a hole between the ventricles. 2. The umbilical vein contains oxygen-poor blood. 3. The right atrium contains both oxygen-rich and oxygen-poor blood. 4. The ductus venosus lies between the aorta and pulmonary artery.

3. The right atrium does contain both oxygen-rich and oxygen-poor blood.

After nutrition counseling, a woman, G3 P1101, proclaims that she certainly can't eat any strawberries during her pregnancy. Which of the following is the likely reason for this statement? 1. The woman is allergic to strawberries. 2. Strawberries have been shown to cause birth defects. 3. The woman believes in old wives' tales. 4. The premature baby died because the woman ate strawberries.

3. The woman believes in old wives' tales.

A woman provides the nurse with the following obstetrical history: Delivered a son, now 7 years old, at 28 weeks' gestation; delivered a daughter, now 5 years old, at 39 weeks' gestation; had a miscarriage 3 years ago, and had a first-trimester abortion 2 years ago. She is currently pregnant. Which of the following portrays an accurate picture of this woman's gravidity and parity? 1. G4 P2121. 2. G4 P1212. 3. G5 P1122. 4. G5 P2211.

3. This accurately reflects this woman's gravidity and parity—G5P1122.

A 34-week gestation woman calls the obstetric office stating, "Since last night I have had three nosebleeds." Which of the following responses by the nurse is appropriate? 1. "You should see the doctor to make sure you are not becoming severely anemic." 2. "Do you have a temperature?" 3. "One of the hormones of pregnancy makes the nasal passages prone to bleeds." 4. "Do you use any inhaled drugs?"

3. This is an accurate statement. Hor monal changes in pregnancy make the nasal passages prone to bleeds.

A woman is 36-weeks' gestation. Which of the following tests will be done during her prenatal visit? 1. Glucose challenge test. 2. Amniotic fluid volume assessment. 3. Vaginal and rectal cultures. 4. Karyotype analysis.

3. Vaginal and rectal cultures are done at approximately 36 weeks' gestation.

a pregnant client is seen for a regular prenatal visit and tells the nurse she is experiencing irregular contractions. the nurse determines that she is experiencing Braxton hicks contractions. on the basis of the findings, which nursing action is appropriate? 1. contact the health care provider 2. instruct the client to maintain bed rest for the remainder of the pregnancy 3. inform the client that these contractions are common and may occur throughout the pregnancy 4. call the maternity unit and inform them that the client will be admitted in a preterm labor condition

3. inform the client that these contractions are common and may occur throughout the pregnancy

the nurse is providing instructions to a pregnant client who is scheduled for an amniocentesis. what instructions should the nurse provide? 1. strict bed rest is required after the procedure 2. hospitalization is necessary for 24 hours after the procedure 3. informed consent needs to be signed before the procedure 4. a fever is expected after the procedure because of the trauma to the abdomen.

3. informed consent needs to be signed before the procedure

the nurse is performing an assessment of a pregnant client who is at a 28 weeks of gestation. the nurse measures the fundal height in centimeters and notes that the fundal height is 30 cm. how should the nurse interpret this finding? 1. the client is measuring large for gestational age 2. the client is measuring small for gestational age 3. the client is measuring normal for gestational age 4. ore evidence is needed to determine size for gestational age

3. the client is measuring normal for gestational age

A client is 15 weeks pregnant. She calls the obstetric office to request a medication for a headache. The nurse answers the telephone. Which of the following is the nurse's best response? 1. "Because the organ systems in the baby are developing right now, you may take no medication." 2. "You can take any of the over-the-counter medications because they are all safe in pregnancy." 3. the physician will prescribe a medication for you that had been shown not to cause any fetal injuries 4.the physician will prescribe a rectal suppository because the medicine will not enter your blood stream

3. the physician will prescribe a medication for you that had been shown not to cause any fetal injuries

Water Recommendation for Lactation:

3.8 L

A nurse is assessing a pregnant woman during a prenatal visit. Several presumptive indicators of pregnancy are documented. Presumptive indicators include: (Circle all that apply.) A. Nausea and vomiting B. Quickening C. Hegar sign D. Palpation of fetal movement by the nurse E. Amenorrhea F. Positive pregnancy test

30. A nurse is assessing a pregnant woman during a prenatal visit. Several presumptive indicators of pregnancy are documented. Presumptive indicators include: (Circle all that apply.) A. Nausea and vomiting B. Quickening C. Hegar sign D. Palpation of fetal movement by the nurse E. Amenorrhea F. Positive pregnancy test "Choices A, B, and E are correct; choices C and F are probable signs and choice D is a positive sign, diagnostic of pregnancy." A. Nausea and vomiting B. Quickening E. Amenorrhea

Hct

33% - 1st and 3rd trimester 32% second 36% non pregnant

Which hematocrit (HCT) and hemoglobin (HGB) results represent the lowest acceptable values for a woman in the third trimester of pregnancy? 38% HCT; 14 g/dL HGB 35% HCT; 13 g/dL HGB 33% HCT; 11 g/dL HGB 32% HCT; 10.5 g/dL HGB

33% HCT; 11 g/dL HGB 38% HCT; 14 g/dL HGB and 35% HCT; 13 g/dL HGB are within normal limits in a nonpregnant woman. 33% HCT; 11 g/dL HGB represents the lowest acceptable values during the first and the third trimesters, and 32% HCT; 10.5 g/dl HGB represents the lowest acceptable values for the second trimester, when the hemodilution effect of blood volume expansion is at its peak.

lightening

38-40 weeks fundal height decreases fetus begins to descend into the pelvis

A woman with a blood volume of 2700 mL prior to pregnancy can expect to have a blood volume of during pregnancy.

3915 Total blood volume during pregnancy increases by 45% in order to meet maternal and fetal needs during pregnancy. A woman with 2700 mL prior to pregnancy would be expected to have an increase of 1215 mL during pregnancy for a total of 3915 mL.

When does the thyroid gland and all other structures of the neck form?

3rd and 4th WEEK

When do blood vessel and blood cell formation begin?

3rd week

The nurse takes the history of a client, G2 P1001, at her first prenatal visit. Which of the following statements would indicate that the client should be referred to a genetic counselor? 1. "My first child has cerebral palsy." 2. "My first child has hypertension." 3. "My first child has asthma." 4. "Mt first child has cystic fibrosis."

4. Cystic Fibrosis is an autosomal recessive genetic disease, so the client with a family history of CF should be referred to a genetic counselor.

A multigravid client is 22 weeks pregnant. Which of the following symptoms would the nurse expect the client to exhibit? 1. Nausea 2. Dyspnea 3. Urinary frequency 4. Leg cramping

4. Leg cramping is often a complaint of clients in the second trimester.

A woman whose prenatal weight was 105 lb weights 109 lb at her 12 week visit. Which of the following comments by the nurse is appropriate at this time? 1. "We expect you to gain about 1 lb per week, so your weight is a little low at this time." 2. "Most women gain no weight during the first trimester, so I would suggest you eat fewer desserts for the next few weeks." 3. "You entered the pregnancy well underweight, so we should check your diet to make sure you are getting the nutrients you need." 4. "Your weight gain is exactly what we would expect it to be at this time."

4. A weight gain of 3-5 lb is expected during the entire first trimester.

A client is having an ultrasound assessment done at her prenatal appointment at 8 weeks' gestation. She asks the nurse, "Can you tell what sex my baby is yet?" Which of the following responses would be appropriate for the nurse to make at this time? 1. "The technician did tell me the sex, but I will have to let the doctor tell you what it is." 2. "The organs are completely formed and present, but the baby is too small for any to be seen." 3. "The technician says that the baby has a penis. It looks like you are having a boy." 4. "I am sorry. It will not be possible to see which sex the baby is for another month or so."

4. "I am sorry. It will not be possible to see which sex the baby is for another month or so."

A client informs the nurse that she is "very constipated." Which of the following foods would be best for the nurse to recommend to the client? 1. pasta 2. Rice. 3. Yogurt. 4. Celery.

4. Celery.

A woman asks the nurse about consuming herbal supplements during pregnancy. Which of the following responses is appropriate? 1. Herbals are natural substances, so they are safely ingested during pregnancy. 2. It is safe to take licorice and cat's claw, but no other herbs are safe. 3. A federal commission has established the safety of herbals during pregnancy. 4. The woman should discuss everything she eats with a health care practitioner.

4. Every woman should advise her health care practitioner of what she is con suming, including food, medicines, herbals, and all other substances.

A 37-week gravid client states that she noticed a "white liquid" leaking from her breasts during a recent shower. Which of the following nursing responses is appro priate at this time? 1. Advise the woman that she may have a galactocele. 2. Encourage the woman to pump her breasts to stimulate an adequate milk supply. 3. Assess the liquid because a breast discharge is diagnostic of a mammary infection. 4. Reassure the mother that this is normal in the third trimester.

4. It is normal for colostrum to be expressed late in pregnancy.

A nurse has identified the following nursing diagnosis for a prenatal client: Altered nutrition: less than body requirements related to poor folic acid intake. Which of the following foods should the nurse suggest the client consume? 1. Potatoes and grapes. 2. Cranberries and squash. 3. Apples and corn. 4. Oranges and spinach

4. Oranges and spinach are excellent folic acid sources.

The nurse is providing anticipatory guidance to a woman in her second trimester regarding signs/symptoms that she might experience in the coming weeks. Which of the following comments by the client indicates that further teaching is needed? 1. "During the third trimester I may experience frequent urination." 2. "During the third trimester I may experience heartburn." 3. "During the third trimester I may experience back pain." 4. "During the third trimester I may experience persistent headache."

4. Persistent headache should not be seen in pregnant women.

A client asks the nurse, "Could you explain how the baby's blood and my blood sep arate at delivery?" Which of the following responses is appropriate for the nurse to make? 1. "When the placenta is born, the circulatory systems separate." 2. "When the doctor clamps the cord, the blood stops mixing." 3. "The separation happens after the baby takes the first breath. The baby's oxygen no longer has to come from you." 4. "The blood actually never mixes. Your blood supply and the baby's blood supply are completely separate."

4. The blood supplies are completely separate.

The nurse asks a 31-week gestation client to lie on the examining table during a prenatal examination. In which of the following positions should the client be placed? 1. Orthopneic. 2. Lateral-recumbent. 3. Sims'. 4. Semi-Fowler's.

4. The client should be placed in a semi Fowler's position.

A client is 35 weeks' gestation. Which of the following findings would the nurse ex pect to see? 1. Nausea and vomiting. 2. Maternal ambivalence. 3. Fundal height 10 cm above the umbilicus. 4. Use of three pillows for sleep comfort

4. The use of three pillows for sleep comfort is often seen in clients who are 35 weeks' gestation.

The partner of a gravida accompanies her to her prenatal appointment. The nurse notes that the father of the baby has gained weight since she last saw him. Which of the following comments is most appropriate for the nurse to make to the father? 1. "I see that you are gaining weight right along with your partner." 2. "You and your partner will be able to go on a diet together after the baby is born." 3. "I can see that you are a bad influence on your partner's eating habits." 4. "I am so glad to see that you are taking so much interest in your partner's pregnancy."

4. This is an appropriate comment to make at this time.

A 12-week gestation client tells the nurse that she and her husband eat sushi at least once per week. She states, "I know that fish is good for me, so I make sure we eat it regularly." Which of the following responses by the nurse is appropriate? 1. "You are correct. Fish is very healthy for you." 2. "You can eat fish, but sushi is too salty to eat during pregnancy." 3. "Sushi is raw. Raw fish is especially high in mercury." 4. "It is recommended that fish be cooked to destroy harmful bacteria."

4. This is correct. It is recommended that during pregnancy the client eat only well-cooked fish.

A vegan is being counseled regarding vitamin intake. It is essential that this woman supplement which of the following B vitamins? 1. B1 (thiamine). 2. B2 (niacin). 3. B6 (pyridoxine). 4. B12 (cobalamin).

4. Vitamin B12 (cobalamin) should be supplemented.

Protein Recommendation for Non-Pregnant Woman:

46 g

When do muscles and bones develop?

4th WEEK

When does the liver and biliary tract develop?

4th WEEK

When do the kidneys begin to form?

5 WEEKS

Daily Amount of Oils:

5-6 teaspoons (25-30 ml)

pancreas form

5-8 week

Daily Amount of Protein:

5.5-6.5 ounces Most meat and poultry choices should be lean or low fat. Fish, nuts, and seeds contain Healthy Oils.

Hegar sign

6 weeks of gestation Softening and compressibility of the lower uterine segment (uterine isthmus) Urinary frequency

Daily Amount of Grains:

6-8 oz At least half the grain servings should be whole grains.

adrenal cortex formed/ produces hormones

6th week/ 8th-9th week

When does sex differentiation begin?

7th WEEK

What is the recommended fluid intake for a pregnant woman?

8 to 10 glasses of water (2.3L) daily. Helps with bowl movements, needed for cells, amniotic fluid, blood and lymph.

nerves travel through the body

8 weeks

MAP in pregnant

84 to 87 +/- 7

MAP in nonpregnant

86 =/- 7.5

When does the secretion of thyroxine start?

8th WEEK

When do kidneys begin to function?

9 weeks

When do distinguishing characteristics of F and M genitalia appear?

9th WEEK

A nurse working with an infertile couple has made the following nursing diagnosis: Sexual dysfunction related to decreased libido. Which of the following assessments is the likely reason for this diagnosis? A. The couple has established a set schedule for their sexual encounters B. The couple has been married for more than eight years C. The couple lives with one set of parents D. The couple has close friends who gave birth to a baby within the past yea

A Clients who "schedule" intercourse often complain that their sexual relationship is unsatisfying.

The likelihood of conceiving dizygotic twins may increase in which patients? Select all that apply. A 39-year-old woman A patient with African heritage A patient with Chinese heritage A patient who underwent in vitro fertilization (IVF) A 25-year-old woman whose father was a dizygotic twin

A 39-year-old woman A patient with an advanced maternal age has an increased likelihood of conceiving dizygotic twins because advanced maternal age is associated with the increased incidence of dizygotic twin births. Correct A patient with African heritage A patient with African heritage has an increased likelihood of conceiving dizygotic twins because dizygotic twins occur at a rate of 1 in 20 in individuals with African heritage. A patient who underwent in vitro fertilization (IVF) A patient who underwent IVF has an increased likelihood of conceiving dizygotic twins because infertility therapy is associated with the increased incidence of dizygotic twin births. Correct A 25-year-old woman whose father was a dizygotic twin This patient may have an increased likelihood of conceiving dizygotic twins because of an inherited tendency of women to release more than one ovum per cycle.

Which of the following could affect female fertility? Select all that apply. Partner relationship status Financial history A clinical diagnosis of anemia Bicornate uterus Uterine abnormality Cephalopelvic disproportion (CPD)

A clinical diagnosis of anemia A diagnosis of anemia and the presence of a uterine abnormality can both affect a woman's fertility status. Partner relationship status, although important in one's well-being, does not typically affect female fertility, and financial history is not a factor.The presence of CPD relates to whether or not a woman can successfully deliver a baby vaginally on the basis of obtained pelvic measurements. Bicornate uterus A diagnosis of anemia and the presence of a uterine abnormality can both affect a woman's fertility status. Partner relationship status, although important in one's well-being, does not typically affect female fertility, and financial history is not a factor.The presence of CPD relates to whether or not a woman can successfully deliver a baby vaginally on the basis of obtained pelvic measurements. Uterine abnormality A diagnosis of anemia and the presence of a uterine abnormality can both affect a woman's fertility status. Partner relationship status, although important in one's well-being, does not typically affect female fertility, and financial history is not a factor.The presence of CPD relates to whether or not a woman can successfully deliver a baby vaginally on the basis of obtained pelvic measurements.

Puppa

A common dermatosis that can occur during pregnancy

When teaching a woman about correct diaphragm usage, the nurse should include which information? A diaphragm can be used with oil-based lubricants. A diaphragm can be used with or without spermicide. A diaphragm is suitable for women with a latex allergy. A diaphragm must be fitted by a health care practitioner.

A diaphragm must be fitted by a health care practitioner. The diaphragm must be fitted by a health care practitioner before use. This information should be included when teaching about correct diaphragm usage.

Donor embryo

A donated embryo is transferred to the uterus of an infertile woman at the appropriate time of the menstrual cycle

Quickening

A flutter and is difficult to distinguish from peristalsis that becomes more intense and frequent over time May help predict dating in gestation multiparous feel 14-16 weeks nulliparous 18 or later

What does the placenta function as?

A means for metabolic exchange

The nurse obtains the history from the pregnant woman during the initial prenatal visit. Which would concern the nurse? A pregnant woman who consumes 400 mcg of folic acid daily. A pregnant woman with type I diabetes and who is on insulin. A pregnant woman who had a full-term viable pregnancy two years ago. A pregnant woman who denies history of sexually transmitted infection.

A pregnant woman with type I diabetes and who is on insulin. Type I diabetes would concern the nurse as this increases the risk of fetal anomalies, spontaneous abortion, and maternal complications such as preeclampsia.

uterine souffle

A rushing or blowing sound of maternal blood through the Uterine arteries to the placenta that is synchronous with the maternal Pulse

Under which circumstances might a woman need to seek an alternative natural family planning method? A woman using the Standard Days method has a menstrual cycle that is 36 days long. A woman using the symptothermal method has a thermometer that displays temperature in tenths of a degree. A woman recording basal body temperature (BBT) experiences a slight drop in temperature just before ovulation. A woman using the two-day method is unable to distinguish normal cervical secretions from secretions with a sticky, egg-white consistency.

A woman using the Standard Days method has a menstrual cycle that is 36 days long. The Standard Days method can be used by women whose cycles range from 26 to 32 days in length but is ineffective for other women. This woman would need to seek an alternative natural family planning method.

Primipara

A woman who has completed one pregnancy with a fetus or fetuses who have reached 20 weeks of gestation

primapara

A woman who has completed one pregnancy with the fetus or fetuses who have reached 20 weeks of gestation

multipara

A woman who has completed two or more pregnancies to 20 weeks of gestation

multigravida

A woman who has had two or more pregnancies

Nulligravida

A woman who has never been pregnant

nullipara

A woman who has not completed a pregnancy with a fetus or fetuses beyond 20 weeks of gestation

Nullipara

A woman who has not completed a pregnancy with a fetus or fetuses who have reached 20 weeks of gestation

nulligravida

A woman who is never been pregnant and is not currently pregnant

Primigravida

A woman who is pregnant for the first time

In vitro fertilization and embryo transfer

A woman's eggs are collected from her ovaries, fertilized in the laboratory with sperm and transferred to her uterus once normal embryo development has occurred

A 30 year old woman at 16 weeks of gestation comes for a routine prenatal visit. Her 24-dietary recall is evaluated by the nurse. Which entry indicates that this woman needs further instructions regarding nutrient needs during pregnancy? A. 8 ozs total from the meat, poultry, fish, dry beans, eggs, and nut groups B. 3 cups of vegetables and 2 cups of fruits C. seven ozs of grains, including whole-grain bread and bran cereal D. three cups from the milk, yogurt, and cheese group

A. 8 ozs total from the meat, poultry, fish, dry beans, eggs, and nut groups

A nurse taught a pregnant women about the importance of iron and and taking her iron supplement daily. Which statement by the woman indicates the need for further instruction? A. I take my iron supplement at bedtime with a cup of tea B. I include a lot of citrus fruit and tomatoes in my diet C. my stools can turn black or dark green D. I need to keep track of my bowel movements because taking iron can lead to constipation

A. I take my iron supplement at bedtime with a cup of tea

Lifestyle and sexual practices can affect fertility. Which of the following practices could enhance a couple's ability to conceive? A. Use balanced nutrition and exercise to achieve a normal BMI B. abstain from alcohol C. avoid use of lubricants during intercourse D. relax in a hot tub every day before bed

A. Use balanced nutrition and exercise to achieve a normal BMI

an infertile woman is given clomiphene citrate to achieve which of the following therapeutic outcomes? A. stimulate the pituitary gland to secrete FSH and LH B. enhance the development of a secretory endometrium C. induce the formation of the corpus luteum D. increase secretion of favorable cervical mucus to enhance sperm viability.

A. stimulate the pituitary gland to secrete FSH and LH

The BMI for a woman who is 51 kg before pregnancy and 1.57 m tall is _________.

ANS: 20.7 A commonly used method of evaluating the appropriateness of weight for height is the BMI, which is calculated by the following formula. BMI = Weight in kg divided by the height in meters squared BMI = 51 kg ¸ (1.57 m)2 51 ¸ 2.47 = 20.69

To provide optimal prenatal care, a blood pressure reading should be obtained at every prenatal visit. Calculating the mean arterial pressure (MAP) can increase the value of this diagnostic finding. MAP readings for a pregnant woman at term are 90+ = 5.8 mm Hg. The nurse has just obtained a BP of 106/70 mm Hg on a 37-week primiparous client. The formula for the MAP reading is (systolic + [2 ´ diastolic]) ¸ 3. The MAP reading for this client is _____ mm Hg.

ANS: 82 (106 + [2 ´ 70]) ¸ 3 (106 + 140) ¸ 3 246 ¸ 3 = 82 mm Hg The MAP can also be thought of as the mean of the blood pressure present in arterial circulation.

What is the primary role of the doula during labor? a. Helps the woman perform Lamaze breathing techniques and to provide support to the woman and her partner b. Checks the fetal monitor tracing for effects of the labor process on the fetal heart rate c. Takes the place of the father as a coach and support provider d. Administers pain medications as needed by the woman

ANS: A A doula is professionally trained to provide labor support, including physical, emotional, and informational support, to both the woman and her partner during labor and the birth. The doula does not become involved with clinical tasks.

What is important for the nurse to recognize regarding the new father and his acceptance of the pregnancy and preparation for childbirth? a. The father goes through three phases of acceptance of his own. b. The father's attachment to the fetus cannot be as strong as that of the mother because it does not start until after the birth. c. In the last 2 months of pregnancy, most expectant fathers suddenly get very protective of their established lifestyle and resist making changes to the home. d. Typically, men remain ambivalent about fatherhood right up to the birth of their child.

ANS: A A father typically goes through three phases of development to reach acceptance of fatherhood: the announcement phase, the moratorium phase, and the focusing phase. The father-child attachment can be as strong as the mother-child relationship and can also begin during pregnancy. During the last 2 months of the pregnancy, many expectant fathers work hard to improve the environment of the home for the child. Typically, the expectant father's ambivalence ends by the first trimester, and he progresses to adjusting to the reality of the situation and then to focusing on his role.

What is the correct term for a woman who has completed one pregnancy with a fetus (or fetuses) reaching the stage of fetal viability? a. Primipara b. Primigravida c. Multipara d. Nulligravida

ANS: A A primipara is a woman who has completed one pregnancy with a viable fetus. To help remember the terms: gravida is a pregnant woman; para comes from parity, meaning a viable fetus; primi means first; multi means many; and null means none. Therefore, a primigravida is a woman pregnant for the first time; a multipara is a woman who has completed two or more pregnancies with a viable fetus; and a nulligravida is a woman who has never been pregnant.

The nurse has formulated a diagnosis of Imbalanced nutrition: Less than body requirements for the client. Which goal is most appropriate for this client to obtain? a. Gain a total of 30 pounds. b. Consistently take daily supplements. c. Decrease her intake of snack foods. d. Increase her intake of complex carbohydrates.

ANS: A A weight gain of 30 pounds is one indication that the client has gained a sufficient amount for the nutritional needs of pregnancy. A daily supplement is not the best goal for this client and does not meet the basic need of proper nutrition during pregnancy. Decreasing snack foods may be needed and should be assessed; however, assessing weight gain is the best method of monitoring nutritional intake for this pregnant client. Although increasing the intake of complex carbohydrates is important for this client, monitoring the weight gain should be the end goal.

In vitro fertilization-embryo transfer (IVF-ET) is a common approach for women with blocked fallopian tubes or with unexplained infertility and for men with very low sperm counts. A husband and wife have arrived for their preprocedural interview. Which explanation regarding the procedure is most accurate? a. "The procedure begins with collecting eggs from your wife's ovaries." b. "A donor embryo will be transferred into your wife's uterus." c. "Donor sperm will be used to inseminate your wife." d. "Don't worry about the technical stuff; that's what we are here for."

ANS: A A woman's eggs are collected from her ovaries, fertilized in the laboratory with the partner's sperm, and transferred to her uterus after normal embryonic development has occurred. Transferring a donor embryo to the woman's uterus describes the procedure for a donor embryo. Inseminating the woman with donor sperm describes therapeutic donor insemination. Telling the client not to worry discredits the client's need for teaching and is not the most appropriate response.

Which presumptive sign or symptom of pregnancy would a client experience who is approximately 10 weeks of gestation? a. Amenorrhea b. Positive pregnancy test c. Chadwick sign d. Hegar sign

ANS: A Amenorrhea is a presumptive sign of pregnancy. Presumptive signs of pregnancy are those felt by the woman. A positive pregnancy test and the presence of the Chadwick and Hegar signs are all probable signs of pregnancy.

Which information regarding amniotic fluid is important for the nurse to understand? a. Amniotic fluid serves as a source of oral fluid and a repository for waste from the fetus. b. Volume of the amniotic fluid remains approximately the same throughout the term of a healthy pregnancy. c. The study of fetal cells in amniotic fluid yields little information. d. A volume of more than 2 L of amniotic fluid is associated with fetal renal abnormalities.

ANS: A Amniotic fluid serves as a source of oral fluid, serves as a repository for waste from the fetus, cushions the fetus, and helps maintain a constant body temperature. The volume of amniotic fluid constantly changes. The study of amniotic fluid yields information regarding the sex of the fetus and the number of chromosomes. Too much amniotic fluid (hydramnios) is associated with gastrointestinal and other abnormalities.

Which women should undergo prenatal testing for the human immunodeficiency virus (HIV)? a. All women, regardless of risk factors b. Women who have had more than one sexual partner c. Women who have had a sexually transmitted infection (STI) d. Woman who are monogamous with one partner

ANS: A An HIV test is recommended for all women, regardless of risk factors. The incidence of perinatal transmission from an HIV-positive mother to her fetus ranges from 25% to 35%. Women who test positive for HIV can then be treated.

What kind of fetal anomalies are most often associated with oligohydramnios? a. Renal b. Cardiac c. Gastrointestinal d. Neurologic

ANS: A An amniotic fluid volume of less than 300 ml (oligohydramnios) is often associated with fetal renal anomalies. The amniotic fluid volume has no bearing on the fetal cardiovascular system. Gastrointestinal anomalies are associated with hydramnios or an amniotic fluid volume greater than 2 L. The amniotic fluid volume has no bearing on the fetal neurologic system.

A woman in labor passes some thick meconium as her amniotic fluid ruptures. The client asks the nurse where the baby makes the meconium. What is the correct response by the nurse? a. Fetal intestines b. Fetal kidneys c. Amniotic fluid d. Placenta

ANS: A As the fetus nears term, fetal waste products accumulate in the intestines as dark green-to-black, tarry meconium. Meconium is not produced by the fetal kidneys nor should it be present in the amniotic fluid, which may be an indication of fetal compromise. The placenta does not produce meconium.

With regard to medications, herbs, boosters, and other substances normally encountered by pregnant women, what is important for the nurse to be aware of? a. Both prescription and over-the-counter (OTC) drugs that otherwise are harmless can be made hazardous by metabolic deficiencies of the fetus. b. The greatest danger of drug-caused developmental deficits in the fetus is observed in the final trimester. c. Killed-virus vaccines (e.g., tetanus) should not be administered during pregnancy, but live-virus vaccines (e.g., measles) are permissible. d. No convincing evidence exists that secondhand smoke is potentially dangerous to the fetus.

ANS: A Both prescription and OTC drugs that otherwise are harmless can be made hazardous by metabolic deficiencies of the fetus. This is especially true for new medications and combinations of drugs. The greatest danger of drug-caused developmental defects exists in the interval from fertilization through the first trimester, during which a woman may not realize that she is pregnant. Live-virus vaccines should be part of postpartum care; killed-virus vaccines may be administered during pregnancy. Secondhand smoke is associated with fetal growth restriction and increases in infant mortality.

A woman is in her seventh month of pregnancy. She has been complaining of nasal congestion and occasional epistaxis. Which statement best describes why this may be happening to this client? a. This respiratory change is normal in pregnancy and caused by an elevated level of estrogen. b. This cardiovascular change is abnormal, and the nosebleeds are an ominous sign. c. The woman is a victim of domestic violence and is being hit in the face by her partner. d. The woman has been intranasally using cocaine.

ANS: A Elevated levels of estrogen cause capillaries to become engorged in the respiratory tract, which may result in edema in the nose, larynx, trachea, and bronchi. This congestion may cause nasal stuffiness and epistaxis. Cardiovascular changes in pregnancy may cause edema in the lower extremities. Domestic violence cannot be determined on the basis on the sparse facts provided. If the woman had been hit in the face, then she most likely would have additional physical findings. Cocaine use cannot be determined on the basis on the sparse facts provided.

A woman asks the nurse, "What protects my baby's umbilical cord from being squashed while the baby's inside of me?" What is the nurse's best response? a. "Your baby's umbilical cord is surrounded by connective tissue called Wharton's jelly, which prevents compression of the blood vessels." b. "Your baby's umbilical cord floats around in blood and amniotic fluid." c. "You don't need to be worrying about things like that." d. "The umbilical cord is a group of blood vessels that are very well protected by the placenta.

ANS: A Explaining the structure and function of the umbilical cord is the most appropriate response. Connective tissue called Wharton's jelly surrounds the umbilical cord, prevents compression of the blood vessels, and ensures continued nourishment of the embryo or fetus. The umbilical cord does not float around in blood or fluid. Telling the client not to worry negates her need for information and discounts her feelings. The placenta does not protect the umbilical cord.

Which information regarding protein in the diet of a pregnant woman is most helpful to the client? a. Many protein-rich foods are also good sources of calcium, iron, and B vitamins. b. Many women need to increase their protein intake during pregnancy. c. As with carbohydrates and fat, no specific recommendations exist for the amount of protein in the diet. d. High-protein supplements can be used without risk by women on macrobiotic diets.

ANS: A Good sources for protein, such as meat, milk, eggs, and cheese, have a lot of calcium and iron. Most women already eat a high-protein diet and do not need to increase their intake. Protein is sufficiently important that specific servings of meat and dairy are recommended. High-protein supplements are not recommended because they have been associated with an increased incidence of preterm births.

A pregnant woman reports that she is still playing tennis at 32 weeks of gestation. Which recommendation would the nurse make for this particular client after a tennis match? a. Drink several glasses of fluid. b. Eat extra protein sources such as peanut butter. c. Enjoy salty foods to replace lost sodium. d. Consume easily digested sources of carbohydrate.

ANS: A If no medical or obstetric problems contraindicate physical activity, then pregnant women should get 30 minutes of moderate physical exercise daily. Liberal amounts of fluid should be consumed before, during, and after exercise because dehydration can trigger premature labor. The woman's caloric intake should be sufficient to meet the increased needs of pregnancy and the demands of exercise.

An expectant couple attending childbirth classes have questions regarding multiple births since twins "run in the family." What information regarding multiple births is important for the nurse to share? a. Twinning and other multiple births are increasing because of the use of fertility drugs and delayed childbearing. b. Dizygotic twins (two fertilized ova) have the potential to be conjoined twins. c. Identical twins are more common in Caucasian families. d. Fraternal twins are the same gender, usually male.

ANS: A If the parents-to-be are older and have taken fertility drugs, then they would be very interested to know about twinning and other multiple births. Conjoined twins are monozygotic; that is, they are from a single fertilized ovum in which division occurred very late. Identical twins show no racial or ethnic preference, and fraternal twins are more common among African-American women. Fraternal twins can be different genders or the same gender, and identical twins are the same gender.

Which guidance might the nurse provide for a client with severe morning sickness? a. Trying lemonade and potato chips b. Drinking plenty of fluids early in the day c. Immediately brushing her teeth after eating d. Never snacking before bedtime

ANS: A Interestingly, some women can tolerate tart or salty foods when they are nauseated. Lemonade and potato chips are an ideal combination. The woman should avoid drinking too much when nausea is most likely, but she should increase her fluid levels later in the day when she feels better. The woman should avoid brushing her teeth immediately after eating. A small snack of cereal and milk or yogurt before bedtime may help the stomach in the morning.

A pregnant client tells her nurse that she is worried about the blotchy, brownish coloring over her cheeks, nose, and forehead. The nurse can reassure her that this is a normal condition related to hormonal changes. What is the correct term for this integumentary finding? a. Melasma b. Linea nigra c. Striae gravidarum d. Palmar erythema

ANS: A Melasma, (also called chloasma, the mask of pregnancy), usually fades after birth. This hyperpigmentation of the skin is more common in women with a dark complexion. Melasma appears in 50% to 70% of pregnant women. Linea nigra is a pigmented line that runs vertically up the abdomen. Striae gravidarum are also known as stretch marks. Palmar erythema is signified by pinkish red blotches on the hands.

Which nutrient's recommended dietary allowance (RDA) is higher during lactation than during pregnancy? a. Energy (kcal) b. Iron c. Vitamin A d. Folic acid

ANS: A Nutrient needs for energy—protein, calcium, iodine, zinc, B vitamins, and vitamin C—remain higher during lactation than during pregnancy. The need for iron is not higher during lactation than during pregnancy. A lactating woman does not have a greater requirement for vitamin A than a nonpregnant woman. Folic acid requirements are the highest during the first trimester of pregnancy.

The nurse caring for a pregnant client is evaluating his or her health teaching regarding fetal circulation. Which statement from the client reassures the nurse that his or her teaching has been effective? a. "Optimal fetal circulation is achieved when I am in the side-lying position." b. "Optimal fetal circulation is achieved when I am on my back with a pillow under my knees." c. "Optimal fetal circulation is achieved when the head of the bed is elevated." d. "Optimal fetal circulation is achieved when I am on my abdomen."

ANS: A Optimal circulation is achieved when the woman is lying at rest on her side. Decreased uterine circulation may lead to intrauterine growth restriction. Previously, it was believed that the left lateral position promoted maternal cardiac output, enhancing blood flow to the fetus. However, it is now known that the side-lying position enhances uteroplacental blood flow. If a woman lies on her back with the pressure of the uterus compressing the vena cava, then blood return to the right atrium is diminished. Although having the head of the bed elevated is recommended and ideal for later in pregnancy, the woman still must maintain a lateral tilt to the pelvis to avoid compressing the vena cava. Many women find lying on their abdomen uncomfortable as pregnancy advances. Side-lying is the ideal position to promote blood flow to the fetus.

A client has arrived for her first prenatal appointment. She asked the nurse to explain exactly how long the pregnancy will be. What is the nurse's best response? a. Normal pregnancy is 10 lunar months. b. Pregnancy is made up of four trimesters. c. Pregnancy is considered term at 36 weeks. d. Estimated date of delivery (EDD) is 40 completed weeks.

ANS: A Pregnancy spans 9 calendar months; but, health care providers prefer to use the lunar month of 28 days or 4 weeks. Pregnancy consists of three trimesters, each approximately 13 weeks long. A pregnancy is considered term at 37 completed weeks; however, EDD is based upon 40 weeks of gestation.

A new mother asks the nurse about the "white substance" covering her infant. How should the nurse explain the purpose of vernix caseosa? a. Vernix caseosa protects the fetal skin from the amniotic fluid. b. Vernix caseosa promotes the normal development of the peripheral nervous system. c. Vernix caseosa allows the transport of oxygen and nutrients across the amnion. d. Vernix caseosa regulates fetal temperature.

ANS: A Prolonged exposure to the amniotic fluid during the fetal period could result in the breakdown of the skin without the protection of the vernix caseosa. Normal development of the peripheral nervous system was dependent on nutritional intake of the mother. The amnion was the inner membrane that surrounded the fetus and was not involved in the oxygen and nutrient exchange. The amniotic fluid helped maintain fetal temperature.

After the nurse completes nutritional counseling for a pregnant woman, she asks the client to repeat the instructions to assess the client's understanding. Which statement indicates that the client understands the role of protein in her pregnancy? a. "Protein will help my baby grow." b. "Eating protein will prevent me from becoming anemic." c. "Eating protein will make my baby have strong teeth after he is born." d. "Eating protein will prevent me from being diabetic."

ANS: A Protein is the nutritional element basic to growth. An adequate protein intake is essential to meeting the increasing demands of pregnancy. These demands arise from the rapid growth of the fetus; the enlargement of the uterus, mammary glands, and placenta; the increase in the maternal blood volume; and the formation of the amniotic fluid. Iron intake prevents anemia. Calcium intake is needed for fetal bone and tooth development. Glycemic control is needed in those with diabetes; protein is one nutritional factor to consider for glycemic control but not the primary role of protein intake.

Which nutritional recommendation regarding fluids is accurate? a. A woman's daily intake should be six to eight glasses of water, milk, and/or juice. b. Coffee should be limited to no more than 2 cups, but tea and cocoa can be consumed without worry. c. Of the artificial sweeteners, only aspartame has not been associated with any maternity health concerns. d. Water with fluoride is especially encouraged because it reduces the child's risk of tooth decay.

ANS: A Six to eight glasses is still the standard for fluids; however, they should be the right fluids. All beverages containing caffeine, including tea, cocoa, and some soft drinks, should be avoided or should be consumed only in limited amounts. Artificial sweeteners, including aspartame, have no ill effects on the normal mother or fetus. However, mothers with phenylketonuria (PKU) should avoid aspartame. Although no evidence indicates that prenatal fluoride consumption reduces childhood tooth decay, fluoride still helps the mother.

A first-time mother at 18 weeks of gestation is in for her regularly scheduled prenatal visit. The client tells the nurse that she is afraid that she is going into premature labor because she is beginning to have regular contractions. The nurse explains that these are Braxton Hicks contractions. What other information is important for the nurse to share? a. Braxton Hicks contractions should be painless. b. They may increase in frequency with walking. c. These contractions might cause cervical dilation. d. Braxton Hicks contractions will impede oxygen flow to the fetus.

ANS: A Soon after the fourth month of gestation, uterine contractions can be felt through the abdominal wall. Braxton Hicks contractions are regular and painless and continue throughout the pregnancy. Although they are not painful, some women complain that they are annoying. This type of contraction usually ceases with walking or exercise. Braxton Hicks contractions can be mistaken for true labor; however, they do not increase in intensity, frequency, or cause cervical dilation. These contractions facilitate uterine blood flow through the intervillous spaces of the placenta and thereby promote oxygen delivery to the fetus.

Which condition would be inappropriate to treat with exogenous progesterone (human chorionic gonadotropin)? a. Thyroid dysfunction b. Recent miscarriage c. PCOD d. Oocyte retrieval

ANS: A Synthroid is administered for anovulation associated with hypothyroidism. For women with polycystic ovulation syndrome or a history of miscarriage, oocyte retrieval may have insufficient progesterone and require exogenous progesterone until placental production is sufficient.

Why might it be more difficult to diagnose appendicitis during pregnancy? a. The appendix is displaced upward and laterally, high and to the right. b. The appendix is displaced upward and laterally, high and to the left. c. The appendix is deep at the McBurney's point. d. The appendix is displaced downward and laterally, low and to the right.

ANS: A The appendix is displaced high and to the right, not to the left. It is displaced beyond the McBurney's point and is not displaced in a downward direction.

Which behavior indicates that a woman is "seeking safe passage" for herself and her infant? a. She keeps all prenatal appointments. b. She "eats for two." c. She slowly drives her car. d. She wears only low-heeled shoes.

ANS: A The goal of prenatal care is to foster a safe birth for the infant and mother. Although properly eating, carefully driving, and using proper body mechanics all are healthy measures that a mother can take, obtaining prenatal care is the optimal method for providing safety for both herself and her baby.

The various systems and organs of the fetus develop at different stages. Which statement is most accurate? a. Cardiovascular system is the first organ system to function in the developing human. b. Hematopoiesis originating in the yolk sac begins in the liver at 10 weeks of gestation. c. Body changes from straight to C-shape occurs at 8 weeks of gestation. d. Gastrointestinal system is mature at 32 weeks of gestation.

ANS: A The heart is developmentally complete by the end of the embryonic stage. Hematopoiesis begins in the liver during the sixth week. The body becomes C-shaped at 21 weeks of gestation. The gastrointestinal system is complete at 36 weeks of gestation.

An infertility specialist prescribes clomiphene citrate (Clomid, Serophene) for a woman experiencing infertility. She is very concerned about the risk of multiple pregnancies. What is the nurse's most appropriate response? a. "This is a legitimate concern. Would you like to discuss further the chances of multiple pregnancies before your treatment begins?" b. "No one has ever had more than triplets with Clomid." c. "Ovulation will be monitored with ultrasound to ensure that multiple pregnancies will not happen." d. "Ten percent is a very low risk, so you don't need to worry too much."

ANS: A The incidence of multiple pregnancies with the use of these medications is higher than 25%. The client's concern is legitimate and should be discussed so that she can make an informed decision. Stating that no one has ever had more than triplets with Clomid is inaccurate and negates the client's concerns. Ultrasound cannot ensure that a multiple pregnancy will not occur, and 10% is inaccurate. Furthermore, the client's concern is discredited with a statement such as, "...don't worry."

Which condition is likely to be identified by the quadruple marker screen? a. Down syndrome b. Diaphragmatic hernia c. Congenital cardiac abnormality d. Anencephaly

ANS: A The maternal serum level marker of alpha-fetoprotein is used to screen for Down syndrome, trisomy 18, neural tube defects, and other chromosomal anomalies. The quadruple-marker screen will not detect diaphragmatic hernia. Additional testing, such as ultrasonography, is required to diagnose diaphragmatic hernia. Congenital cardiac abnormality will most likely be identified during an ultrasound examination. The quadruple-marker screen will not detect anencephaly.

What is the correct term used to describe the mucous plug that forms in the endocervical canal? a. Operculum b. Leukorrhea c. Funic souffle d. Ballottement

ANS: A The operculum protects against bacterial invasion. Leukorrhea is the mucus that forms the endocervical plug (the operculum). The funic souffle is the sound of blood flowing through the umbilical vessels. Ballottement is a technique for palpating the fetus.

A client at 34 weeks of gestation seeks guidance from the nurse regarding personal hygiene. Which information should the nurse provide? a. Tub bathing is permitted even in late pregnancy unless membranes have ruptured. b. The perineum should be wiped from back to front. c. Bubble bath and bath oils are permissible because they add an extra soothing and cleansing action to the bath. d. Expectant mothers should use specially treated soap to cleanse the nipples.

ANS: A The primary danger from taking baths is falling in the tub. The perineum should be wiped from front to back. Bubble baths and bath oils should be avoided because they may irritate the urethra. Soap, alcohol, ointments, and tinctures should not be used to cleanse the nipples because they remove protective oils. Warm water is sufficient.

Which development related to the integumentary system is correct? a. Very fine hairs called lanugo appear at 12 weeks of gestation. b. Eyelashes, eyebrows, and scalp hair appear at 28 weeks of gestation. c. Fingernails and toenails develop at 28 weeks of gestation. d. By 32 weeks, scalp hair becomes apparent.

ANS: A Very fine hairs, called lanugo appear first at 12 weeks of gestational age on the fetus' eyebrows and upper lip. By 20 weeks of gestation, lanugo covers the entire body. By 20 weeks of gestation the eyelashes, eyebrows, and scalp hair also begin to grow. By 28 weeks of gestation, the scalp hair is longer than these fine hairs, which is thin and may disappear by term. Fingernails and toenails develop from thickened epidermis, beginning during the 10th week. Fingernails reach the fingertips at 32 weeks of gestation, and the toenails reach the toe tips at 36 weeks of gestation.

To provide adequate care, the nurse should be cognitive of which important information regarding infertility? a. Is perceived differently by women and men. b. Has a relatively stable prevalence among the overall population and throughout a woman's potential reproductive years. c. Is more likely the result of a physical flaw in the woman than in her male partner. d. Is the same thing as sterility.

ANS: A Women tend to be more stressed about infertility tests and to place more importance on having children. The prevalence of infertility is stable among the overall population, but it increases with a woman's age, especially after age 40 years. Of cases with an identifiable cause, approximately 40% are related to female factors, 40% to male factors, and 20% to both partners. Sterility is the inability to conceive. Infertility or subfertility is a state of requiring a prolonged time to conceive.

Which signs and symptoms should a woman immediately report to her health care provider? (Select all that apply.) a. Vaginal bleeding b. Rupture of membranes c. Heartburn accompanied by severe headache d. Decreased libido e. Urinary frequency

ANS: A, B, C Vaginal bleeding, rupture of membranes, and severe headaches are signs of potential complications in pregnancy. Clients should be advised to report these signs to their health care provider. Decreased libido and urinary frequency are common discomforts of pregnancy that do not require immediate health care interventions.

Many factors, male and female, contribute to normal fertility. Approximately 40% of cases of infertility are related to the female partner. Which factors are possible causes for female infertility? (Select all that apply.) a. Congenital or developmental b. Hormonal or ovulatory c. Tubal or peritoneal d. Uterine e. Emotional or psychologic

ANS: A, B, C, D Female infertility can be attributed to alterations in any one of these systems along with possible vaginal-cervical factors. Although the diagnosis and treatment of infertility require considerable emotional investment and may cause psychologic stress, these are not considered factors associated with infertility. Feelings connected with infertility are many and complex. Resolve is an organization that provides support, advocacy, and education for both clients and health care providers.

Most women with uncomplicated pregnancies can use the nurse as their primary source for nutritional information. However, the nurse or midwife may need to refer a client to a registered dietitian for in-depth nutritional counseling. Which conditions would require such a consultation? (Select all that apply.) a. Preexisting or gestational illness such as diabetes b. Ethnic or cultural food patterns c. Obesity d. Vegetarian diets e. Multifetal pregnancy

ANS: A, B, C, D The nurse should be especially aware that conditions such as diabetes can require in-depth dietary planning and evaluation. To prevent issues with hypoglycemia and hyperglycemia, as well as an increased risk for perinatal morbidity and mortality, the client with a preexisting or gestational illness would benefit from a referral to a dietitian. Consultation with a dietitian may ensure that cultural food beliefs are congruent with modern knowledge of fetal development and that adjustments can be made to ensure that all nutritional needs are met. The obese pregnant client may be under the misapprehension that, because of her excess weight, little or no weight gain is necessary. According to the Institute of Medicine, a client with a BMI in the obese range should gain at least 7 kg to ensure a healthy outcome. This client may require in-depth counseling on the optimal food choices. The vegetarian client needs to have her dietary intake carefully assessed to ensure that the optimal combination of amino acids and protein intake is achieved. Very strict vegetarians (vegans) who consume only plant products may also require vitamin B and mineral supplementation. A multifetal pregnancy can be managed by increasing the number of servings of complex carbohydrates and proteins.

Pregnancy is a hypercoagulable state in which women are at a fivefold to sixfold increased risk for thromboembolic disease. The tendency for blood to clot is greater, attributable to an increase in various clotting factors. Which of these come into play during pregnancy? (Select all that apply.) a. Factor VII b. Factor VIII c. Factor IX d. Factor XIII e. Fibrinogen

ANS: A, B, C, E Factors VII, VIII, IX, X, and fibrinogen increase in pregnancy. Factors that inhibit coagulation decrease. Fibrinolytic activity (dissolving of a clot) is depressed during pregnancy and the early postpartum period to protect the women from postpartum hemorrhage.

Congenital disorders refer to those conditions that are present at birth. These disorders may be inherited and caused by environmental factors or maternal malnutrition. Toxic exposures have the greatest effect on development between 15 and 60 days of gestation. For the nurse to be able to conduct a complete assessment of the newly pregnant client, he or she should be knowledgeable regarding known human teratogens. Which substances might be considered a teratogen? (Select all that apply.) a. Cytomegalovirus (CMV) b. Ionizing radiation c. Hypothermia d. Carbamazepine e. Lead

ANS: A, B, D, E Exposure to radiation and a number of infections may result in profound congenital deformities. These include but are not limited to varicella, rubella, syphilis, parvovirus, CMV, and toxoplasmosis. Certain maternal conditions such as diabetes and phenylketonuria (PKU) may also affect organs and other parts of the embryo during this developmental period. Drugs such as antiseizure medications (e.g., carbamazepine) and some antibiotics, as well as chemicals including lead, mercury, tobacco, and alcohol, may also result in structural and functional abnormalities.

A woman is in for a routine prenatal checkup. The nurse is assessing her urine for glycosuria and proteinuria. Which findings are considered normal? (Select all that apply.) a. Dipstick assessment of trace to +1 b. <300 mg/24 hours c. Dipstick assessment of +2 glucose d. >300 mg/24 hours e. Albumin < 30 mg/24 hours

ANS: A, B, E Small amounts of protein in the urine are acceptable during pregnancy. The presence of protein in greater amounts may indicate renal problems. A dipstick assessment of +2 and proteinuria >300 mg/24 hours, and albuminuria greater than 30 mg/24 hours are excessive and should be further evaluated.

A woman has just moved to the United States from Mexico. She is 3 months pregnant and has arrived for her first prenatal visit. During her assessment interview, the nurse learns that the client has not had any immunizations. Which immunizations should she receive at this point in her pregnancy? (Select all that apply.) a. Tetanus b. Diphtheria c. Chickenpox d. Rubella e. Hepatitis B

ANS: A, B, E Vaccines consisting of killed viruses may be used. Those that may be administered during pregnancy include tetanus, diphtheria, recombinant hepatitis B, and rabies vaccines. Immunizations with live or attenuated viruses are contraindicated during pregnancy because of their potential teratogenicity. Live-virus vaccines include those for measles (rubeola and rubella), chickenpox, and mumps.

The diagnosis of pregnancy is based on which positive signs of pregnancy? (Select all that apply.) a. Identification of fetal heartbeat b. Palpation of fetal outline c. Visualization of the fetus d. Verification of fetal movement e. Positive hCG test

ANS: A, C, D Identification of a fetal heartbeat, the visualization of the fetus, and verification of fetal movement are all positive, objective signs of pregnancy. Palpation of fetal outline and positive hCG test are probable signs of pregnancy. A tumor also can be palpated. Medication and tumors may lead to false-positive results on pregnancy tests.

Relating to the fetal circulatory system, which special characteristics allow the fetus to obtain sufficient oxygen from the maternal blood? (Select all that apply.) a. Fetal hemoglobin (Hb) carries 20% to 30% more oxygen than maternal Hb. b. Fetal Hb carries 40% to 50% more oxygen than maternal Hb. c. Hb concentration is 50% higher than that of the mother. d. Fetal heart rate is 110 to 160 beats per minute. e. Fetal heart rate is 160 to 200 beats per minute.

ANS: A, C, D The following three special characteristics enable the fetus to obtain sufficient oxygen from maternal blood: (1) the fetal Hb carries 20% to 30% more oxygen; (2) the concentration is 50% higher than that of the mother; and (3) the fetal heart rate is 110 to 160 beats per minute, a cardiac output that is higher than that of an adult.

A probable cause for increasing infertility is the societal delay in pregnancy until later in life. What are the natural reasons for the decrease in female fertility? (Select all that apply.) a. Ovulation dysfunction b. Endocrine dysfunction c. Organ damage from toxins d. Endometriosis e. Tubal infections

ANS: A, C, D, E All of these factors may result in a cumulative effect, decreasing fertility in women. Male infertility is more often caused by unfavorable sperm production attributable to endocrine dysfunction or cumulative metabolic disease.

What type of cultural concern is the most likely deterrent to many women seeking prenatal care? a. Religion b. Modesty c. Ignorance d. Belief that physicians are evil

ANS: B A concern for modesty is a deterrent to many women seeking prenatal care. For some women, exposing body parts, especially to a man, is considered a major violation of their modesty. Many cultural variations are found in prenatal care. Even if the prenatal care described is familiar to a woman, some practices may conflict with the beliefs and practices of a subculture group to which she belongs.

A client states that she plans to breastfeed her newborn infant. What guidance would be useful for this new mother? a. The mother's intake of vitamin C, zinc, and protein can now be lower than during pregnancy. b. Caffeine consumed by the mother accumulates in the infant, who may be unusually active and wakeful. c. Critical iron and folic acid levels must be maintained. d. Lactating women can go back to their prepregnant caloric intake.

ANS: B A lactating woman needs to avoid consuming too much caffeine. Vitamin C, zinc, and protein levels need to be moderately higher during lactation than during pregnancy. The recommendations for iron and folic acid are lower during lactation. Lactating women should consume approximately 500 kcal more than their prepregnancy intake, at least 1800 kcal daily overall.

During the first trimester, which of the following changes regarding her sexual drive should a client be taught to expect? a. Increased sexual drive, because of enlarging breasts b. Decreased sexual drive, because of nausea and fatigue c. No change in her sexual drive d. Increased sexual drive, because of increased levels of female hormones

ANS: B A pregnant woman usually experiences a decrease, not an increase, in libido during the first trimester. Maternal physiologic changes, such as breast enlargement, nausea, fatigue, abdominal changes, perineal enlargement, leukorrhea, pelvic vasocongestion, and orgasmic responses, may affect sexuality and sexual expression. Libido may be depressed in the first trimester but often increases during the second and third trimesters. During pregnancy, the breasts may become enlarged and tender, which tends to interfere with coitus, thereby decreasing the desire to engage in sexual activity.

Assessment of a woman's nutritional status includes a diet history, medication regimen, physical examination, and relevant laboratory tests. Which finding might require consultation to a higher level of care? a. Oral contraceptive use may interfere with the absorption of iron. b. Illnesses that have created nutritional deficits, such as PKU, may require nutritional care before conception. c. The woman's socioeconomic status and educational level are not relevant to her examination; they are the province of the social worker. d. Testing for diabetes is the only nutrition-related laboratory test most pregnant women need.

ANS: B A registered dietitian can help with therapeutic diets. Oral contraceptive use may interfere with the absorption of folic acid. Iron deficiency can appear if placement of an intrauterine device (IUD) results in blood loss. A woman's finances can affect her access to good nutrition; her education (or lack thereof) can influence the nurse's teaching decisions. The nutrition-related laboratory test that pregnant women usually need is a screen for anemia.

Which action is the highest priority for the nurse when educating a pregnant adolescent? a. Emphasize the need to eliminate common teenage snack foods because they are high in fat and sodium. b. Determine the weight gain needed to meet adolescent growth, and add 35 pounds. c. Suggest that she not eat at fast-food restaurants to avoid foods of poor nutritional value. d. Realize that most adolescents are unwilling to make dietary changes during pregnancy.

ANS: B Adolescents should gain in the upper range of the recommended weight gain. They also need to gain weight that would be expected for their own normal growth. Changes in the diet should be kept at a minimum. Snack foods can be included in moderation, and other foods can be added to make up for lost nutrients. Eliminating fast foods would make the adolescent appear different to her peers. The client should be taught to choose foods that add needed nutrients. Adolescents are willing to make changes; however, they still have the need to be similar to their peers.

A woman who is 16 weeks pregnant asks the nurse, "Is it possible to tell by ultrasound if the baby is a boy or girl yet?" What is the best answer? a. "A baby's sex is determined as soon as conception occurs." b. "The baby has developed enough to enable us to determine the sex by examining the genitals through an ultrasound scan." c. "Boys and girls look alike until approximately 20 weeks after conception, and then they begin to look different." d. "It might be possible to determine your baby's sex, but the external organs look very similar right now."

ANS: B Although gender is determined at conception, the external genitalia of males and females look similar through the ninth week. By the twelfth week, the external genitalia are distinguishable as male or female.

Which sign of a potential complication is the most important for the nurse to share with the client? a. Constipation b. Alteration in the pattern of fetal movement c. Heart palpitations d. Edema in the ankles and feet at the end of the day

ANS: B An alteration in the pattern or amount of fetal movement may indicate fetal jeopardy. Constipation, heart palpitations, and ankle and foot edema are normal discomforts of pregnancy that occur in the second and third trimesters.

The musculoskeletal system adapts to the changes that occur throughout the pregnancy. Which musculoskeletal alteration should the client expect? a. Her center of gravity will shift backward. b. She will have increased lordosis. c. She will have increased abdominal muscle tone. d. She will notice decreased mobility of her pelvic joints.

ANS: B An increase in the normal lumbosacral curve (lordosis) develops, and a compensatory curvature in the cervicodorsal region develops to help her maintain balance. The center of gravity shifts forward. She will have decreased abdominal muscle tone and will notice increased mobility of her pelvic joints.

A 31-year-old woman believes that she may be pregnant. She took an over-the-counter (OTC) pregnancy test 1 week ago after missing her period; the test was positive. During her assessment interview, the nurse inquires about the woman's last menstrual period and asks whether she is taking any medications. The client states that she takes medicine for epilepsy. She has been under considerable stress lately at work and has not been sleeping well. Her physical examination does not indicate that she is pregnant. She has an ultrasound scan, which confirms that she is not pregnant. What is the most likely cause of the false-positive pregnancy test result? a. The pregnancy test was taken too early. b. Anticonvulsant medications may cause the false-positive test result. c. The woman has a fibroid tumor. d. She has been under considerable stress and has a hormone imbalance.

ANS: B Anticonvulsants may cause false-positive pregnancy test results. OTC pregnancy tests use enzyme-linked immunosorbent assay (ELISA) technology, which can yield positive results as soon as 4 days after implantation. Implantation occurs 6 to 10 days after conception. If the woman were pregnant, then she would be into her third week at this point (having missed her period 1 week ago). Fibroid tumors do not produce hormones and have no bearing on human chorionic gonadotropin (hCG) pregnancy tests. Although stress may interrupt normal hormone cycles (menstrual cycles), it does not affect hCG levels or produce positive pregnancy test results.

Many clients are concerned about the increased levels of mercury in fish and may be reluctant to include this source of nutrients in their diet. What is the best advice for the nurse to provide? a. Canned white tuna is a preferred choice. b. Shark, swordfish, and mackerel should be avoided. c. Fish caught in local waterways is the safest. d. Salmon and shrimp contain high levels of mercury.

ANS: B As a precaution, the pregnant client should avoid eating shark, swordfish, and mackerel, as well as the less common tilefish. High levels of mercury can harm the developing nervous system of the fetus. Assisting the client in understanding the differences between numerous sources of mercury is essential for the nurse. A pregnant client may eat as much as 12 ounces a week of canned light tuna; however, canned white, albacore, or tuna steaks contain higher levels of mercury and should be limited to no more than 6 ounces per week. Pregnant women and mothers of young children should check with local advisories about the safety of fish caught by families and friends in nearby bodies of water. If no information is available, then these fish sources should be avoided, limited to less than 6 ounces per week, or the only fish consumed that week. Commercially caught fish that is low in mercury includes salmon, shrimp, pollock, or catfish. The pregnant client may eat up to 12 ounces of commercially caught fish per week. Additional information on levels of mercury in commercially caught fish is available at www.cfsan.fda.gov.

Which statement regarding the structure and function of the placenta is correct? a. Produces nutrients for fetal nutrition b. Secretes both estrogen and progesterone c. Forms a protective, impenetrable barrier to microorganisms such as bacteria and viruses d. Excretes prolactin and insulin

ANS: B As one of its early functions, the placenta acts as an endocrine gland, producing four hormones necessary to maintain the pregnancy and to support the embryo or fetus: human chorionic gonadotropin (hCG), human placental lactogen (hPL), estrogen, and progesterone. The placenta does not produce nutrients. It functions as a means of metabolic exchange between the maternal and fetal blood supplies. Many bacteria and viruses can cross the placental membrane.

Which renal system adaptation is an anticipated anatomic change of pregnancy? a. Increased urinary output makes pregnant women less susceptible to urinary infections. b. Increased bladder sensitivity and then compression of the bladder by the enlarging uterus result in the urge to urinate even when the bladder is almost empty. c. Renal (kidney) function is more efficient when the woman assumes a supine position. d. Using diuretic agents during pregnancy can help keep kidney function regular.

ANS: B Bladder sensitivity and then compression of the bladder by the uterus result in the urge to urinate more often, even when the bladder is almost empty. A number of anatomic changes in pregnancy make a woman more susceptible to urinary tract infections. Renal function is more efficient when the woman lies in the lateral recumbent position and is less efficient when she is supine. Diuretic use during pregnancy can overstress the system and cause problems.

A 3-year-old girl's mother is 6 months pregnant. What concern is this child most likely to verbalize? a. How the baby will get out? b. How will the baby eat? c. Will you die having the baby? d. What color eyes will the baby have?

ANS: B By age 3 or 4 years, children like to be told the story of their own beginning and accept it being compared with the present pregnancy. They like to listen to the fetal heartbeat and feel the baby move. Sometimes they worry about how the baby is being fed and what it will wear. School-age children take a more clinical interest in their mother's pregnancy and may want to know "How did the baby get in there?" and "How will it get out?" Whether the child's mother will die does not tend to be the focus of her questions about the impending birth of a sibling. The baby's eye color does not tend to be the focus of children's questions about the impending birth of a sibling.

With regard to the assessment of female, male, or couple infertility, the nurse should be aware of which important information? a. The couple's religious, cultural, and ethnic backgrounds provide emotional clutter that does not affect the clinical scientific diagnosis. b. The investigation will take several months and can be very costly. c. The woman is assessed first; if she is not the problem, then the male partner is analyzed. d. Semen analysis is for men; the postcoital test is for women.

ANS: B Fertility assessment and diagnosis take time, money, and commitment from the couple. Religious, cultural, and ethnic-bred attitudes about fertility and related issues always have an effect on diagnosis and assessment. Both partners are systematically and simultaneously assessed, first as individuals and then as a couple. Semen analysis is for men; however, the postcoital test is for the couple.

Which statement concerning neurologic and sensory development in the fetus is correct? a. Brain waves have been recorded on an electroencephalogram as early as the end of the first trimester (12 weeks of gestation). b. Fetuses respond to sound by 24 weeks of gestation and can be soothed by the sound of the mother's voice. c. Eyes are first receptive to light at 34 to 36 weeks of gestation. d. At term, the fetal brain is at least one third the size of an adult brain.

ANS: B Hearing develops early and is fully developed at birth. Brain waves have been recorded at week 8. Eyes are receptive to light at 28 weeks of gestation. The fetal brain is approximately one fourth the size of an adult brain.

A client is seen at the clinic at 14 weeks of gestation for a follow-up appointment. At which level does the nurse expect to palpate the fundus? a. Nonpalpable above the symphysis at 14 weeks of gestation b. Slightly above the symphysis pubis c. At the level of the umbilicus d. Slightly above the umbilicus

ANS: B In normal pregnancies, the uterus grows at a predictable rate. It may be palpated above the symphysis pubis sometime between the 12th and 14th weeks of pregnancy. As the uterus grows, it may be palpated above the symphysis pubis sometime between the 12th and 14th weeks of pregnancy. At 14 weeks, the uterus is not yet at the level of the umbilicus. The fundus is not palpable above the umbilicus until 22 to 24 weeks of gestation.

The pancreas forms in the foregut during the 5th to 8th week of gestation. A client with poorly controlled gestational diabetes asks the nurse what the effects of her condition will be on the fetus. What is the best response by the nurse? Poorly controlled maternal gestational diabetes will: a. produce fetal hypoglycemia. b. result in a macrocosmic fetus. c. result in a microcosmic fetus. d. enhance lung maturation.

ANS: B Insulin is produced by week 20 of gestation. In the fetus of a mother with uncontrolled diabetes, maternal hypoglycemia produces fetal hypoglycemia and macrocosmia results. Hyperinsulinemia blocks lung maturation, placing the neonate at risk for respiratory distress.

To prevent gastrointestinal (GI) upset, when should a pregnant client be instructed to take the recommended iron supplements? a. On a full stomach b. At bedtime c. After eating a meal d. With milk

ANS: B Iron supplements taken at bedtime may reduce GI upset and should be taken at bedtime if abdominal discomfort occurs when iron supplements are taken between meals. Iron supplements are best absorbed if they are taken when the stomach is empty. Bran, tea, coffee, milk, and eggs may reduce absorption.

Which statement made by a lactating woman leads the nurse to believe that the client might have lactose intolerance? a. "I always have heartburn after I drink milk." b. "If I drink more than a cup of milk, I usually have abdominal cramps and bloating." c. "Drinking milk usually makes me break out in hives." d. "Sometimes I notice that I have bad breath after I drink a cup of milk."

ANS: B Lactose intolerance, which is an inability to digest milk sugar because of a lack of the enzyme lactose in the small intestine, is a problem that interferes with milk consumption. Milk consumption may cause abdominal cramping, bloating, and diarrhea in such people, although many lactose-intolerant individuals can tolerate small amounts of milk without symptoms. A woman with lactose intolerance is more likely to experience bloating and cramping, not heartburn. A client who breaks out in hives after consuming milk is more likely to have a milk allergy and should be advised to simply brush her teeth after consuming dairy products.

The nurse working with clients who have infertility concerns should be aware of the use of leuprolide acetate (Lupron) as a gonadotropin-releasing hormone (GnRH) agonist. For which condition would this medication be prescribed? a. Anovulatory cycles b. Uterine fibroids c. Polycystic ovary disease (PCOD) d. Luteal phase inadequacy

ANS: B Leuprolide acetate is used to treat endometriosis and uterine fibroids. Anovulatory cycles are treated with Clomid, Serophene, Pergonal, or Profasi, all of which stimulate ovulation induction. Metrodin is used to treat PCOD. Progesterone is used to treat luteal phase inadequacy.

A woman is 15 weeks pregnant with her first baby. She asks how long it will be before she feels the baby move. What is the nurse's best answer? a. "You should have felt the baby move by now." b. "Within the next month, you should start to feel fluttering sensations." c. "The baby is moving; however, you can't feel it yet." d. "Some babies are quiet, and you don't feel them move."

ANS: B Maternal perception of fetal movement usually begins 16 to 20 weeks after conception. Because this is her first pregnancy, movement is felt toward the later part of the 16- to 20-week time period. Stating, "you should have felt the baby move by now" is incorrect and may be an alarming statement to the client. Fetal movement should be felt by 16 to 20 weeks. If movement is not felt by the end of that time, then further assessment is necessary.

The nurse working with pregnant clients must seek to gain understanding of the process whereby women accept their pregnancy. Which statement regarding this process is most accurate? a. Nonacceptance of the pregnancy very often equates to a rejection of the child. b. Mood swings are most likely the result of worries about finances and a changed lifestyle, as well as profound hormonal changes. c. Ambivalent feelings during pregnancy are usually only expressed in emotionally immature or very young mothers. d. Conflicts such as not wanting to be pregnant or childrearing and career-related decisions need not be addressed during pregnancy because they will naturally resolve themselves after birth.

ANS: B Mood swings are natural and are likely to affect every woman to some degree. A woman may dislike being pregnant, refuse to accept it, and still love and accept the child. Ambivalent feelings about pregnancy are normal for the mature or immature woman and for the younger or older woman. Conflicts such as not wanting to be pregnant or childrearing and career-related decisions need to be resolved. The baby ends the pregnancy but not all the issues.

A newly married couple plans to use the natural family planning method of contraception. Understanding how long an ovum can live after ovulation is important to them. The nurse knows that his or her teaching was effective when the couple responds that an ovum is considered fertile for which period of time? a. 6 to 8 hours b. 24 hours c. 2 to 3 days d. 1 week

ANS: B Most ova remain fertile for approximately 24 hours after ovulation, much longer than 6 to 8 hours. However, ova do not remain fertile for 2 to 3 days or are viable for 1 week. If unfertilized by a sperm after 24 hours, the ovum degenerates and is reabsorbed.

A patient in her first trimester complains of nausea and vomiting. She asks, "Why does this happen?" What is the nurse's best response? a. "Nausea and vomiting are due to an increase in gastric motility." b. "Nausea and vomiting may be due to changes in hormones." c. "Nausea and vomiting are related to an increase in glucose levels." d. "Nausea and vomiting are caused by a decrease in gastric secretions."

ANS: B Nausea and vomiting are believed to be caused by increased levels of hormones, decreased gastric motility, and hypoglycemia. Gastric motility decreases during pregnancy. Glucose levels decrease in the first trimester. Although gastric secretions decrease, these secretions are not the primary cause of the nausea and vomiting.

Maternal nutritional status is an especially significant factor of the many that influence the outcome of pregnancy. Why is this the case? a. Maternal nutritional status is extremely difficult to adjust because of an individual's ingrained eating habits. b. Adequate nutrition is an important preventive measure for a variety of problems. c. Women love obsessing about their weight and diets. d. A woman's preconception weight becomes irrelevant.

ANS: B Nutritional status draws so much attention not only for its effect on a healthy pregnancy and birth but also because significant changes are within relatively easy reach. Pregnancy is a time when many women are motivated to learn about adequate nutrition and make changes to their diet that will benefit their baby. Pregnancy is not the time to begin a weight loss diet. Clients and their caregivers should still be concerned with appropriate weight gain.

A woman inquires about herbal alternative methods for improving fertility. Which statement by the nurse is most appropriate when informing the client on which herbal preparations may improve ovulation induction therapy? a. "You should avoid nettle leaf, dong quai, and vitamin E while you are trying to get pregnant." b. "You may want to try black cohosh or phytoestrogens." c. "You should take vitamins E and C, selenium, and zinc." d. "Herbs have no bearing on fertility."

ANS: B Ovulation therapy may have better outcomes when supplemented by black cohosh, progesterone, or plant estrogens. Antioxidant vitamins E and C, selenium, zinc, coenzyme 10, and ginseng have been shown to improve male fertility. Although most herbal remedies have not been clinically proven, many women find them helpful. They should be prescribed by a health care provider who has knowledge of herbalism.

A pregnant woman at 18 weeks of gestation calls the clinic to report that she has been experiencing occasional backaches of mild-to-moderate intensity. Which intervention should the nurse recommend? a. Kegel exercises b. Pelvic rock exercises c. Softer mattress d. Bed rest for 24 hours

ANS: B Pelvic rock exercises may help stretch and strengthen the abdominal and lower back muscles and relieve low back pain. Stretching and other exercises to relieve back pain should be performed several times a day. Kegel exercises increase the tone of the pelvic area, not the back. A softer mattress may not provide the support needed to maintain proper alignment of the spine and may contribute to back pain.

Significant advances have been made with most reproductive technologies. Which improvement has resulted in increased success related to preimplantation genetic diagnosis? a. Embryos are transferred at the cleavage stage. b. Embryos are transferred at the blastocyst stage. c. More than two embryos can be transferred at a time. d. Two cells are removed from each embryo.

ANS: B Preimplantation genetic diagnosis can be performed on a single cell removed from each embryo after 3 to 4 days. With the availability of extended culture mediums, embryos are transferred at the blastocyst stage (day 5), which increases the chance of a live birth, compared with the older practice of transferring embryos at the cleavage stage (day 3). No more than two embryos should be transferred at a time.

Ideally, when should prenatal care begin? a. Before the first missed menstrual period b. After the first missed menstrual period c. After the second missed menstrual period d. After the third missed menstrual period

ANS: B Prenatal care should begin soon after the first missed menstrual period. This offers the greatest opportunities to ensure the health of the expectant mother and her infant. Prenatal care before missing the first menstrual period is too early. It is unlikely the woman is even aware of the pregnancy. Ideally, prenatal visits should begin soon after the first period is missed. Beginning prenatal care after the third missed menstrual period is too late. The woman will have completed the first trimester by that time.

Which statement regarding the probable signs of pregnancy is most accurate? a. Determined by ultrasound b. Observed by the health care provider c. Reported by the client d. Confirmed by diagnostic tests

ANS: B Probable signs are those detected through trained examination. Fetal visualization is a positive sign of pregnancy. Presumptive signs are those reported by the client. The term diagnostic tests is open for interpretation. To actually diagnose pregnancy, one would have to see positive signs of pregnancy.

Which finding in the urinalysis of a pregnant woman is considered a variation of normal? a. Proteinuria b. Glycosuria c. Bacteria in the urine d. Ketones in the urine

ANS: B Small amounts of glucose may indicate physiologic spilling. The presence of protein could indicate kidney disease or preeclampsia. Urinary tract infections are associated with bacteria in the urine. An increase in ketones indicates that the patient is exercising too strenuously or has an inadequate fluid and food intake.

The client is instructed to place her thumb and forefinger on the areola and gently press inward. What is the purpose of this exercise? a. To check the sensitivity of the nipples b. To determine whether the nipple is everted or inverted c. To calculate the adipose buildup in the abdomen d. To see whether the fetus has become inactive

ANS: B Sometimes known as the pinch test, this exercise is used to determine whether the nipple is everted or inverted. Nipples must be everted to allow breastfeeding. The pinch does not determine the level of sensitivity of the nipples, nor is it not used to determine the level of adipose tissue in the abdomen. Fetal activity is not determined by using the pinch test.

Which time-based description of a stage of development in pregnancy is correct? a. Viability—22 to 37 weeks of gestation since the last menstrual period (assuming a fetal weight greater than 500 g) b. Term—pregnancy from the beginning of 38 weeks of gestation to the end of 42 weeks of gestation c. Preterm—pregnancy from 20 to 28 weeks of gestation d. Postdate—pregnancy that extends beyond 38 weeks of gestation

ANS: B Term is 38 to 42 weeks of gestation. Viability is the ability of the fetus to live outside the uterus before coming to term, or 22 to 24 weeks since the last menstrual period. Preterm is 20 to 37 weeks of gestation. Postdateor postterm is a pregnancy that extends beyond 42 weeks of gestation or what is considered the limit of full term.

During the physical examination of a client beginning prenatal care, which initial action is most important for the nurse to perform? a. Only women who show physical signs or meet the sociologic profile should be assessed for physical abuse. b. The client should empty her bladder before the pelvic examination. c. The distribution, amount, and quality of body hair are of no particular importance. d. The size of the uterus is discounted in the initial examination because it will be increasing in size during the second trimester.

ANS: B The nurse should instruct the client to empty her bladder. An empty bladder facilitates the examination and also provides an opportunity to obtain a urine sample for a number of tests. All women should be assessed for a history of physical abuse, particularly because the likelihood of abuse increases during pregnancy. Noting body hair is important because body hair reflects nutritional status, endocrine function, and hygiene. Particular attention is paid to the size of the uterus because it is an indication of the duration of gestation.

Which client might be well advised to continue condom use during intercourse throughout her pregnancy? a. Unmarried pregnant women b. Women at risk for acquiring or transmitting STIs c. All pregnant women d. Women at risk for candidiasis

ANS: B The objective of safer sex is to provide prophylaxis against the acquisition and transmission of STIs. Because these diseases may be transmitted to the woman and then to her fetus, condom use is recommended throughout the pregnancy if the woman is at risk for acquiring an STI. Pregnant women are encouraged to practice safer sex behaviors. An unmarried pregnant woman may be in a monogamous relationship and not require the use of a condom. The client should be educated as to what may place both herself and her fetus at risk. Any pregnant woman can develop candidiasis, which is an infection not related to condom use.

The maternity nurse is cognizant of what important structure and function of the placenta? a. As the placenta widens, it gradually thins to allow easier passage of air and nutrients. b. As one of its early functions, the placenta acts as an endocrine gland. c. The placenta is able to keep out most potentially toxic substances, such as cigarette smoke, to which the mother is exposed. d. Optimal blood circulation is achieved through the placenta when the woman is lying on her back or standing.

ANS: B The placenta produces four hormones necessary to maintain the pregnancy: hCG, hPL, estrogen, and progesterone. The placenta widens until 20 weeks of gestation and continues to grow thicker. Toxic substances such as nicotine and carbon monoxide readily cross the placenta into the fetus. Optimal circulation occurs when the woman is lying on her side.

Cardiac output increases from 30% to 50% by the 32nd week of pregnancy. What is the rationale for this change? a. To compensate for the decreased renal plasma flow b. To provide adequate perfusion of the placenta c. To eliminate metabolic wastes of the mother d. To prevent maternal and fetal dehydration

ANS: B The primary function of increased vascular volume is to transport oxygen and nutrients to the fetus via the placenta. Renal plasma flow increases during pregnancy. Assisting with pulling metabolic wastes from the fetus for maternal excretion is one purpose of the increased vascular volume.

Of which physiologic alteration of the uterus during pregnancy is it important for the nurse to alert the patient? a. Lightening occurs near the end of the second trimester as the uterus rises into a different position. b. Woman's increased urinary frequency in the first trimester is the result of exaggerated uterine antireflexion caused by softening. c. Braxton Hicks contractions become more painful in the third trimester, particularly if the woman tries to exercise. d. Uterine souffle is the movement of the fetus.

ANS: B The softening of the lower uterine segment is called the Hegar sign. In this position, the uterine fundus presses on the bladder, causing urinary frequency that is a normal change of pregnancy. Lightening occurs in the last 2 weeks of pregnancy, when the fetus descends. Braxton Hicks contractions become more defined in the final trimester but are not painful. Walking or exercise usually causes them to stop. The uterine souffle is the sound made by blood in the uterine arteries; it can be heard with a fetal stethoscope.

The nurse is providing health education to a pregnant client regarding the cardiovascular system. Which information is correct and important to share? a. A pregnant woman experiencing disturbed cardiac rhythm, such as sinus arrhythmia, requires close medical and obstetric observation no matter how healthy she may appear otherwise. b. Changes in heart size and position and increases in blood volume create auditory changes from 20 weeks of gestation to term. c. Palpitations are twice as likely to occur in twin gestations. d. All of the above changes will likely occur.

ANS: B These auscultatory changes should be discernible after 20 weeks of gestation. A healthy woman with no underlying heart disease does not need any therapy. The maternal heart rate increases in the third trimester, but palpitations may not necessarily occur, let alone double. Auditory changes are discernible at 20 weeks of gestation.

Which statement regarding multifetal pregnancy is incorrect? a. The expectant mother often develops anemia because the fetuses have a greater demand for iron. b. Twin pregnancies come to term with the same frequency as single pregnancies. c. The mother should be counseled to increase her nutritional intake and gain more weight. d. Backache and varicose veins often are more pronounced with a multifetal pregnancy.

ANS: B Twin pregnancies often end in prematurity. Serious efforts should be made to bring the pregnancy to term. A woman with a multifetal pregnancy often develops anemia, suffers more or worse backache, and needs to gain more weight. Counseling is needed to help her adjust to these conditions.

Which test is performed around the time of ovulation to diagnose the basis of infertility? a. Hysterosalpingogram b. Ultrasonography c. Laparoscopy d. Follicle-stimulating hormone (FSH) level

ANS: B Ultrasonography is performed around the time of ovulation to assess pelvic structures for abnormalities, to verify follicular development, and to assess the thickness of the endometrium. A hysterosalpingogram is scheduled 2 to 5 days after menstruation to avoid flushing a potentially fertilized ovum out through a uterine tube into the peritoneal cavity. Laparoscopy is usually scheduled early in the menstrual cycle. Hormone analysis is performed to assess endocrine function of the hypothalamic-pituitary-ovarian axis when menstrual cycles are absent or irregular.

A woman's obstetric history indicates that she is pregnant for the fourth time, and all of her children from previous pregnancies are living. One was born at 39 weeks of gestation, twins were born at 34 weeks of gestation, and another child was born at 35 weeks of gestation. What is her gravidity and parity using the GTPAL system? a. 3-1-1-1-3 b. 4-1-2-0-4 c. 3-0-3-0-3 d. 4-2-1-0-3

ANS: B Using the GTPAL system, 4-1-2-0-4 is the correct calculation of this woman's gravidity and parity. The numbers reflect the woman's gravidity and parity information. Her information is calculated as: G reflects the total number of times the woman has been pregnant; she is pregnant for the fourth time. T indicates the number of pregnancies carried to term, not the number of deliveries at term; only one of her pregnancies resulted in a fetus at term. P is the number of pregnancies that resulted in a preterm birth; the woman has had two pregnancies in which she delivered preterm. A signifies whether the woman has had any abortions or miscarriages before the period of viability; she has not. L signifies the number of children born who are currently living; the woman has four children. 3-1-1-1-3 is an incorrect calculation of this woman's gravidity and parity; 3-0-3-0-3 is an incorrect calculation of this woman's gravidity and parity; and 4-2-1-0-3 is an incorrect calculation of this woman's gravidity and parity.

Which statement best describes the rationale for the physiologic anemia that occurs during pregnancy? a. Physiologic anemia involves an inadequate intake of iron. b. Dilution of hemoglobin concentration occurs in pregnancy with physiologic anemia. c. Fetus establishes the iron stores. d. Decreased production of erythrocytes occur.

ANS: B When blood volume expansion is more pronounced and occurs earlier than the increase in red blood cells, the woman has physiologic anemia, which is the result of the dilution of hemoglobin concentration rather than inadequate hemoglobin. An inadequate intake of iron may lead to true anemia. The production of erythrocytes increases during pregnancy.

Women who have undergone an oophorectomy, have ovarian failure, or a genetic defect may be eligible to receive donor oocytes (eggs). Which statements regarding oocyte donation are accurate? (Select all that apply.) a. Donor is inseminated with semen from the parent. b. Donor eggs are fertilized with the male partner's sperm. c. Donors are under 35 years of age. d. Recipient undergoes hormonal stimulation. e. Ovum is placed into a surrogate.

ANS: B, C, D Oocyte donation is usually provided by healthy women under the age of 35 years, who are recruited and paid to undergo ovarian stimulation and oocyte retrieval. The donor eggs are fertilized in a laboratory with the male partner's sperm. The woman undergoes hormonal stimulation to allow the development of the uterine lining. Embryos are then transferred. A donor that is inseminated with the male partner's semen or receives the fertilized ovum and then carries it to gestation is known as a surrogate mother.

Foodborne illnesses can cause adverse effects for both mother and fetus. The nurse is in an ideal position to evaluate the client's knowledge regarding steps to prevent a foodborne illness. The nurse asks the client to "teach back" the fours simple steps of food preparation. What are they? (Select all that apply.) a. Purchase b. Clean c. Separate d. Cook e. Chill

ANS: B, C, D, E According to the U.S. Food and Drug Administration (2013), the "four simple steps" are: • Clean: Frequently cleanse hands, food preparation surfaces, and utensils. • Separate: Avoid contact among raw meat, fish, or poultry and other foods that will not be cooked before consumption. • Cook: Cook foods to the proper temperature. • Chill: Properly store foods, and promptly refrigerate.

The number of routine laboratory tests during follow-up visits is limited; however, those that are performed are essential. Which statements regarding group B Streptococcus (GBS) testing are correct? (Select all that apply.) a. Performed between 32 and 34 weeks of gestation. b. Performed between 35 and 37 weeks of gestation. c. All women should be tested. d. Only women planning a vaginal birth should be tested. e. Women with a history of GBS should be retested.

ANS: B, D, E GBS testing is recommended between 35 and 37 weeks of gestation; cultures collected earlier will not accurately predict the presence of GBS at birth. All women should be tested, even those planning an elective cesarean birth. Membranes may rupture early, requiring prophylactic antibiotics. Clients with a history of GBS should be retested.

A pregnant woman's diet consists almost entirely of whole grain breads and cereals, fruits, and vegetables. Which dietary requirement is the nurse most concerned about? a. Calcium b. Protein c. Vitamin B12 d. Folic acid

ANS: C A pregnant woman's diet is consistent with that followed by a strict vegetarian (vegan). Vegans consume only plant products. Because vitamin B12 is found in foods of animal origin, this diet is deficient in vitamin B12. Depending on the woman's food choices, a pregnant woman's diet may be adequate in calcium. Protein needs can be sufficiently met by a vegetarian diet. The nurse should be more concerned with the woman's intake of vitamin B12 attributable to her dietary restrictions. Folic acid needs can be met by enriched bread products.

Which statement regarding the development of the respiratory system is a high priority for the nurse to understand? a. The respiratory system does not begin developing until after the embryonic stage. b. The infant's lungs are considered mature when the L/S ratio is 1:1, at approximately 32 weeks of gestation. c. Maternal hypertension can reduce maternal-placental blood flow, accelerating lung maturity. d. Fetal respiratory movements are not visible on ultrasound scans until at least 16 weeks of gestation.

ANS: C A reduction in placental blood flow stresses the fetus, increases blood levels of corticosteroids, and thus accelerates lung maturity. The development of the respiratory system begins during the embryonic phase and continues into childhood. The infant's lungs are considered mature when the L/S ratio is 2:1, at approximately 35 weeks of gestation. Lung movements have been visualized on ultrasound scans at 11 weeks of gestation.

Pregnant adolescents are at greater risk for decreased BMI and "fad" dieting with which condition? a. Obesity b. Gestational diabetes c. Low-birth-weight babies d. High-birth-weight babies

ANS: C Adolescents tend to have lower BMIs. In addition, the fetus and the still-growing mother appear to compete for nutrients. These factors, along with inadequate weight gain, lend themselves to a higher incidence of low-birth-weight babies. Obesity is associated with a higher-than-normal BMI. Unless the teenager has type 1 diabetes, an adolescent with a low BMI is less likely to develop gestational diabetes. High-birth-weight or large-for-gestational age (LGA) babies are most often associated with gestational diabetes.

Some pregnant clients may complain of changes in their voice and impaired hearing. What should the nurse explain to the client concerning these findings? a. Voice changes are caused by decreased estrogen levels. b. Displacement of the diaphragm results in thoracic breathing. c. Voice changes and impaired hearing are due to the results of congestion and swelling of the upper respiratory tract. d. Increased blood volume causes changes in the voice.

ANS: C Although the diaphragm is displaced and the volume of blood is increased, neither causes changes in the voice nor impairs hearing. The key is that estrogen levels increase, not decrease, which causes the upper respiratory tract to become more vascular, which produces swelling and congestion in the nose and ears and therefore voice changes and impaired hearing.

A pregnant woman at 25 weeks of gestation tells the nurse that she dropped a pan last week and her baby jumped at the noise. Which response by the nurse is most accurate? a. "That must have been a coincidence; babies can't respond like that." b. "The fetus is demonstrating the aural reflex." c. "Babies respond to sound starting at approximately 24 weeks of gestation." d. "Let me know if it happens again; we need to report that to your midwife."

ANS: C Babies respond to external sound starting at approximately 24 weeks of gestation. Acoustic stimulations can evoke a fetal heart rate response. There is no such thing as an aural reflex. The last statement is inappropriate and may cause undue psychologic alarm to the client.

During the initial visit with a client who is beginning prenatal care, which action should be the highest priority for the nurse? a. The first interview is a relaxed, get-acquainted affair during which the nurse gathers some general impressions of his or her new client. b. If the nurse observed handicapping conditions, he or she should be sensitive and not inquire about them because the client will do that in her own time. c. The nurse should be alert to the appearance of potential parenting problems, such as depression or lack of family support. d. Because of legal complications, the nurse should not ask about illegal drug use; that is left to the physician.

ANS: C Besides these potential problems, the nurse needs to be alert to the woman's attitude toward keeping regular health care appointments. If the client lacks insurance, then the nurse may be able to direct her to resources that provide assistance for pregnant women (i.e., Women, Infants, and Children [WIC]; Medicaid). The initial interview needs to be planned, purposeful, and focused on specific content. A lot of ground must be covered. The nurse must be sensitive to special problems; he or she should inquire because discovering individual needs is important. A client with a chronic or handicapping condition might forget to mention it because she has adapted to it. Obtaining information on drug use is important and can be confidentially done. Actual testing for drug use requires the client's consent.

The nurse is having her first meeting with a couple experiencing infertility. The nurse has formulated the nursing diagnosis, "Deficient knowledge, related to lack of understanding of the reproductive process with regard to conception." Which nursing intervention does not apply to this diagnosis? a. Assess the current level of factors promoting conception. b. Provide information regarding conception in a supportive manner. c. Evaluate the couple's support system. d. Identify and describe the basic infertility tests.

ANS: C Evaluating the couple's support system would be a nursing action more suitable to the diagnosis, "Ineffective individual coping, related to the ability to conceive."

Which statement accurately describes the centering model of care? a. Group sessions begin with the first prenatal visit. b. Blood pressure (BP), weight, and urine dipsticks are obtained by the nurse at each visit. c. Approximately 8 to 12 women are placed in each gestational-age cohort group. d. Outcomes are similar to traditional prenatal care.

ANS: C Gestational-age cohorts comprise the groups, with approximately 8 to 12 women in each group. The groups remain intact throughout the pregnancy. Individual follow-up visits are scheduled as needed. Group sessions begin at 12 to 16 weeks of gestation and end with an early postpartum visit. Before the group sessions, the client has an individual assessment, physical examination, and history. At the beginning of each group meeting, clients measure their own BP, weight, and urine dips and enter these findings in their record. Fetal heart rate assessment and fundal height are obtained by the nurse. Results evaluating this approach have been very promising. In a recent study of adolescent clients, the number of LBW infants decreased and breastfeeding rates increased.

With regard to weight gain during pregnancy, the nurse should be aware of which important information? a. In pregnancy, the woman's height is not a factor in determining her target weight. b. Obese women may have their health concerns, but their risk of giving birth to a child with major congenital defects is the same as with women of normal weight. c. Women with inadequate weight gain have an increased risk of delivering a preterm infant with intrauterine growth restriction (IUGR). d. Greater than expected weight gain during pregnancy is almost always attributable to old-fashioned overeating.

ANS: C IUGR is associated with women with inadequate weight gain. The primary factor in making a weight gain recommendation is the appropriateness of the prepregnancy weight for the woman's height. Obese women are twice as likely as women of normal weight to give birth to a child with major congenital defects. Overeating is only one of several likely causes.

Which minerals and vitamins are usually recommended as a supplement in a pregnant client's diet? a. Fat-soluble vitamins A and D b. Water-soluble vitamins C and B6 c. Iron and folate d. Calcium and zinc

ANS: C Iron should generally be supplemented, and folic acid supplements are often needed because folate is so important in pregnancy. Fat-soluble vitamins should be supplemented as a medical prescription, as vitamin D might be for lactose-intolerant women. Water-soluble vitamin C is sometimes naturally consumed in excess; vitamin B6 is prescribed only if the woman has a very poor diet; and zinc is sometimes supplemented. Most women get enough calcium.

The nurse is providing education to a client regarding the normal changes of the breasts during pregnancy. Which statement regarding these changes is correct? a. The visibility of blood vessels that form an intertwining blue network indicates full function of the Montgomery tubercles and possibly an infection of the tubercles. b. The mammary glands do not develop until 2 weeks before labor. c. Lactation is inhibited until the estrogen level declines after birth. d. Colostrum is the yellowish oily substance used to lubricate the nipples for breastfeeding.

ANS: C Lactation is inhibited until after birth. The visible blue network of blood vessels is a normal outgrowth of a richer blood supply. The mammary glands are functionally complete by midpregnancy. Colostrum is a creamy white-to-yellow premilk fluid that can be expressed from the nipples before birth.

What should the nurse be cognizant of concerning the client's reordering of personal relationships during pregnancy? a. Because of the special motherhood bond, a woman's relationship with her mother is even more important than with the father of the child. b. Nurses need not get involved in any sexual issues the couple has during pregnancy, particularly if they have trouble communicating them to each other. c. Women usually express two major relationship needs during pregnancy: feeling loved and valued and having the child accepted by the father. d. The woman's sexual desire is likely to be highest in the first trimester because of the excitement and because intercourse is physically easier.

ANS: C Love and support help a woman feel better about her pregnancy. The most important person to the pregnant woman is usually the father of the child. Nurses can facilitate communication between partners about sexual matters if, as is common, they are nervous about expressing their worries and feelings to one another. The second trimester is the time when a woman's sense of well-being, along with certain physical changes, increases her desire for sex. Sexual desire is down in the first and third trimesters.

An infertile woman is about to begin pharmacologic treatment. As part of the regimen, she will take purified FSH (Metrodin). The nurse instructs her that this medication is administered in the form of what? a. Intranasal spray b. Vaginal suppository c. Intramuscular (IM) injection d. Tablet

ANS: C Metrodin is only administered by IM injection, and the dose may vary. An intranasal spray or a vaginal suppository are not appropriate routes for Metrodin, nor can Metrodin be given by mouth in tablet form.

The labor and delivery nurse is preparing a client who is severely obese (bariatric) for an elective cesarean birth. Which piece of specialized equipment will not likely be needed when providing care for this pregnant woman? a. Extra-long surgical instruments b. Wide surgical table c. Temporal thermometer d. Increased diameter blood pressure cuff

ANS: C Obstetricians today are seeing an increasing number of morbidly obese pregnant women weighing 400, 500, and 600 pounds. To manage their conditions and to meet their logistical needs, a new medical subspecialty,bariatric obstetrics, has arisen. Extra-wide blood pressure cuffs, scales that can accommodate up to 880 pounds, and extra-wide surgical tables designed to hold the weight of these women are used. Special techniques for ultrasound examination and longer surgical instruments for cesarean birth are also required. A temporal thermometer can be used for a pregnant client of any size.

Which clinical finding in a primiparous client at 32 weeks of gestation might be an indication of anemia? a. Ptyalism b. Pyrosis c. Pica d. Decreased peristalsis

ANS: C Pica (a desire to eat nonfood substances) is an indication of iron deficiency and should be evaluated. Cravings include ice, clay, and laundry starch. Ptyalism (excessive salivation), pyrosis (heartburn), and decreased peristalsis are normal findings.

A couple arrives for their first appointment at an infertility center. Which of the following is a noninvasive test performed during the initial diagnostic phase of testing? a. Hysterosalpingogram b. Endometrial biopsy c. Sperm analysis d. Laparoscopy

ANS: C Sperm analysis is the basic noninvasive test performed during initial diagnostic phase of testing for male infertility. Radiographic film examination allows visualization of the uterine cavity after the instillation of a radiopaque contrast medium through the cervix. The endometrial biopsy is an invasive procedure, during which a small cannula is introduced into the uterus and a portion of the endometrium is removed for histologic examination. Laparoscopy is useful to view the pelvic structures intraperitoneally and is an invasive procedure.

The measurement of lecithin in relation to sphingomyelin (lecithin/sphingomyelin [L/S] ratio) is used to determine fetal lung maturity. Which ratio reflects fetal maturity of the lungs? a. 1.4:1 b. 1.8:1 c. 2:1 d. 1:1

ANS: C The L/S ratio indicates a 2:1 ratio of lecithin to sphingomyelin, which is an indicator of fetal lung maturity and occurs at approximately the middle of the third trimester. L/S ratios of 1.4:1, 1.8:1, and 1:1 each indicate immaturity of the fetal lungs.

A woman's cousin gave birth to an infant with a congenital heart anomaly. The woman asks the nurse when such anomalies occur during development. Which response by the nurse is most accurate? a. "We don't really know when such defects occur." b. "It depends on what caused the defect." c. "Defects occur between the third and fifth weeks of development." d. "They usually occur in the first 2 weeks of development."

ANS: C The cardiovascular system is the first organ system to function in the developing human. Blood vessel and blood formation begins in the third week, and the heart is developmentally complete in the fifth week. "We don't really know when such defects occur" is an inaccurate statement. Regardless of the cause, the heart is vulnerable during its period of development—in the third to fifth weeks; therefore, the statement, "They usually occur in the first 2 weeks of development" is inaccurate

While obtaining a diet history, the nurse might be told that the expectant mother has cravings for ice chips, cornstarch, and baking soda. Which nutritional problem does this behavior indicate? a. Preeclampsia b. Pyrosis c. Pica d. Purging

ANS: C The consumption of foods low in nutritional value or of nonfood substances (e.g., dirt, laundry starch) is called pica. Preeclampsia is a vasospastic disease process encountered after 20 weeks of gestation. Characteristics of preeclampsia include increasing hypertension, proteinuria, and hemoconcentration. Pyrosis is a burning sensation in the epigastric region, otherwise known as heartburn. Purging refers to self-induced vomiting after consuming large quantities of food.

What is the primary role of the nonpregnant partner during pregnancy? a. To provide financial support b. To protect the pregnant woman from "old wives' tales" c. To support and nurture the pregnant woman d. To make sure the pregnant woman keeps prenatal appointments

ANS: C The partner's primary role in pregnancy is to nurture the pregnant woman and respond to her feelings of vulnerability. Although financial support is important, it is not the partner's primary role in pregnancy. Protecting the pregnant woman from "old wives' tales" is not the partner's role. The woman's partner can encourage the client to keep all appointments; however, this is not the most important role during the pregnancy.

A client arrives for her initial prenatal examination. This is her first child. She asks the nurse, "How does my baby get air inside my uterus?" What is the correct response by the nurse? a. "The baby's lungs work in utero to exchange oxygen and carbon dioxide." b. "The baby absorbs oxygen from your blood system." c. "The placenta provides oxygen to the baby and excretes carbon dioxide into your bloodstream." d. "The placenta delivers oxygen-rich blood through the umbilical artery to the baby's abdomen."

ANS: C The placenta delivers oxygen-rich blood through the umbilical vein, not the artery, to the fetus and excretes carbon dioxide into the maternal bloodstream. The fetal lungs do not function as respiratory gas exchange in utero. The baby does not simply absorb oxygen from a woman's blood system; rather, blood and gas transport occur through the placenta.

If a client's normal prepregnancy diet contains 45 g of protein daily, how many more grams of protein should she consume per day during pregnancy? a. 5 b. 10 c. 25 d. 30

ANS: C The recommended intake of protein for the pregnant woman is 70 g. Therefore, additional protein intakes of 5, 10, or 15 g would be inadequate to meet protein needs during pregnancy. A protein intake of 30 g is more than would be necessary and would add extra calories.

Dental care during pregnancy is an important component of good prenatal care. Which instruction regarding dental health should the nurse provide? a. Regular brushing and flossing may not be necessary during early pregnancy because it may stimulate the woman who is already nauseated to vomit. A cleaning is all that is necessary. b. Dental surgery, in particular, is contraindicated during pregnancy and should be delayed until after delivery. c. If dental treatment is necessary, then the woman will be most comfortable with it in the second trimester. d. If a woman has dental anxiety, then dental care may interfere with the expectant mother's need to practice conscious relaxation and to prepare for labor.

ANS: C The second trimester is the best time for dental treatment because the woman will be able to sit most comfortably in the dental chair. Dental care, such as brushing with a fluoride toothpaste, is especially important during pregnancy. Periodontal disease has been linked to both preterm labor and low-birth-weight (LBW) infants. Emergency dental surgery is permissible; however, the mother must clearly understand the risks and benefits. Conscious relaxation is useful and may even help the woman get through any dental appointments, but it is not a reason to avoid them.

Which consideration is essential for the nurse to understand regarding follow-up prenatal care visits? a. The interview portions become more intensive as the visits become more frequent over the course of the pregnancy. b. Monthly visits are scheduled for the first trimester, every 2 weeks for the second trimester, and weekly for the third trimester. c. During the abdominal examination, the nurse should be alert for supine hypotension. d. For pregnant women, a systolic BP of 130 mm Hg and a diastolic BP of 80 mm Hg is sufficient to be considered hypertensive.

ANS: C The woman lies on her back during the abdominal examination, possibly compressing the vena cava and aorta, which can cause a decrease in BP and a feeling of faintness. The interview portion of the follow-up examinations is less extensive than in the initial prenatal visits, during which so much new information must be gathered. Monthly visits are routinely scheduled for the first and second trimesters; visits increase to every 2 weeks at week 28 and to once a week at week 36. For pregnant women, hypertension is defined as a systolic BP of 140 mm Hg or higher and a diastolic BP of 90 mm Hg or higher.

At a routine prenatal visit, the nurse explains the development of the fetus to her client. At approximately ____ weeks of gestation, lecithin is forming on the alveolar surfaces, the eyelids open, and the fetus measures approximately 27 cm crown to rump and weighs approximately 1110 g. The client is how many weeks of gestation at today's visit? a. 20 b. 24 c. 28 d. 30

ANS: C These milestones in human development occur at 28 weeks of gestation. These milestones have not occurred by 20 or 24 weeks of gestation but have been reached before 30 weeks of gestation.

A couple comes in for an infertility workup, having attempted to achieve pregnancy for 2 years. The woman, 37 years of age, has always had irregular menstrual cycles but is otherwise healthy. The man has fathered two children from a previous marriage and had a vasectomy reversal 2 years ago. The man has had two normal semen analyses, but the sperm seem to be clumped together. What additional testing is needed? a. Testicular biopsy b. Antisperm antibodies c. FSH level d. Examination for testicular infection

ANS: C This scenario does not indicate that the woman has had any testing related to her irregular menstrual cycles. Hormone analysis is performed to assess endocrine function of the hypothalamic-pituitary-ovarian axis when menstrual cycles are absent or irregular. Determining the blood levels of prolactin, FSH, luteinizing hormone (LH), estradiol, progesterone, and thyroid hormones may be necessary to diagnose the cause of the woman's irregular menstrual cycles. A testicular biopsy is indicated only in cases of azoospermia (no sperm cells) or severe oligospermia (low number of sperm cells). Although unlikely to be the case because the husband has already produced children, antisperm antibodies may be produced by the man against his own sperm. Examination for testicular infection would be performed before semen analysis. Furthermore, infection would affect spermatogenesis.

A 27-year-old pregnant woman had a preconceptual body mass index (BMI) of 19. What is this client's total recommended weight gain during pregnancy? a. 20 kg (44 lb) b. 16 kg (35 lb) c. 12.5 kg (27.5 lb) d. 10 kg (22 lb)

ANS: C This woman has a normal BMI and should gain 11.5 to 16 kg during her pregnancy. A weight gain of 20 kg (44 lb) is unhealthy for most women; a weight gain of 16 kg (35 lb) is at the high end of the range of weight this woman should gain in her pregnancy; and a weight gain of 10 kg (22 lb) is appropriate for an obese woman. This woman has a normal BMI, which indicates that her weight is average.

A pregnant woman at 10 weeks of gestation jogs three or four times per week. She is concerned about the effect of the exercise on the fetus. Which guidance should the nurse provide? a. "You don't need to modify your exercising any time during your pregnancy." b. "Stop exercising because it will harm the fetus." c. "You may find that you need to modify your exercise to walking later in your pregnancy, around the seventh month." d. "Jogging is too hard on your joints; switch to walking now."

ANS: C Typically, running should be replaced with walking around the seventh month of pregnancy. The nurse should inform the woman that she may need to reduce her exercise level as the pregnancy progresses. Physical activity promotes a feeling of well-being in pregnant women. It improves circulation, promotes relaxation and rest, and counteracts boredom. Simple measures should be initiated to prevent injuries, such as warm-up and stretching exercises to prepare the joints for more strenuous exercise.

A woman arrives at the clinic for a pregnancy test. Her last menstrual period (LMP) was February 14, 2015. What is the client's expected date of birth (EDB)? a. September 17, 2015 b. November 7, 2015 c. November 21, 2015 d. December 17, 2015

ANS: C Using the Nägele's rule, the EDB is calculated by subtracting 3 months from the month of the LMP and adding 7 days + 1 year to the day of the LMP. Therefore, with an LMP of February 14, 2015, her due date is November 21, 2015. September 17, 2015, is too short a period to complete a normal pregnancy. Using the Nägele's rule, an EDB of November 7, 2015, is 2 weeks early. December 17, 2015, is almost a month past the correct EDB.

A couple is attempting to cope with an infertility problem. They want to know what they can do to preserve their emotional equilibrium. What is the nurse's most appropriate response? a. "Tell your friends and family so that they can help you." b. "Talk only to other friends who are infertile, because only they can help." c. "Get involved with a support group. I'll give you some names." d. "Start adoption proceedings immediately, because adopting an infant can be very difficult."

ANS: C Venting negative feelings may unburden the couple. A support group may provide a safe haven for the couple to share their experiences and gain insight from others' experiences. Although talking about their feelings may unburden them of negative feelings, infertility can be a major stressor that affects the couple's relationships with family and friends. Limiting their interactions to other infertile couples may be a beginning point for addressing psychosocial needs. However, depending on where the other couple is in their own recovery process, limiting their interactions may not be of assistance to them. Telling the couple to start adoption proceedings immediately is not supportive of the psychosocial needs of this couple and may be detrimental to their well-being.

Which procedure falls into the category of micromanipulation techniques of the follicle? (Select all that apply.) a. Intrauterine insemination b. Preimplantation genetic diagnosis c. Intracytoplasmic sperm injection (ISCI) d. Assisted hatching e. IVF-ET

ANS: C, D ISCI makes it possible to achieve fertilization even with a few or poor quality sperm by introducing sperm beneath the zone pellucid into the egg. Another micromanipulation technique is assisted hatching. An infrared laser breaks through the thick or tough zone pellucid, enabling the blastocyst to hatch.

During pregnancy, many changes occur as a direct result of the presence of the fetus. Which of these adaptations meet this criterion? (Select all that apply.) a. Leukorrhea b. Development of the operculum c. Quickening d. Ballottement e. Lightening

ANS: C, D, E Quickening is the first recognition of fetal movements or "feeling life." Quickening is often described as a flutter and is felt earlier in the multiparous woman than in the primiparous woman. Passive movement of the unengaged fetus is referred to as ballottement. Lightening occurs when the fetus begins to descend into the pelvis and occurs 2 weeks before labor in the nulliparous woman and at the start of labor in the multiparous woman. Leukorrhea is a white or slightly gray vaginal discharge that develops in response to cervical stimulation by estrogen and progesterone. Mucus fills the cervical canal creating a plug otherwise known as the operculum. The operculum acts as a barrier against bacterial invasion during the pregnancy.

A woman has come to the clinic for preconception counseling because she wants to start trying to get pregnant. Which guidance should she expect to receive? a. "Discontinue all contraception now." b. "Lose weight so that you can gain more during pregnancy." c. "You may take any medications you have been regularly taking." d. "Make sure you include adequate folic acid in your diet."

ANS: D A healthy diet before conception is the best way to ensure that adequate nutrients are available for the developing fetus. A woman's folate or folic acid intake is of particular concern in the periconception period. Neural tube defects are more common in infants of women with a poor folic acid intake. Depending on the type of contraception that she has been using, discontinuing all contraception at this time may not be appropriate. Advising this client to lose weight now so that she can gain more during pregnancy is also not appropriate advice. Depending on the type of medications the woman is taking, continuing to take them regularly may not be appropriate.

Which pregnant woman should strictly follow weight gain recommendations during pregnancy? a. Pregnant with twins b. In early adolescence c. Shorter than 62 inches or 157 cm d. Was 20 pounds overweight before pregnancy

ANS: D A weight gain of 5 to 9 kg will provide sufficient nutrients for the fetus. Overweight and obese women should be advised to lose weight before conception to achieve the best pregnancy outcomes. A higher weight gain in twin gestations may help prevent low birth weights. Adolescents need to gain weight toward the higher acceptable range, which provides for their own growth, as well as for fetal growth. In the past, women of short stature were advised to restrict their weight gain; however, evidence to support these guidelines has not been found.

What is the most basic information that a nurse should be able to share with a client who asks about the process of conception? a. Ova are considered fertile 48 to 72 hours after ovulation. b. Sperm remain viable in the woman's reproductive system for an average of 12 to 24 hours. c. Conception is achieved when a sperm successfully penetrates the membrane surrounding the ovum. d. Implantation in the endometrium occurs 6 to 10 days after conception.

ANS: D After implantation, the endometrium is called the decidua. Ova are considered fertile for approximately 24 hours after ovulation. Sperm remain viable in the woman's reproductive system for an average of 2 to 3 days. Penetration of the ovum by the sperm is called fertilization. Conception occurs when the zygote, the first cell of the new individual, is formed.

In her work with pregnant women of different cultures, a nurse practitioner has observed various practices that seemed unfamiliar. The nurse practitioner has learned that cultural rituals and practices during pregnancy seem to have one purpose in common. Which statement best describes that purpose? a. To promote family unity b. To ward off the "evil eye" c. To appease the gods of fertility d. To protect the mother and fetus during pregnancy

ANS: D Although many cultures consider pregnancy normal, certain practices are expected of women of all cultures to ensure a good outcome. Cultural prescriptions tell women what to do, and cultural proscriptions establish taboos. The purposes of these practices are to prevent maternal illness resulting from a pregnancy-induced imbalanced state and to protect the vulnerable fetus. Promoting family unity is important, although not usually the premise for cultural rituals and practices. Warding off the "evil eye" may be specific to one particular culture; however, it is not the primary purpose of these practices. Appeasing the gods of fertility is not the impetus behind cultural rituals

Although remarkable developments have occurred in reproductive medicine, assisted reproductive therapies are associated with numerous legal and ethical issues. Nurses can provide accurate information about the risks and benefits of treatment alternatives to enable couples to make informed decisions about their choice of treatment. Which concern is unnecessary for the nurse to address before treatment? a. Risks of multiple gestation b. Whether or how to disclose the facts of conception to offspring c. Freezing embryos for later use d. Financial ability to cover the cost of treatment

ANS: D Although the method of payment is important, obtaining this information is not the responsibility of the nurse. Many states have mandated some form of insurance to assist couples with coverage for infertility. Multiple gestation is a risk of treatment of which the couple needs to be aware. To minimize the chance of multiple gestation, generally only three or fewer embryos are transferred. The couple should be informed that multifetal reduction may be needed. Nurses can provide anticipatory guidance on this matter. Depending on the therapy chosen, donor oocytes, sperm, embryos, or a surrogate mother may be needed. Couples who have excess embryos frozen for later transfer must be fully informed before consenting to the procedure. A decision must be made regarding the disposal of embryos in the event of death or divorce or if the couple no longer wants the embryos at a future time.

Who is most likely to experience the phenomenon of someone other than the mother-to-be having pregnancy-like symptoms such as nausea and weight gain? a. Mother of the pregnant woman b. Couple's teenage daughter c. Sister of the pregnant woman d. Expectant father

ANS: D An expectant father's experiencing of his partner's pregnancy-like symptoms is called the couvade syndrome. The mother of the pregnant woman is unlikely to experience this phenomenon. She may be excited about becoming a grandmother or see her daughter's pregnancy as a reminder that she is getting old. A couple's teenage daughter is usually preoccupied with her own sexual development and may have difficulty accepting the overwhelming evidence of her parents' sexual activity. It is the father of the pregnant woman, not the sister, who experiences these symptoms.

While assessing the vital signs of a pregnant woman in her third trimester, the client complains of feeling faint, dizzy, and agitated. Which nursing intervention is appropriate? a. Have the patient stand up, and then retake her BP. b. Have the patient sit down, and then hold her arm in a dependent position. c. Have the patient lie supine for 5 minutes, and then recheck her BP on both arms. d. Have the patient turn to her left side, and then recheck her BP in 5 minutes.

ANS: D BP is affected by maternal position during pregnancy. The supine position may cause occlusion of the vena cava and descending aorta. Turning the pregnant woman to a lateral recumbent position alleviates pressure on the blood vessels and quickly corrects supine hypotension. Pressures are significantly higher when the client is standing. This option causes an increase in systolic and diastolic pressures. The arm should be supported at the same level of the heart. The supine position may cause occlusion of the vena cava and descending aorta, creating hypotension.

A pregnant couple has formulated a birth plan and is reviewing it with the nurse at an expectant parent's class. Which aspect of their birth plan should be considered potentially unrealistic and require further discussion with the nurse? a. "My husband and I have agreed that my sister will be my coach because he becomes anxious with regard to medical procedures and blood. He will be nearby and check on me every so often to make sure everything is okay." b. "We plan to use the techniques taught in the Lamaze classes to reduce the pain experienced during labor." c. "We want the labor and birth to take place in a birthing room. My husband will come in the minute the baby is born." d. "Regardless of the circumstances, we do not want the fetal monitor used during labor because it will interfere with movement and doing effleurage."

ANS: D Because monitoring is essential to assess fetal well-being, fetal monitoring is not a factor that can be determined by the couple. The nurse should fully explain its importance. The option for intermittent electronic monitoring could be explored if this is a low-risk pregnancy and as long as labor is normally progressing. The birth plan is a tool with which parents can explore their childbirth options; however, the plan must be viewed as tentative. Having the woman's sister as her coach with her husband nearby is an acceptable request for a laboring woman. Using breathing techniques to alleviate pain is a realistic part of a birth plan. Not all fathers are able to be present during the birth; however, this couple has made a realistic plan that works for their specific situation.

Which structure is responsible for oxygen and carbon dioxide transport to and from the maternal bloodstream? a. Decidua basalis b. Blastocyst c. Germ layer d. Chorionic villi

ANS: D Chorionic villi are fingerlike projections that develop out of the trophoblast and extend into the blood-filled spaces of the endometrium. The villi obtain oxygen and nutrients from the maternal bloodstream and dispose carbon dioxide and waste products into the maternal blood. The decidua basalis is the portion of the decidua (endometrium) under the blastocyst where the villi attach. The blastocyst is the embryonic development stage after the morula; implantation occurs at this stage. The germ layer is a layer of the blastocyst.

A man smokes two packs of cigarettes a day. He wants to know if smoking is contributing to the difficulty he and his wife are having getting pregnant. Which guidance should the nurse provide? a. "Your sperm count seems to be okay in the first semen analysis." b. "Only marijuana cigarettes affect sperm count." c. "Although smoking has no effect on sperm count, it can give you lung cancer." d. "Smoking can reduce the quality of your sperm."

ANS: D Cigarette smoking has detrimental effects on sperm and has been associated with abnormal sperm, a decreased number of sperm, and chromosomal damage. The nurse may suggest a smoking cessation program to increase the fertility of the male partner. Sperm counts vary from day to day and are dependent on emotional and physical status and sexual activity. Therefore, a single analysis may be inconclusive. A minimum of two analyses must be performed several weeks apart to assess male fertility. Marijuana use may depress the number and motility of sperm. Smoking is indeed a causative agent for lung cancer.

The major source of nutrients in the diet of a pregnant woman should be composed of what? a. Simple sugars b. Fats c. Fiber d. Complex carbohydrates

ANS: D Complex carbohydrates supply the pregnant woman with vitamins, minerals, and fiber. The most common simple carbohydrate is table sugar, which is a source of energy but does not provide any nutrients. Fats provide 9 kcal in each gram, in contrast to carbohydrates and proteins, which provide only 4 kcal in each gram. Fiber is primarily supplied by complex carbohydrates.

To reassure and educate their pregnant clients regarding changes in their blood pressure, nurses should be cognizant of what? a. A blood pressure cuff that is too small produces a reading that is too low; a cuff that is too large produces a reading that is too high. b. Shifting the client's position and changing from arm to arm for different measurements produces the most accurate composite blood pressure reading at each visit. c. Systolic blood pressure slightly increases as the pregnancy advances; diastolic pressure remains constant. d. Compression of the iliac veins and inferior vena cava by the uterus contributes to hemorrhoids in the later stage of a term pregnancy.

ANS: D Compression of the iliac veins and inferior vena cava by the uterus contributes to hemorrhoids in the later stage of a term pregnancy. This compression also leads to varicose veins in the legs and vulva. The tightness of a blood pressure cuff that is too small produces a reading that is too high; similarly, the looseness of a cuff that is too large results in a reading that is too low. Because maternal positioning affects readings, blood pressure measurements should be obtained in the same arm and with the woman in the same position. The systolic blood pressure generally remains constant but may decline slightly as the pregnancy advances. The diastolic blood pressure first decreases and then gradually increases.

A woman who is 8 months pregnant asks the nurse, "Does my baby have any antibodies to fight infection?" What is the most appropriate response by the nurse? a. "Your baby has all the immunoglobulins necessary: immunoglobulin G (IgG), immunoglobulin M (IgM), and immunoglobulin A (IgA)." b. "Your baby won't receive any antibodies until he is born and you breastfeed him." c. "Your baby does not have any antibodies to fight infection." d. "Your baby has IgG and IgM."

ANS: D During the third trimester, IgG is the only immunoglobulin that crosses the placenta; it provides passive acquired immunity to specific bacterial toxins. However, the fetus produces IgM by the end of the first trimester. IgA immunoglobulins are not produced by the baby. Therefore, by the third trimester, the fetus has both IgG and IgM. Breastfeeding supplies the newborn infant with IgA.

A woman in the 34th week of pregnancy reports that she is very uncomfortable because of heartburn. Which recommendation would be appropriate for this client? a. Substitute other calcium sources for milk in her diet. b. Lie down after each meal. c. Reduce the amount of fiber she consumes. d. Eat five small meals daily.

ANS: D Eating small, frequent meals may help with heartburn, nausea, and vomiting. Substituting other calcium sources for milk, lying down after eating, and reducing fiber intake are inappropriate dietary suggestions for all pregnant women and do not alleviate heartburn.

A woman is 3 months pregnant. At her prenatal visit she tells the nurse that she does not know what is happening; one minute she is happy that she is pregnant and the next minute she cries for no reason. Which response by the nurse is most appropriate? a. "Don't worry about it; you'll feel better in a month or so." b. "Have you talked to your husband about how you feel?" c. "Perhaps you really don't want to be pregnant." d. "Hormone changes during pregnancy commonly result in mood swings."

ANS: D Explaining that hormone changes can result in mood swings is an accurate statement and the most appropriate response by the nurse. Telling the woman not to worry dismisses her concerns and is not the most appropriate response. Although the woman should be encouraged to share her feelings, asking if she has spoken to her husband about them is not the most appropriate response and does not provide her with a rationale for the psychosocial dynamics of her pregnancy. Suggesting that the woman does not want to be pregnant is completely inappropriate and deleterious to the psychologic well-being of the woman. Hormonal and metabolic adaptations often cause mood swings in pregnancy. The woman's responses are normal. She should be reassured about her feelings.

hCG is an important biochemical marker for pregnancy and therefore the basis for many tests. Which statement regarding hCG is true? a. hCG can be detected as early as weeks after conception. b. hCG levels gradually and uniformly increase throughout pregnancy. c. Significantly lower-than-normal increases in the levels of hCG may indicate a postdate pregnancy. d. Higher-than-normal levels of hCG may indicate an ectopic pregnancy or Down syndrome.

ANS: D Higher hCG levels also could be a sign of a multiple gestation. hCG can be detected as early as 7 to 10 days after conception. The hCG levels fluctuate during pregnancy, peaking, declining, stabilizing, and then increasing again. Abnormally slow increases may indicate impending miscarriage.

Which vitamins or minerals may lead to congenital malformations of the fetus if taken in excess by the mother? a. Zinc b. Vitamin D c. Folic acid d. Vitamin A

ANS: D If taken in excess, vitamin A causes a number of problems. An analog of vitamin A appears in prescribed acne medications, which must not be taken during pregnancy. Zinc, vitamin D, and folic acid are all vital to good maternity and fetal health and are highly unlikely to be consumed in excess.

Pregnancy hormones prepare the vagina for stretching during labor and birth. Which change related to the pelvic viscera should the nurse share with the client? a. Because of a number of changes in the cervix, abnormal Papanicolaou (Pap) tests are easier to evaluate. b. Quickening is a technique of palpating the fetus to engage it in passive movement. c. The deepening color of the vaginal mucosa and cervix (Chadwick sign) usually appears in the second trimester or later as the vagina prepares to stretch during labor. d. Increased vascularity of the vagina increases sensitivity and may lead to a high degree of arousal, especially in the second trimester.

ANS: D Increased sensitivity and an increased interest in sex sometimes go together and frequently occur during the second trimester. These cervical changes make evaluation of abnormal Pap tests more difficult. Quickening is the first recognition of fetal movements by the mother. Ballottement is a technique used to palpate the fetus. The Chadwick sign appears from the 6 to 8 weeks of gestation.

Which gastrointestinal alteration of pregnancy is a normal finding? a. Insufficient salivation (ptyalism) is caused by increases in estrogen. b. Acid indigestion (pyrosis) begins early but declines throughout pregnancy. c. Hyperthyroidism often develops (temporarily) because hormone production increases. d. Nausea and vomiting rarely have harmful effects on the fetus and may be beneficial.

ANS: D Normal nausea and vomiting rarely produce harmful effects and may be less likely to result in miscarriage or preterm labor. Ptyalism is excessive salivation that may be caused by a decrease in unconscious swallowing or by stimulation of the salivary glands. Pyrosis begins as early as the first trimester and intensifies through the third trimester. Increased hormone production does not lead to hyperthyroidism in pregnant women.

Male fertility declines slowly after age 40 years; however, no cessation of sperm production analogous to menopause in women occurs in men. What condition is not associated with advanced paternal age? a. Autosomal dominant disorder b. Schizophrenia c. Autism spectrum disorder d. Down syndrome

ANS: D Paternal age older than 40 years is associated with an increased risk for autosomal dominant disorder, schizophrenia, and autism spectrum disorder in their offspring. Although Down syndrome can occur in any pregnancy, it is often associated with advanced maternal age.

Many pregnant women have questions regarding work and travel during pregnancy. Which education is a priority for the nurse to provide? a. Women should sit for as long as possible and cross their legs at the knees from time to time for exercise. b. Women should avoid seat belts and shoulder restraints in the car because they press on the fetus. c. Metal detectors at airport security checkpoints can harm the fetus if the woman passes through them a number of times. d. While working or traveling in a car or on an airplane, women should arrange to walk around at least every hour or so.

ANS: D Periodic walking helps prevent thrombophlebitis. Pregnant women should avoid sitting or standing for long periods and crossing the legs at the knees. Pregnant women must wear lap belts and shoulder restraints. The most common injury to the fetus comes from injury to the mother. Metal detectors at airport security checkpoints do not harm fetuses.

Which hormone is essential for maintaining pregnancy? a. Estrogen b. hCG c. Oxytocin d. Progesterone

ANS: D Progesterone is essential for maintaining pregnancy; it does so by relaxing smooth muscles, which reduces uterine activity and prevents miscarriage. Estrogen plays a vital role in pregnancy, but it is not the primary hormone for maintaining pregnancy. hCG levels rise at implantation but decline after 60 to 70 days. Oxytocin stimulates uterine contractions.

A client states that she does not drink milk. Which foods should the nurse encourage this woman to consume in greater amounts to increase her calcium intake? a. Fresh apricots b. Canned clams c. Spaghetti with meat sauce d. Canned sardines

ANS: D Sardines are rich in calcium. Fresh apricots, canned clams, and spaghetti with meat sauce are not high in calcium

Which sign or symptom is considered a first-trimester warning sign and should be immediately reported by the pregnant woman to her health care provider? a. Nausea with occasional vomiting b. Fatigue c. Urinary frequency d. Vaginal bleeding

ANS: D Signs and symptoms that must be reported include severe vomiting, fever and chills, burning on urination, diarrhea, abdominal cramping, and vaginal bleeding. These symptoms may be signs of complications of the pregnancy. Nausea with occasional vomiting is a normal first-trimester complaint. Although it may be worrisome or annoying to the mother, it is not usually an indication of a problem with the pregnancy. Fatigue is common during the first trimester. Because of physiologic changes that happen during pregnancy, clients should be taught that urinary frequency is normal.

Which statement regarding gamete intrafallopian transfer (GIFT) is most accurate? a. Semen is collected after laparoscopy. b. Women must have two normal fallopian tubes. c. Ovulation spontaneously occurs. d. Ova and sperm are transferred to one tube.

ANS: D Similar to in vitro fertilization (IVF), GIFT requires the woman to have at least one normal tube. Ovulation is induced, and the oocytes are aspirated during laparoscopy. Semen is collected before laparoscopy. The ova and sperm are then transferred to one uterine tube, permitting natural fertilization and cleavage.

A pregnant woman's diet may not meet her increased need for folates. Which food is a rich source of this nutrient? a. Chicken b. Cheese c. Potatoes d. Green leafy vegetables

ANS: D Sources of folates include green leafy vegetables, whole grains, fruits, liver, dried peas, and beans. Chicken and cheese are excellent sources of protein but are poor sources for folates. Potatoes contain carbohydrates and vitamins and minerals but are poor sources for folates.

Numerous changes in the integumentary system occur during pregnancy. Which change persists after birth? a. Epulis b. Chloasma c. Telangiectasia d. Striae gravidarum

ANS: D Striae gravidarum, or stretch marks, reflect a separation within the underlying connective tissue of the skin. They usually fade after birth, although they never completely disappear. An epulis is a red, raised nodule on the gums that easily bleeds; it disappears or shrinks after giving birth. Chloasma, or the mask of pregnancy, is a blotchy, brown hyperpigmentation of the skin over the cheeks, nose, and forehead, especially in dark-complexioned pregnant women. Chloasma usually fades after the birth. Telangiectasia, or vascular spiders, are tiny, star-shaped or branchlike, slightly raised, pulsating end-arterioles usually found on the neck, thorax, face, and arms. They occur as a result of elevated levels of circulating estrogen and usually disappear after birth.

Some of the embryo's intestines remain within the umbilical cord during the embryonic period. What is the rationale for this development of the gastrointestinal system? a. Umbilical cord is much larger at this time than it will be at the end of pregnancy. b. Intestines begin their development within the umbilical cord. c. Nutrient content of the blood is higher in this location. d. Abdomen is too small to contain all the organs while they are developing.

ANS: D The abdominal contents grow more rapidly than the abdominal cavity; therefore, part of their development takes place in the umbilical cord. By 10 weeks of gestation, the abdomen is large enough to contain them. Intestines begin their development within the umbilical cord but only because the liver and kidneys occupy most of the abdominal cavity. Blood supply is adequate in all areas.

Nutrition is an alterable and important preventive measure for a variety of potential problems such as low birth weight and prematurity. While completing the physical assessment of the pregnant client, the nurse is able to evaluate the client's nutritional status by observing a number of physical signs. Which physical sign indicates to the nurse that the client has unmet nutritional needs? a. Normal heart rate, rhythm, and blood pressure b. Bright, clear, and shiny eyes c. Alert and responsive with good endurance d. Edema, tender calves, and tingling

ANS: D The physiologic changes of pregnancy may complicate the interpretation of physical findings. Lower extremity edema often occurs when caloric and protein deficiencies are present; however, edema in the lower extremities may also be a common physical finding during the third trimester. Completing a thorough health history and physical assessment and requesting further laboratory testing, if indicated, are essential for the nurse. The malnourished pregnant client may display rapid heart rate, abnormal rhythm, enlarged heart, and elevated blood pressure. A client receiving adequate nutrition will have bright, shiny eyes with no sores and moist, pink membranes. Pale or red membranes, dryness, infection, dull appearance of the cornea, or blue sclerae are signs of poor nutrition. A client who is alert and responsive with good endurance is well nourished. A listless, cachectic, easily fatigued, and tired presentation would be an indication of a poor nutritional status.

A woman who is 14 weeks pregnant tells the nurse that she always had a glass of wine with dinner before she became pregnant. She has abstained during her first trimester and would like to know if it is safe for her to have a drink with dinner now. Which guidance should the nurse provide? a. "Since you're in your second trimester, there's no problem with having one drink with dinner." b. "One drink every night is too much. One drink three times a week should be fine." c. "Since you're in your second trimester, you can drink as much as you like." d. "Because no one knows how much or how little alcohol it takes to cause fetal problems, the best course is to abstain throughout your pregnancy."

ANS: D The statement "Because no one knows how much or how little alcohol it takes to cause fetal problems, the best course is to abstain throughout your pregnancy" is accurate. A safe level of alcohol consumption during pregnancy has not yet been established. Although the consumption of occasional alcoholic beverages may not be harmful to the mother or her developing fetus, complete abstinence is strongly advised.

What represents a typical progression through the phases of a woman's establishing a relationship with the fetus? a. Accepts the fetus as distinct from herself—accepts the biologic fact of pregnancy—has feelings of caring and responsibility. b. Fantasizes about the child's gender and personality—views the child as part of herself—becomes introspective. c. Views the child as part of herself—has feelings of well-being—accepts the biologic fact of the pregnancy. d. "I am pregnant"—"I am going to have a baby"—"I am going to be a mother."

ANS: D The woman first centers on herself as pregnant, then on the baby as an entity separate from herself, and then on her responsibilities as a mother. The expressions "I am pregnant," "I am going to have a baby," and "I am going to be a mother" sum up the progression through the three phases. In phase one, the woman views the child as part of herself and not as a separate being. This is only the first step of the progression through phases of attachment. Accepting the fetus as distinct from herself occurs during the second phase of emotional attachment. Fantasizing about the child's sex and personality based on fetal activity occurs during the third phase of attachment.

A woman who is 16 weeks pregnant has come in for a follow-up visit with her significant other. To reassure the client regarding fetal well-being, which is the highest priority action for the nurse to perform? a. Assess the fetal heart tones with a Doppler stethoscope. b. Measure the girth of the woman's abdomen. c. Complete an ultrasound examination (sonogram). d. Offer the woman and her family the opportunity to listen to the fetal heart tones.

ANS: D To provide the parents with the greatest sense of reassurance, the nurse should offer to have the client and her significant other the chance to listen to their baby's heartbeat. A fetoscope can detect the fetal heart rate around 20 weeks of gestation. Doppler can detect the fetal heart rate between 10 and 12 weeks and should be performed as part of routine fetal assessment. Abdominal girth is not a valid measure for determining fetal well-being. Fundal height is an important measure that should be determined with precision, with the same technique and positioning of the client consistently used at every prenatal visit. Routine ultrasound examinations are recommended in early pregnancy; they date the pregnancy and provide useful information about the health of the fetus. However, they are not necessary at each prenatal visit.

Which action is the first priority for the nurse who is assessing the influence of culture on a client's diet? a. Evaluate the client's weight gain during pregnancy. b. Assess the socioeconomic status of the client. c. Discuss the four food groups with the client. d. Identify the food preferences and methods of food preparation common to the client's culture.

ANS: D Understanding the client's food preferences and how she prepares food will assist the nurse in determining whether the client's culture is adversely affecting her nutritional intake. An evaluation of a client's weight gain during pregnancy should be included for all clients, not only for clients from different cultural backgrounds. The socioeconomic status of the client may alter the nutritional intake but not the cultural influence. Teaching the food groups to the client should come after assessing her food preferences.

Which are considered probable signs of pregnancy? Select all that apply. Quickening Nausea and vomiting Abdominal distension Compressibility of lower uterus Auscultation of fetal heart tones Visualization of the fetus by ultrasound

Abdominal distension Abdominal distension is considered a probable sign of pregnancy as the growing uterus causes the stomach to expand. Compressibility of lower uterus Uterine softening is considered a probable sign of pregnancy and is indicated by compressibility of the lower uterus on palpation.

diastasis recti abdominis

Abdominal wall muscles separated

The rate of fertility declines dramatically after age 35. While explaining the cause of this rapid decline in fertility to the client, the nurse is aware that the primary reason for this is related to: Endometriosis Abnormalities of oocytes Infection Metabolic disease

Abnormalities of oocytes By age 40, the total number of ovarian follicles is diminishing and the quality of the remaining eggs is poor. Endometriosis is more common in women who delay childbearing until after age 30. Like infection and metabolic disease, it is a cumulative factor that may contribute to age-related infertility.

How long are ova considered fertile?

About 24 hours after ovultion

How much semen do males ejaculate?

About a teaspoon of semen 200 to 500 million sperms transit time to the site of fertilization - 5 min, but average 4-6 hrs

A pregnant patient should be instructed to include which information in her birth plan? Select all that apply. Activity during labor Methods of pain relief Timing of contractions Fetal monitoring interpretation Positioning during pushing/birth Support persons present during labor/birth

Activity during labor Activity during labor should be included in a birth plan. This informs the health care provider of the woman's preference for ambulation during labor. Allowing women to have choices increases their confidence and decreases anxiety related to labor and birth. Methods of pain relief Methods of pain relief should be included in a birth plan. This informs the health care provider of the woman's preference for pain relief, such as natural comfort measures, IV pain medication, and/or epidural anesthesia. Allowing women to have choices increases their confidence and decreases anxiety related to labor and birth. Positioning during pushing/birth Positioning during pushing/birth should be included in a birth plan. This informs the health care provider of the woman's preference for positions while pushing. Allowing women to have choices increases their confidence and decreases anxiety related to labor and birth. Support persons present during labor/birth Support persons present during labor/birth should be included in a birth plan. This will allow for the woman to have more control and less anxiety during labor and birth.

Why is it important to maintain adequate iron levels during pregnancy?

Additional iron is need to transfer to the fetus. Protective measure: During pregnancy plasma increases more than RBCs causing decrease in hemoglobin concentration (physiologic anemia- normal for preg)

A patient with a family history of developmental disability refuses a recommended amniocentesis without explanation. Which patient education is appropriate? Select all that apply. Affirm that the patient has the right to refuse any procedure. Explain to the patient that amniocentesis can be used to diagnose developmental disabilities. Inform the patient that refusing the amniocentesis may hinder the effectiveness of prenatal care. Inform the patient that prenatal diagnosis can give the family more time to prepare for a child with special needs. Remind the patient that the health care provider ordering the procedure may simply order a test besides amniocentesis if the patient refuses.

Affirm that the patient has the right to refuse any procedure. Affirming the patient's right to refuse treatment provides reassurance to the patient and respects the patient's autonomy. Correct Explain to the patient that amniocentesis can be used to diagnose developmental disabilities. Explaining the purpose of amniocentesis as it relates to the patient's history provides the patient with context necessary to make an informed decision. Inform the patient that prenatal diagnosis can give the family more time to prepare for a child with special needs. Informing the patient of the potential benefits of amniocentesis as it relates to the patient's history provides context to address a potential knowledge deficit that may have influenced the patient's refusal.

semen analysis

After 2-5 days of abstinence from ejaculation basic test complete semen analysis, assesses sperm number, morphology, motility

Zygote intrafallopian transfer

After in vitro fertilization the ova are placed in one uterine tube during the zygote stage

Collaborative Care

After obtaining information through the assessment process, the data are analyzed to identify deviations from the norm and unique needs of the pregnant woman and her family.

Which age group would be at risk for interrupted psychological adaptation to pregnancy? Ages 13-18 Ages 20-25 Ages 25-30 Ages 30-35

Ages 13-18 Adolescents, or females aged 13-18, would be at risk for interrupted psychological adaptation to pregnancy.

Which situations can negatively affect a woman's ability to psychologically adapt to pregnancy? Select all that apply. Marriage Alcoholism First pregnancy Domestic violence Supportive extended family

Alcoholism Substance abuse can inhibit a woman's ability to psychologically adapt to pregnancy. Domestic violence Domestic violence can negatively affect a woman's ability to psychologically adapt to pregnancy.

A woman who is 32 weeks pregnant is informed by the nurse that a danger sign of pregnancy could be: Constipation Alteration in the pattern of fetal movement Heart palpitations Edema in the ankles and feet at the end of the day

Alteration in the pattern of fetal movement An alteration in the pattern or amount of fetal movement may indicate fetal jeopardy. Constipation is a normal discomfort of pregnancy that occurs in the second and third trimesters. Heart palpitations are a normal change related to pregnancy; they are most likely to occur during the second and third trimesters. As the pregnancy progresses, edema in the ankles and feet at the end of the day is not uncommon.

Clothing/ culturally

Although most cultural groups do not prescribe specific clothing for pregnancy, modesty is an expectation for many.

A woman is at 36 weeks' gestation when her toddler hits her in the abdomen with a whiffle ball bat. Which auxiliary component protects the fetus from this insult? Placenta Amniotic fluid Amnion membrane Chorion membrane

Amniotic fluid Amniotic fluid is an auxiliary component composed of a protective liquid and contained by the amniotic sac; it provide cushioning from maternal abdominal impact.

Which woman would be at risk of impaired psychological adaptation to pregnancy? An adolescent with an unplanned pregnancy A woman who has a close relationship with her mother A woman who has been trying to conceive for two years A woman who adheres to recommendations by the health care provider

An adolescent with an unplanned pregnancy An adolescent would be at risk for impaired psychological adaptation to pregnancy.

nuchal cord

An umbilical cord that is wrapped around the fetus's neck.

Increased awareness of the need to breathe as pregnancy progresses

And essential component of prenatal health assessment of pregnant women is the determination of vital signs. And expected change and vital sign findings as a result of pregnancy is

Which placental function has the potential to cause harm to the fetus? Gas exchange Waste removal Antibody transfer Endocrine production

Antibody transfer The passage of antibodies from mother to fetus is most often protective, although it may be harmful in the case of blood incompatibility. In this case, the mother's antibodies may then destroy fetal erythrocytes, causing fetal anemia or, even, fetal death.

In reviewing the history of a woman who wants to become pregnant, which medication profile would indicate a potential concern relative to toxic exposure? Select all that apply. Tylenol OTC occasionally for a headache; twice last week Anticonvulsant for seizure disorder Lithium for bipolar disorder Coumadin for atrial fibrillation Multivitamins once a day

Anticonvulsant for seizure disorder A patient being treated with an anticonvulsant or lithium is at risk for toxic effects during pregnancy. Warfarin (Coumadin) can put a patient at risk during pregnancy. Although acetaminophen (Tylenol) can have toxic effects on the liver, the reported frequency is not a concern at this time. Taking multivitamins is a healthy recommended option. Lithium for bipolar disorder A patient being treated with an anticonvulsant or lithium is at risk for toxic effects during pregnancy. Warfarin (Coumadin) can put a patient at risk during pregnancy. Although acetaminophen (Tylenol) can have toxic effects on the liver, the reported frequency is not a concern at this time. Taking multivitamins is a healthy recommended option. Coumadin for atrial fibrillation A patient being treated with an anticonvulsant or lithium is at risk for toxic effects during pregnancy. Warfarin (Coumadin) can put a patient at risk during pregnancy. Although acetaminophen (Tylenol) can have toxic effects on the liver, the reported frequency is not a concern at this time. Taking multivitamins is a healthy recommended option.

Vitamin E Role in relation to Pregnancy and Lactation:

Antioxidant (protects cell membranes from damage), esp. Important for Preventing Breakdown of Red Blood Cells (RBCs)

GI system changes in pregnancy

Appetite fluctuations, morning sickness, pica, hyperemic and swollen gums, epulis (benign tumor on the gingiva), ptyalism (excessive salivation), hiatal hernia, constipation, gallstones, abdominal discomfort, pyrosis (heartburn)

What health risks are associated w/ excessive weight gain in pregnant women?

Arteriosclerosis heart disease (thickening and hardening of the walls of the arteries), diabetes, hypertension.

What is Mitosis?

Asexual reproduction when cells divide in two and produce a replica of itself body cells replicate toyeild 2 cells with the same genetic make up as the parent cell 1. cell makes a copy of its DNA and then it divides facilitates growth and development or cell replacement

A woman reports dissatisfaction when using her vaginal hormonal ring and asks about starting another method of contraception. Which steps should the nurse take? Select all that apply. Ask the woman what qualities she is looking for in a contraceptive method. Provide the woman with information about multiple other contraceptive methods. Ask the woman what she finds dissatisfying about her current contraceptive method. Instruct the woman about proper vaginal ring usage and plan on discussing switching methods at the next visit. Inform the woman that learning how to use another contraceptive method may decrease the overall effectiveness of her contraception.

Ask the woman what qualities she is looking for in a contraceptive method. Assessing the woman's contraceptive needs helps the nurse determine which contraceptive methods may be right for the woman. It increases the efficacy of nursing education. Correct Provide the woman with information about multiple other contraceptive methods. Providing the woman with information about multiple other methods gives the woman a choice, and it allows the woman to determine which methods she believes would be right for her. Correct Ask the woman what she finds dissatisfying about her current contraceptive method. Asking what the woman finds dissatisfying about her current method of contraception allows the nurse to determine which methods might be better suited for the woman's needs.

When does the placenta stop growing?

At 20 weeks

When is hearing fully developed?

At birth

When does the placenta begin to form?

At implantation

When are the kidneys fully developed?

At term GFR is low kidneys cannot concenyrate urine newborns susceptible to overhydration and dehydration

An infertile man is being treated with Viagra (sildenafil citrate) for erectile dysfunction (ED). Which of the following is a contraindication for this medication? A. Preexisting diagnosis of herpes simplex 2 B. Nitroglycerin ingestion for angina pectoris C. Retinal damage from type I diabetes mellitus D. Complications after resection of the prostate

B It is unsafe to take Viagra while also taking nitroglycerin for angina.

A client is receiving Pergonal (menotropins) intramuscularly for ovarian stimulation. Which of the following is a common side effect of this therapy? A. Piercing rectal pain B. Mood swings C. Visual disturbances D. Jerky tremors

B Mood swings and depression are common side effects of the hormonal therapy.

A client is to receive Pergonal (menotropins) injections for infertility prior to in-vitro fertilization. Which of the following is the expected action of this medication? A. Prolongation of the luteal phase B. Stimulation of ovulation C. Suppression of menstruation D. Promotion of cervical mucus production

B Pergonal is administered to infertile women to increase follicular growth and maturation of the follicles and to stimulate ovulation.

Which instruction by the nurse should be included in the teaching plan for an infertile woman who has been shown to have a 28-day biphasic menstrual cycle? A. Douche with a cider vinegar solution immediately before having intercourse B. Schedule intercourse every day from day 8 to day 14 of the menstrual cycle C. Be placed on follicle-stimulating hormone therapy by the fertility specialist D. Assess the basal body temperature pattern for at least 6 more months

B This action is recommended. Pregnancy is most likely to occur with daily intercourse from 6 days before ovulation up to the day of ovulation.

A nurse is educating a client who has been diagnosed with infertility on how to complete a basal body temperature chart to determine her ovulation pattern. The client states, "I really don't want to take my temperature every day. Is there any other way to find out if and when I ovulate?" A. "There are a number of other ways to determine ovulation, but they all require you to be examined by an obstetrician every month." B. "A test you can do at home requires you to spit on a microscopic slide and then look at the slide under a microscope." C. "You can test your vaginal discharge each day to determine when you should have intercourse because the hormone progesterone is elevated." D. "Although there are some tests that you can perform at home, they all cost well over a hundred dollars to purchase."

B This statement is correct. One of the at-home ovulation predictor kits requires women to place saliva on a microscopic slide and, after allowing the saliva to dry, to look at the slide under a microscope. If ovulation is occurring, the saliva appears ferned, that is, the image on the slide looks like the leaflets of a fern. The test detects the presence of high levels of estrogen in the woman's saliva.

when teaching a class of preg women, about fetal development the nurse would include which statements? (Select all) A)the sex of your baby is determined by the 9th week of preg B )Your baby will be able to suck and hiccup while in your uterus c )the baby heart begins to pump blood during the 10th week of preg d the baby heart beat will be audible using a special ultra sound stethoscope beginning at the 18th week of preg e You should be able to feel your baby move by week16 to 20 of preg f by the 24th week of your preg you will notice the baby responding to sounds in the environment, including music and your voice

B )Your baby will be able to suck and hiccup while in your uterus e You should be able to feel your baby move by week16 to 20 of preg f by the 24th week of your preg you will notice the baby responding to sounds in the environment, including music and your voice

A woman demonstrates and understanding of the importance of increasing her intake of foods that are good sources of folic acid (50 mcg or more) when she includes which foods in her diet? (Select all that apply) A. Baked haddock B. Lentil soup C. Scrambled eggs D. fruit salad of papaya and oranges E. Steamed asparagus F. Corn on the cob

B, D, E B. Lentil soup D. fruit salad of papaya and oranges E. Steamed asparagus

A 25 year old pregnant woman is at 10 weeks of gestation. Her BMI is calculated to be 24. Regarding weight gain during pregnancy, the nurse should recommend: A. A total weight gain of 18kg B. First-trimester of 1-2 kg C. Weight gain of 0.4kg each week for 40 weeks D. weight gain of 3 kg per month during the second and third trimesters

B. First-trimester of 1-2 kg

A woman must assess herself for signs that ovulation is occurring. Which of the following sign is associated with ovulation? A. reduction in level of LH in the urine 12-24 hrs prior to ovulation B. Spinnbarkeit C. Drop in basal body temp following ovulation D. Increased thickness of cervical mucus

B. Spinnbarkeit

What vitamin can't vegetarian women get from their diets?

B12. Only found in foods of animal origin. They are recommend to consume B12 fortified foods. risks associated with deficiencies: neuro deficits in mom, swollen tongue, megablastic anemia)

How is a pregnant females weight gain evaluated?

BMI is used to evaluate appropriate weight for height.

When does the fetus begin to lie down stores of brown fat?

Between 26 and 30 WEEKS To prepare for extrauterine cold stress

What is the developing embryo recognized as?

Blastocyst

Which develops to become part of the placenta and fetal membranes? Zygote Morula Blastocyst Conceptus

Blastocyst This choice shows that the student nurse understands that the outer layer of blastocyst cells develops into the placenta and fetal membranes, while the inner layer develops into the fetus.

Which statements explain what occurs during the fusion of the sperm and ovum? Select all that apply. The sperm enters the ovum. Both the sperm head and tail change. Fertilization is complete within 12 hours. Nuclei of the gametes cluster at the center of ovum. Changes in the ovum prevent other sperm from entering.

Both the sperm head and tail change. The sperm head enlarges, and the tail degenerates; therefore, both the sperm head and tail change during fusion. Nuclei of the gametes cluster at the center of ovum. During fusion, nuclei of the gametes move toward the ovum's center, where the membranes surrounding the nuclei touch and dissolve.

Foods Providing 20 mcg or more per serving of Folate:

Bread (1 slice) Egg (1 large) Corn (1/2 cup)

When is the GI system complete?

By 36 WEEKS

When does the fetus have both rods and cones in the retina?

By 7 MONTHS

When is oogenesis established?

By the 16th WEEK

When is the fetus able to distinguish taste?

By the 5th WEEK

A couple is seeking infertility counseling. During the history, it is noted that the man is a cancer survivor, drinks one beer every night with dinner, and takes a sauna every day after work. The response provided by the nurse should be based on which of the following? A. It is unlikely that any of these factors is affecting his fertility. B. Daily alcohol consumption could be altering his sperm count. C. Sperm may be malformed when exposed to the heat of the sauna. D. Cancer survivors have the same fertility rates as healthy males.

C The high temperature of the sauna could alter the number and morphology of the sperm.

A couple is seeking advice regarding actions that they can take to increase their potential of becoming pregnant. Which of the following recommendations should the nurse give to the couple? A. The couple should use vaginal lubricants during intercourse B. The couple should delay having intercourse until the day of ovulation C. The woman should refrain from douching D. The man should be on top during intercourse

C The woman should refrain from douching. Douching can change the normal flora and the pH in the vagina, making the environment hostile to the sperm.

A client who is undergoing ovarian stimulation for infertility calls the infertility nurse and states, "My abdomen feels very bloated, my clothes are very tight, and my urine is very dark." Which of the following is the appropriate statement for the nurse to make at this time? A. "Please take a urine sample to the lab so they can check it for an infection." B. "Those changes indicate that you are likely already pregnant." C. "It is important for you to come into the office to be examined today." D. "Abdominal bloating is an expected response to the medications."

C This client should be seen by her infertility doctor.

A couple has been told that the male partner, who is healthy, is producing no sperm "because he has cystic fibrosis." Which of the following explanations is accurate in relation to this statement? A. Since the man is healthy, he could not possibly have cystic fibrosis B. Men with cystic fibrosis often have no epididymis C. The expressivity of cystic fibrosis is variable D. Cystic fibrosis is a respiratory illness having nothing to do with reproduction

C This statement is correct. Cystic fibrosis can be expressed in a number of ways. Some affected individuals have very serious illness resulting in early death, while others experience few symptoms.

Match the prenatal education classes with their appropriate description. C-section Postpartum Breastfeeding Childbirth preparation

C-section Class covers surgical technique and what to expect after delivery Postpartum Class covers what to expect after delivery Breastfeeding Class covers lactation, includes education related to proper latch, positioning, milk supply, pumping, and supplementation Childbirth preparation Class covers labor and birth process

A pregnant women at 6 weeks of gestation tells her nurse-midwife that she has been experiencing nausea with occasional vomiting everyday. As an effective relief measure, the nurse should recommend: A. Eating starchy foods such as buttery popcorn and peanut butter with crackers in the morning before getting out of bed B Avoid eating before going to bed at night C. Altering eating patterns to small meals every 2-3 hours D. Skipping a meal if nausea is experienced

C. Altering eating patterns to small meals every 2-3 hours

30-year old pregnant woman with a BMI of 31 asks the nurse about recommendations for diet and weight gain during pregnancy. What should the nurse tell this woman? A. Council her to begin a lifestyle change for weight reduction B. recommend a total weight gain goal of 4kg during pregnancy C. Set a weight gain goal of 0.2kg per week during the second and third trimesters D. Limit her third-trimester calorie increase to no more than 600 kcal more than prepregnant needs

C. Set a weight gain goal of 0.2kg per week during the second and third trimesters

A woman taking human menopausal gonadoptropin for infertility should understand which of the following regarding this med? A. she should take the med PO, once a day before breakfast B. this med stimulates the pituitary gland to produce FSH and the LH C. she must report for ultrasound testing as scheduled to monitor follicular development D. Progesterone should be administered after 7 doses of the gonadotropins have been taken

C. she must report for ultrasound testing as scheduled to monitor follicular development

With regard to nutritional needs during lactation, a maternity nurse should be aware that: The mother's intake of vitamin C, zinc, and protein now can be lower than during pregnancy. Caffeine consumed by the mother accumulates in the infant, who therefore may be unusually active and wakeful. Critical iron and folic acid levels must be maintained. Lactating women can go back to their prepregnant calorie intake.

Caffeine consumed by the mother accumulates in the infant, who therefore may be unusually active and wakeful. A lactating woman needs to avoid consuming too much caffeine. Vitamin C, zinc, and protein levels need to be moderately higher during lactation than during pregnancy. The recommendations for iron and folic acid are somewhat lower during lactation. Lactating women should consume about 500 kcal more than their prepregnancy intake, at least 1800 kcal daily overall.

Which assessments are performed during the initial prenatal visit? Select all that apply. Glucose tolerance test Calculation of due date Prenatal lab panel with blood draw Identification of potential risk factors Evaluation of the condition of the reproductive organs

Calculation of due date The menstrual history is taken and used to determine the patient's estimated date of delivery based on 40 weeks' gestation. Correct Prenatal lab panel with blood draw Prenatal lab panels, including complete blood count (CBC), rapid plasma regain (RPR), hemoglobin and hematocrit (H&H), human immunodeficiency virus (HIV), hepatitis B (Hep B), and Rubella, are drawn at the initial prenatal visit. Correct Identification of potential risk factors Risk factors are determined from a thorough history conducted at the initial prenatal visit. Correct Evaluation of the condition of the reproductive organs A bimanual pelvic exam is performed on initial visit to assess the internal genitalia including the uterus and ovaries.

What health risk is associated with dehydration in pregnant women?

Can lead to preterm labor.

What infection is the vagina more susceptible to during pregnancy?

Candida albicans

Viability

Capable (capability) of living outside the uterus; applied to a fetus that has reached a certain stage of development, usually 22 menstrual weeks (20 weeks of gestation); however, there are no clear limits of gestational age or weight.

viability

Capacity to live outside the uterus at about 22 to 25 weeks of gestation

What is the first organ system to function in the developing human?

Cardiovascular System

What are spermatocytes?

Cells that undergo meiosis

Chadwick sign

Cervix and vagina are violet bluish in color

Supine hypotensive syndrome

Change in blood pressure as a result of compression of abdominal blood vessels and decrease in cardiac output when a pregnant woman lives on her back

When caring for a woman who is scheduled for a hysterosalpingogram, the nurse should do what

Change the woman's position if she complains of shoulder pain following the procedure

Probable signs of pregnancy

Changes observed by an examiner Goodell sign Chadwick sign Hegar sign Positive result of serum pregnancy test Positive result of urine pregnancy test Braxton Hicks contractions Ballottement uterine enlargement

What is ultrasonography used for in determining male infertility?

Check scrotum for abnormalities - varicocele, spermatic cord // transrectal ultrasound can be used to evaluate the ejaculatory ducts, seminal vesicles, and vas deferens for obstruction.

chloasma

Cheeks, nose, and four head blotchy; brownish hyperpigmentation

Which is considered a presumptive sign of pregnancy? Goodell's sign Chloasma Positive urine pregnancy test Visualization of the fetus by ultrasound

Chloasma Skin hyperpigmentation is considered a presumptive sign of pregnancy.

During an examination of a pregnant woman, the nurse notes that the woman's cervix is soft on its tip. The nurse documents this finding as: A. Friability B. Goodell sign C. Chadwick sign D. Hegar sign

Choice B is correct; friability refers to cervical fra- gility resulting in slight bleeding when the cervix is scraped or touched; Chadwick sign refers to a deep bluish cervix and vagina as a result of increased cir- culation; Hegar sign refers to softening and compressibility of the lower uterine segment." B. Goodell sign

An essential component of prenatal health assessment of pregnant women is the determination of vital signs. An expected change in vital sign findings as a result of pregnancy is: A. Increase in systolic BP by 30 mm Hg or more after assuming a supine position B. Increase in diastolic BP by 5 to 10 mm Hg beginning in the first trimester C. Increased awareness of the need to breathe as pregnancy progresses D. Gradual decrease in baseline pulse rate of approximately 20 beats/minute

Choice C is correct; although little change occurs in respiratory rate, breathing becomes more tho- racic in nature with the upward displacement of the diaphragm; women normally experience a greater awareness of breathing and may even complain of dyspnea and an increased awareness to breathe at rest as pregnancy progresses; supine hypotension syndrome occurs as a result of vena cava and aorta compression by the uterus when the woman is in a supine position; baseline pulse rate increases by 10 to 15 beats/minute; systolic and diastolic pressure decreases slightly (or may stay the same; it should not increase) beginning in the second trimester, re- turning to prepregnancy levels in the third trimester. C. Increased awareness of the need to breathe as pregnancy progresses

A woman exhibits understanding of the instructions for performing a home pregnancy test to maximize accuracy if she: A. Uses urine collected at the end of the day, just before going to bed B. Avoids using Tylenol or aspirin for a headache for about 1 week prior to performing the test C. Performs the test on the day she expects her menstrual period to occur D. Records the day of her last menstrual period and her usual cycle length

Choice D is correct; recording cycle information as- sists with accuracy of diagnosis; choice C reflects the most common error of performing this test too soon and she will need to repeat the test in 1 week if the result is negative; first-voided morning specimens should be used because they are the most concen- trated and are apt to have the largest amount of hCG; anticonvulsants, tranquilizers, diuretics, and pro- methazine can result in inaccurate results. D. Records the day of her last menstrual period and her usual cycle length"

A pregnant woman with four children reports the following obstetric history: a stillbirth at 32 weeks of gestation, triplets (two sons and a daughter) born via cesarean section at 30 weeks of gestation, a spontaneous abortion at 8 weeks of gestation, and a daughter born vaginally at 39 weeks of gestation. What accurately expresses this woman's current obstetric history, using the 5-digit system? A. 5-1-4-1-4 B. 4-1-3-1-4 C. 5-2-2-0-3 D. 5-1-2-1-4

Choice D is correct; use 5-digit system GTPAL: gravida (total number of pregnancies, including the present one, is 5), para (term birth of daughter at 39 weeks 5 1, preterm stillbirth at 32 weeks and triplets at 30 weeks 5 2, spontaneous abortion at 8 weeks 5 1, total number of living children 5 4). D. 5-1-2-1-4

When assessing pregnant women during routine prenatal checkups, which findings would be categorized as probable signs of pregnancy? (Circle all that apply.) A. Human chorionic gonadotropin in the urine B. Breast tenderness C. Morning sickness D. Fetal heart sounds E. Ballottement F. Hegar sign

Choices A, E, and F are correct; hCG indicates a positive pregnancy test and is a probable sign along with ballottement and Hegar sign; breast tenderness and morning sickness are presumptive signs; fetal heart sounds are a positive sign of pregnancy. A. Human chorionic gonadotropin in the urine E. Ballottement F. Hegar sign

Vitamin C Food Sources:

Citrus Fruits, Strawberries, melons, broccoli, tomatoes, peppers, raw dark green leafy vegetables

Assisted reproductive technology

Collective term used to designate a variety of alternative measures to help infertile couples achieve the goal of a successful pregnancy and live birth, involves fertility treatments in which both eggs and sperm are handled

Zinc Role in relation to Pregnancy and Lactation:

Component of numerous enzyme systems, possibly important in preventing congenital malformations

Neurological changes during pregnancy

Compression of nerves may cause pain and/or sensory changes; carpal tunnel syndrome from edema involving the peripheral nerves in the last trimester; pain in the hand that radiates to the elbow; abnormal sensation such as burning or tingling; acroesthesia: numbness and tingling of the hands caused by the stoop-shouldered stance; tension headache; "light-headedness;" hypocalcemia can cause neuromuscular problems such as muscle cramps or tetany

In order to reassure and educate pregnant clients about changes in their blood pressure, maternity nurses should be aware that: A blood pressure cuff that is too small produces a reading that is too low; a cuff that is too large produces a reading that is too high. Shifting the client's position and changing from arm to arm for different measurements produces the most accurate composite blood pressure reading at each visit. The systolic blood pressure increases slightly as pregnancy advances; the diastolic pressure remains constant. Compression of the iliac veins and inferior vena cava by the uterus contributes to hemorrhoids in the later stage of term pregnancy.

Compression of the iliac veins and inferior vena cava by the uterus contributes to hemorrhoids in the later stage of term pregnancy. In addition to hemorrhoids, compression of the iliac veins and inferior vena cava by the uterus also leads to varicose veins in the legs and vulva. The tightness of a cuff that is too small produces a reading that is too high; similarly, the looseness of a cuff that is too large results in a reading that is too low. Because maternal positioning affects readings, each blood pressure measurement should be obtained in the same arm and with the woman in the same position. The systolic blood pressure generally remains constant but may decline slightly as pregnancy advances. The diastolic blood pressure first drops and then gradually increases.

Supine hypotensive syndrome

Compression of vena cava while laying flat on back → decrease in systolic BP → feeling faint

In which way do monozygotic twins differ from dizygotic twins? Conception Placenta fusion Fetal circulation Maternal circulation

Conception Monozygotic twins are conceived by one ova and one sperm, whereas dizygotic twins are conceived by two sperm and two ova.

When counseling a client about getting enough iron in her diet, the maternity nurse should tell her that: Milk, coffee, and tea aid iron absorption if consumed at the same time as iron. Iron absorption is inhibited by a diet rich in vitamin C. Iron supplements are permissible for children in small doses. Constipation is common with iron supplements.

Constipation is common with iron supplements. Constipation can be a problem with iron supplements. Milk, coffee, and tea actually inhibit iron absorption when consumed at the same time as iron. Vitamin C promotes iron absorption. Children who ingest iron can get very sick and even die.

Diploid cells

Contain the complete set of chromosomes necessary

Why is potassium important for pregnant women?

Counteracts sodium. Maintains blood pressure. most likely to be lack in pregnant diet - low fat meat, dairy,

Colostrum

Creamy white to yellow wish to orange pre-milk fluid expressed from nipples

Colostrum

Creamy white-to-yellowish-to-orange pre-milk fluid may be expressed as early as 16 weeks

Match the contraception method with the manner in which it can prevent pregnancy.

Creates a physical barrier that prevents the sperm from reaching the egg - Male condom Creates a spermicidal environment through a sterile inflammatory response - Intrauterine device (IUD) Delays or inhibits ovulation - Emergency contraception (EC) Prevents sperm from entering semen during ejaculation - Vasectomy

Where do metabolic waste products of the fetus go?

Cross the placental membrane from the fetal blood in the maternal blood

"Proscriptions; taboos"

Cultural practices that tell a woman what not to do during pregnancy; they establish _________.

"Prescriptions"

Cultural practices that tell a woman what to do during pregnancy.

A 35-year-old client is being seen for her yearly gynecological examination. She states that she and her partner have been trying to become pregnant for a little over 6 months and that a friend had recently advised her partner to take ginseng to improve the potency of his sperm. The woman states that they have decided to take their friend's advice. On which of the following information should the nurse base his or her reply? A. Based on their history, the client and her partner have made the appropriate decision regarding their fertility. B. Ginseng can cause permanent chromosomal mutations and should be stopped immediately. C. It is unnecessary to become concerned about this woman's fertility because she has tried to become pregnant for only a few months. D. Although ginseng may be helpful, it would be prudent to encourage the woman to seek fertility counseling.

D Because fertility drops as a woman ages, it is advisable to encourage the couple to use conventional therapies in conjunction with the complementary therapy to maximize their potential of becoming pregnant.

Infertility increases a client's risk of which of the following diseases? A. Diabetes mellitus B. Nystagmus C. Cholecystitis D. Ovarian cancer

D Infertility has been shown to increase a woman's risk of developing ovarian cancer

An Orthodox Jewish couple is seeking infertility counseling. The woman states that her menstrual cycle is 21 days long. After testing, no physical explanation is found for the infertility. Which of the following may explain why the woman has been unable to conceive? A. Her kosher diet is lacking the essential nutrients needed for achieving optimal reproductive health. B. The positions allowed Orthodox Jewish couples during intercourse hinder the process of fertilization C. Orthodox Jewish couples are known to have a high rate of infertility because of inborn genetic diseases D. Orthodox Jewish couples refrain from intercourse during menses and for seven days after it ends

D Jewish law does prohibit intercourse during the menses and for 7 days following menses. The woman then goes through a cleansing bath called a mikvah before she and her husband may have intercourse. With such a short cycle, she is ovulating during the time frame in which intercourse is restricted.

A client is to undergo a postcoital test for infertility. The nurse should include which of the following statements in the client's preprocedure counseling? A. "You will have the test the day after your menstruation ends." B. "You will have a dye put into your vein that will show up on x-ray." C. "You should refrain from having intercourse for the four days immediately before the test." D. "You should experience the same sensations you feel when your doctor does your Pap test."

D The client will undergo a speculum examination when cervical mucus will be harvested.

A client is to have a hysterosalpingogram. In this procedure, the physician will be able to determine which of the following? A. Whether or not the ovaries are maturing properly B. If the endometrium is fully vascularized C. If the cervix is incompetent D. Whether or not the fallopian tubes are obstructed

D The primary goal of a hysterosalpingogram is to learn whether or not the fallopian tubes are patent.

A client is hospitalized in the acute phase of severe ovarian hyperstimulation syndrome. The following nursing diagnosis has been identified: Fluid volume excess (extravascular) related to third spacing. Which of the following nursing goals is highest priority in relation to this diagnosis? A. Client's weight will be within normal limits by date of discharge B. Client's skin will show no evidence of breakdown throughout hospitalization C. Client's electrolyte levels will be within normal limits within one day D. Client's lung fields will remain clear throughout hospitalization

D This is the priority nursing goal related to ovarian hyperstimulation syndrome.

A client has been notified that because of fallopian tube obstruction, her best option for becoming pregnant is through in-vitro fertilization. The client asks the nurse about the procedure. Which of the following responses is correct? A. "During the stimulation phase of the procedure, the physician will make sure that only one egg reaches maturation." B. "Preimplantation genetic diagnosis will be performed on your partner's sperm before they are mixed with your eggs." C. "After ovarian stimulation, you will be artificially inseminated with your partner's sperm." D. "Any extra embryos will be preserved for you if you wish to conceive again in the future."

D This response is correct. Since multiple embryos are usually created during the in-vitro process, there are often more embryos created than are implanted. The couple may preserve the embryos.

A client is to have a hysterosalpingogram. Which of the following information should the nurse provide to the client prior to the procedure? A. "The test will be performed through a small incision next to your belly button." B. "You will be on bedrest for a full day following the procedure." C. "An antibiotic fluid will be instilled through a tube in your cervix." D. "You will be asked to move from side to side so that x-ray pictures can be taken."

D This statement is correct. A number of pictures will be taken throughout the procedure. The client, who will be awake, is asked to move into positions for the x-rays.

When caring for a woman who is scheduled for a hysterosalpingogram, the nurse should do which of the following? A. ensure the woman is in the secretory phase of her menstrual cycle B. explain to the woman that the procedure will allow her to conceive C. report any uterine cramping immediately because it may indicate the woman's uterus was perforated during the procedure D. change the woman's position if she complains of shoulder pain following the procedure.

D. change the woman's position if she complains of shoulder pain following the procedure.

Vitamin A Food Sources:

Dark green leafy vegetables, Dark yellow vegetables and fruits, liver, fortified margarine and butter

(Increased/decreased) urine flow rate

Decreased May lead to urinary stasis, which can cause: -Altered clearance test results -Stagnated urine is an excellent medium for growth of microorganisms -Extra nutrients in the urine can allow bacteria to grow more easily - renal pelves and ureters are dilated -> larger volume of urine is held in the pelvis and ureters bladder capacity -> 1500 ml-> urge to void even with small amount

Primary reason for age-associated infertility

Decreased ovarian reserve, or total number and quality of follicles.

A 15-year-old female reports being curious about methods of contraception but is hesitant to further discuss it with the nurse. Which steps should the nurse take to facilitate effective discussion? Select all that apply. Demonstrate proper contraceptive use to the female. Reassure the female that the discussion is confidential. Provide the female with written material about different methods of contraception. Ask the female about what she has learned about contraception from her peers. Inform the female that a discussion about contraception cannot occur without a parent present.

Demonstrate proper contraceptive use to the female. Demonstration can help the female better understand methods of contraception, which in turn may help the female use their chosen method of contraception more effectively. Correct Reassure the female that the discussion is confidential. Adolescents often do not discuss contraception with health care providers because they do not want anyone to know they are sexually active. Reassurance of confidentiality can facilitate better communication between the nurse and the female. Correct Provide the female with written material about different methods of contraception. Providing written material can help the female by giving an accurate reference to take home. This can help the female better understand contraception and use it more effectively. Correct Ask the female about what she has learned about contraception from her peers. Asking the adolescent female what she has learned from her peers about contraception is an important step in facilitating education. Adolescents frequently have erroneous beliefs about contraception, and determining where the female may lack accurate knowledge helps the nurse effectively educate the female about effective contraceptive use.

A woman reports discontinuing the use of a diaphragm because of uncertainty about its proper use. Which steps can a nurse take to help the woman better understand this method of contraception? Select all that apply. Tell the woman she will get better at it with practice. Demonstrate proper usage with a sample diaphragm. Ask the woman how she normally uses the diaphragm. Ask the woman what led her to choose the diaphragm initially. Suggest the woman switch to a less complicated method of contraception.

Demonstrate proper usage with a sample diaphragm. Demonstrating is a helpful way to instruct the woman on proper use in a clear and tangible way. This will facilitate learning and help the woman better understand the contraceptive method. Ask the woman how she normally uses the diaphragm. Assessing the woman's current understanding of the contraceptive method is an effective way to determine whether further education is needed. Ask the woman what led her to choose the diaphragm initially. Assessing the woman's choice of contraception helps the nurse better understand the woman's contraceptive needs. Using this information, the nurse can help the woman better understand her method of contraception.

Term

Designation given to a pregnancy from the beginning of week 38 of just station to the end of week 42 of gestation

post term

Designation given to a pregnancy that goes beyond 42 weeks of gestation

Preterm

Designation given to a pregnancy that has reached 20 weeks of gestation but ends before completion of 37 weeks of gestation

Which physiological occurrences are responsible for, or related to, the development of dizygotic twins? Select all that apply. Development of two zygotes Frequency of sexual intercourse Two sperm fertilizing two different ova Release of two ova per menstrual cycle One ovum released during menstrual cycle

Development of two zygotes Two separate zygotes result in dizygotic twins. Two sperm fertilizing two different ova Two ova fertilized by two different sperm result in dizygotic twins. Correct Release of two ova per menstrual cycle The release of more than one ovum per cycle increases chances of dizygotic twins.

Secondary infertility

Difficulty conceiving after having had a pregnancy regardless of outcome.

Primary infertility

Difficulty conceiving when there has never been a pregnancy

Abnormally high hCG levels may indicate

Downs syndrome, multiples

Which behaviors could negatively affect the pregnant woman's ability to psychologically adapt to pregnancy? Select all that apply. Taking prenatal vitamins daily Drinking alcohol during pregnancy Drinking eight glasses of water daily Taking illicit drugs during pregnancy Taking unprescribed prescription pain medication

Drinking alcohol during pregnancy Drinking alcohol during pregnancy can impair judgement, and it can decrease the woman's ability to accept the pregnancy and to transition into the role of motherhood. Taking illicit drugs during pregnancy Taking illicit drugs during pregnancy can impair judgement, and it can decrease the woman's ability to accept the pregnancy and to transition into the role of motherhood. Correct Taking unprescribed prescription pain medication Taking unprescribed prescription pain medication can impair judgement, and it can decrease the woman's ability to accept the pregnancy and to transition into the role of motherhood.

Place the structures in order of which fetal blood flows from the inferior vena cava.

Ductus venosus Right atrium Foramen ovale Left atrium Left ventricle Aorta This describes the fetal circulation circuit process. Oxygenated blood from the placenta enters the fetal body through the umbilical vein. About half the oxygenated venous blood goes through the liver during early pregnancy and the rest bypasses the liver and enters the inferior vena cava through the first shunt, the ductus venosus. The blood then enters the right atrium. Most of the blood passes directly into the left atrium through the second shunt, the foramen ovale, where it mixes with the small amount of blood returning from the lungs. Blood travels from the left atrium to the left ventricle and is then pumped from the left ventricle into the aorta to nourish the body. A small amount of blood from the right ventricle is circulated to the lungs to nourish the lung tissue. The rest of the blood from the right ventricle joins oxygenated blood in the aorta through the third shunt, the ductus arteriosus.

Clamping the umbilical cord triggers which physiological change in fetal circulation? First breath Foramen ovale closure Ductus venosus constriction Ductus arteriosus constriction

Ductus venosus constriction The ductus venosus constricts when blood flow from the umbilical cord stops, as would occur once the cord is clamped.

End of 3rd week

During 4th and 5th week

When does Oogenesis begin?

During fetal life in the female all cells that may undergo meiosis in woman's life ate contained in her ovaries at birth

Why is sodium important for pregnant women?

During pregnancy body water expands and sodium maintains body water balance. Slight increase of sodium is needed in diet sodium rstriction stress adrenal gland and kidney salt amount should be 1.5 g daily

When does the nervous system originate?

During the 3rd WEEK after fertilizaiton

When can fetal respiratory movements be seen on an ultrasound exam?

Early as WEEK 11

Changes that occur in the breasts

Early weeks: • Fullness • Heightened sensitivity • Tingling • Heaviness Nipples become more pigmented and erectile Montgomery tubercles: hypertrophy of sebaceous glands embedded in the primary areolae Keep nipples lubricated Intertwining blue network of veins beneath the skin Breast enlargement (2nd and 3rd trimesters) from growth of mammary glands Lactation is inhibited until a decrease in estrogen level occurs after birth

A pregnant woman experiencing nausea and vomiting should: Drink a glass of water with a fat-free carbohydrate before getting out of bed in the morning. Eat small, frequent meals (every 2 to 3 hours). Increase her intake of high-fat foods to keep the stomach full and coated. Limit fluid intake throughout the day.

Eat small, frequent meals (every 2 to 3 hours). Eating small, frequent meals is a correct suggestion for a pregnant woman experiencing nausea and vomiting. She should avoid consuming fluids early in the day or when nauseated, but should compensate by drinking fluids at other times. She should also reduce her intake of fried foods and other fatty foods.

How can the pregnant woman relieve symptoms of nausea and vomiting that occur in the first trimester upon awakening in the morning? Drinking a cup of coffee in the morning Taking prenatal vitamins in the morning Consuming lots of fluids with solid foods Eating a dry piece of toast in the morning

Eating a dry piece of toast in the morning Eating dry carbohydrates before rising in the morning can help absorb gastric acid and therefore decrease nausea and vomiting

Which findings could be considered to be a barrier to a pregnant woman seeking prenatal care? Select all that apply. Patient would prefer to be cared for by a midwife instead of a physician. Economic cost of health care. Patient's cultural beliefs do not include prenatal care as being valued. Patient speaks several languages. Patient had a bad experience the last time she went to a doctor for care.

Economic cost of healthcare, Patient's cultural beliefs do not include prenatal care as being valued, patient had a bad experience the last time she went to a doctor for care. Economic factors can delay the onset of health care treatment. A patient's cultural beliefs and values may be a barrier to seeking prenatal care if her culture does not perceive any inherent value in prenatal care. If the patient had a bad prior experience with a health care provider, it may be a barrier to seeking future care. The fact that this patient is multilingual does not necessarily represent a barrier to seeking prenatal care. Although the patient may prefer to be cared for by a midwife, this fact cannot be considered a barrier to seeking prenatal care because it demonstrates a patient's choice.

Where does the nervous system originate form?

Ectoderm

Abnormally slow increase or low hCG levels may indicate

Ectopic pregnancy, impending miscarriage

Gestational carrier

Embryos from one couple are transferred into the uterus of another woman

Nuchal cord

Encircling of fetal neck by one or more loops of umbilical cord.

When does the tubular heart begin to beat?

End of 3rd week

Examination of the lining of the uterus to assess its response to progesterone and to detect secretory changes and receptivity to implantation. It is not done if pregnancy is possible

Endometrial biopsy

Which fertility exam is not done if there is a chance of pregnancy?

Endometrial biopsy

What causes implantation "bleeding"?

Endometrial blood vessels erode (dissinegrate)

Montgomery tubercles

Enlarged sebaceous glands in areola on both breasts

Over-the-counter (OTC) pregnancy tests usually rely on which technology to test for human chorionic gonadotropin (hCG)? Radioimmunoassay Radioreceptor assay Latex agglutination test Enzyme-linked immunosorbent assay (ELISA)

Enzyme-linked immunosorbent assay (ELISA) OTC pregnancy tests use ELISA for its one-step, accurate results. Radioimmunoassays test for the subunit of hCG in serum or urine samples and must be performed in the laboratory. The radioreceptor assay is a serum test that measures the ability of a blood sample to inhibit the binding of hCG to receptors. The latex agglutination test in no way determines pregnancy. Rather it is done to detect specific antigens and antibodies.

Which pregnancy discomfort requires immediate follow-up with a health care provider? Backache Constipation Hemorrhoids Epigastric pain

Epigastric pain Epigastric pain can indicate elevated blood pressure/preeclampsia.

Vitamin A Role in relation to Pregnancy and Lactation

Essential for Cell Development, Tooth Bud Formation, and Bone Growth

What is the major estrogen secreted by the placenta? ovaries>

Estriol; estradiol

Skin changes during pregnancy are related to an increase in which hormone? Estrogen Testosterone Progesterone Follicle-stimulating hormone (FSH)

Estrogen Increases in estrogen hormone levels during pregnancy are associated with skin changes.

A nurse midwife has advised a 39-week gestation gravid to take evening primrose oil 2500 mg daily as a complementary therapy. This suggestion was made because evening primrose has been shown to perform which of the following actions? 1. Relieve back strain. 2. Improve development of colostrum. 3. Ripen the cervix. 4. Reduce the incidence of hemorrhoids.

Evening primrose converts to a prostaglandin substance in the body. Prostaglandins are responsible for readying the cervix for dilation.

Hysterosalpingography

Examination of the uterine cavity and tubes using radiopaque contrast material instilled through the cervix. It is often used to determine tubal patency and to release a blockage if present

Which statements explain how exchange of nutrients and waste products can occur between maternal and fetal blood without a physical mixing of that blood? Select all that apply. Exchange can occur because of the presence of a fetal artery. Exchange can occur because of the presence of the two umbilical arteries. Exchange can occur because of the intervillous spaces of the placenta. Exchange can occur because of the presence of the umbilical vein and fetal capillaries blood flow. Exchange can occur because fetal capillaries in the chorionic villi do not come into direct contact with the mother's blood.

Exchange can occur because of the intervillous spaces of the placenta. Uteroplacental circulation starts with the maternal blood flow into the intervillous space through decidual spiral arteries. Exchange of oxygen and nutrients take place here, preventing blood mix from occurring. Exchange can occur because fetal capillaries in the chorionic villi do not come into direct contact with the mother's blood. The membranes of each chorionic villus are separated from direct contact with the mother's blood, but are close enough to allow for nutrient and waste exchange.

Water Role in relation to Pregnancy and Lactation:

Expansion of fluid volume, excretion of wastes, and milk secretion

Which actions by the nurse are appropriate when speaking with a patient who needs fetal diagnostic testing? Select all that apply. Remind the patient that an abnormal result almost certainly indicates a fetal abnormality. Explain how repeated testing benefits the patient and fetus even though it may seem tedious. Provide the patient with a clear explanation of why the test is indicated and what abnormalities it can detect. Help the patient understand that there is a baseline risk for fetal abnormalities even when all results are normal. Being honest with the patient if the nurse disagrees with the patient's decision to continue or terminate the pregnancy.

Explain how repeated testing benefits the patient and fetus even though it may seem tedious. Explaining the benefit of repeated testing helps the patient understand how the tests benefit both the patient and the infant. This helps to address potential frustration about repeated testing and is an appropriate action for the nurse to take. Provide the patient with a clear explanation of why the test is indicated and what abnormalities it can detect. Providing the patient with a clear explanation of why the test is indicated and what abnormalities it can detect helps the patient understand the purpose and potential outcomes of the test. Providing the patient with this information is an appropriate action by the nurse. Help the patient understand that there is a baseline risk for fetal abnormalities even when all results are normal. Helping the patient to understand that a baseline risk exists even if all test results are normal helps the patient understand that normal test results do not guarantee the birth of a perfect infant. This helps the patient set realistic goals and is an appropriate action for the nurse to take.

A patient reports frustration at the health care provider's decision not to perform all of the diagnostic tests she wishes to have. Which patient education is appropriate? Select all that apply. Tell the patient that her concerns will be reported to the provider. Explain to the patient that some abnormalities can be detected by more than one test. Inform the patient that she has the right to seek treatment elsewhere if she is not satisfied. Explain to the patient that even if all tests were normal, it would not rule out every abnormality. Inform the patient that each procedure carries risks to both patient and infant, and that risk increases when multiple procedures are performed.

Explain to the patient that some abnormalities can be detected by more than one test. Explaining the overlap between diagnostic test capabilities helps the patient better understand the purpose and efficacy of fetal diagnostic testing. Explain to the patient that even if all tests were normal, it would not rule out every abnormality. Informing the patient of the baseline risk that exists regardless of normal test results helps the patient better understand the purpose of fetal diagnostic testing. Correct Inform the patient that each procedure carries risks to both patient and infant, and that risk increases when multiple procedures are performed. Explaining the risks of diagnostic procedures helps the patient better understand the risks and advantages of fetal diagnostic testing.

A pregnant woman demonstrates understanding of the nurse's instructions regarding relief of leg cramps if she: Wiggles and points her toes during the cramp. Applies cold compresses to the affected leg. Extends her leg and dorsiflexes her foot during the cramp. Avoids weight bearing on the affected leg during the cramp.

Extends her leg and dorsiflexes her foot during the cramp. Extending the leg and dorsiflexing the foot are the appropriate relief measure for a leg cramp. Pointing the toes can aggravate rather than relieve the cramp. Application of heat is recommended. Bearing weight on the affected leg can help relieve the leg cramp, so it should not be avoided.

What happens to the external vaginal structures?

External structures are enlarged (increased vasculature, hypertrophy, and deposition of fat)

A pregnant patient is experiencing some integumentary changes and is concerned that they may represent abnormal findings. The nurse provides information to the patient that the following findings would be considered "normal abnormal" findings during pregnancy so that she should not be alarmed. Select all that apply. Facial edema Melasma Linea nigra Superficial thrombophlebitis Vascular spiders Allodynia

Facial edema is a concern because it can represent toxemia of pregnancy. Superficial thrombophlebitis is a concern because it can represent a risk factor for development of a DVT during pregnancy. The presentation of allodynia (pain upon normal touch) is considered to be a significant finding and requires additional investigation. Melasma (also known as the mask of pregnancy or chloasma), linea nigra (a hyperpigmentation line extending from the fundus to the symphysis pubis), and the presence of vascular spiders are all considered to be normal abnormal findings in pregnancy.

What do the ovaries contain at birth?

Females lifetime supple of ova

Calcium Role in relation to Pregnancy and Lactation:

Fetal Skeleton and Tooth Formation; Maintenance of Maternal Bone and Tooth Mineralization

Ballottement

Fetal head rebounds with general, upward tapping through the vagina

What does cold solutions cause?

Fetal hiccups

A nurse teaches a pregnant woman about the presumptive, probable, and positive signs of pregnancy. The woman demonstrates understanding of the nurse's instructions if she states that a positive sign of pregnancy is: A positive pregnancy test result. Fetal movement palpated by the nurse-midwife. Braxton Hicks contractions. Quickening.

Fetal movement palpated by the nurse-midwife. Positive signs of pregnancy are those that are attributed to the presence of a fetus, such as hearing the fetal heartbeat and palpating fetal movement. A positive pregnancy test result and Braxton Hicks contractions are probable signs of pregnancy. Quickening is a presumptive sign of pregnancy.

Quickening

Fetal movements first fell by the pregnant woman from 14 to 18 weeks of just station

What is Oligohydramnios associated with?

Fetal renal abnormalities

What is viability defined by?

Fetal weight and pregnancy duration

What happens if the fetus doesn't produce thyroxine?

Fetus will be born with congenital hypothyroidism

A pregnant woman has been diagnosed with oligohydramnios. Which presentation would the nurse suspect to find on physical examination? Fetus is in a breech position FHR baseline is within normal range Fetus with possible renal problems Increased fundal height

Fetus with possible renal problems Oligohydramnios reflects a decrease in the amount of amniotic fluid and is associated with renal abnormalities in the fetus and compromised fetal well-being. The position of the fetus is due to gestational age and the maternal uterine environment. FHR may be within normal range because it is affected by gestational age and fetal well-being. An increase in fundal height would be associated with polyhydramnios and/or gestational age assessment.

Although remarkable developments have occurred in reproductive medicine, assisted reproductive therapies are associated with a number of legal and ethical issues. Nurses can provide accurate information about the risks and benefits of treatment alternatives so couples can make informed decisions about their choice of treatment. Which issue would not need to be addressed by an infertile couple before treatment? Risk of multiple gestation Whether or how to disclose the facts of conception to offspring Freezing embryos for later use Financial ability to cover the cost of treatment

Financial ability to cover the cost of treatment Although the method of payment is important, obtaining information about ability to pay is not the nurses' responsibility; it is also of note that 14 states have mandated some form of insurance to assist couples with coverage for infertility. Multiple gestation is indeed a risk of treatment of which the couple needs to be aware. To minimize this risk, generally only three or fewer embryos are transferred. The couple should be informed that there may be a need for multifetal reduction. Nurses can provide anticipatory guidance on disclosure to offspring. Depending on the therapy chosen, there may be a need for donor oocytes, sperm, embryos, or a surrogate mother. Couples who have excess embryos frozen for later transfer must be fully informed before consenting to the procedure. A decision must be made regarding the disposal of embryos in the event of death or divorce or if the couple no longer wants the embryos at a future time.

What are chorionic villi?

Finger-like projections that develop and extend in the blood-filled places of the endometrium

Calorie Recommendation during Lactation:

First 6 months, non pregnant needs plus +330 kcal Second 6 months, non pregnant needs plus +400 kcal

meconium

First stools of infant: viscid, sticky; dark greenish brown, almost black; sterile; odorless. .

The nurse should encourage the pregnant woman to receive which vaccinations during pregnancy? Select all that apply. Flu Polio MMR TDAP Hepatitis B

Flu Flu vaccinations are advised during pregnancy, as the flu often has more serious manifestations in pregnant woman and can result in preterm birth. TDAP TDAP vaccinations are encouraged during pregnancy and can provide protection for the baby against pertussis.

Why should the pregnant patient be encouraged to get adequate amounts of folic acid during pregnancy? Folic acid prevents birth defects. Folic acid prevents preterm labor. Folic acid increases iron absorption. Folic acid helps maintain the pregnancy.

Folic acid prevents birth defects. It is important for a pregnant patient to take adequate folic acid to prevent neural tube defects specific to the spinal cord and the brain.

Which shunt is responsible for the delivery of blood from the right to left atrium? Aorta Foramen ovale Ductus venosus Ductus arteriosus

Foramen ovale From the ductus venosus, blood flows into the vena cava and enters the right atrium; blood flows from the right to left atrium through the foramen ovale.

Match the fetal shunt with the physiological event associated with its closing/constriction after birth.

Foramen ovale Increased oxygen to the lungs Ductus venosus Umbilical cord circulation stops Ductus arteriosus Increase in arterial oxygen

Preimplantation genetic diagnosis

Form of early testing designed to eliminate embryos with serious genetic defects before implantation through one of the ARTs and to avoid future termination of pregnancy for genetic reasons

Vitamin D Food Sources:

Fortified Milk and Breakfast Cereals; Salmon, Tuna, and other oily fish; butter, liver

Which is another term for dizygotic twins? Paternal Maternal Identical Fraternal

Fraternal Dizygotic twins have different genetic makeup and are frequently called fraternal twins by laypeople.

How long does the stage of fetus last?

From 9 weeks until pregnancy

Amniotic fluid functions

Functions: maintain constant body temperature source of oral fluid, repository for waste, maintenance of fluid and electrolyte homeostasis allows freedom of movement cushions the fetus acts as a barrier to infection and allows lung development

lightning

Fundal height decreased and fetal head descended into the pelvis

General apperance

G- Alert: Responsive P-Listless, apathetic, cachectic

eyes

G- Bright, clear, shiny, no sores at corner of eyelids, moist and healthy pink conjunctivae, prominent blood vessels, no fatique circles beneath eyes. P- Eye mebranes pale, redness of membrane, dryness, signs of infecton.

General vitality

G- Endurance, energy, sleeps well P- Easily fatigued, no energy, falls asleep early, tired and apathetic

Posture

G- Erect posture; straight arms and legs P- Sagging shoulders, sunken chest, humped back

nails

G- Firm, pink P- Spoon shape, brittleness, ridges

Gastrointestinal function

G- Good appetite and digestion,, normal regular elimination, no palpable organs or masses. P- Anorexia, indegestion, constipation, diarrehea, liver or spleen enlargement.

Nervous system control

G- Good attention span, not irritable or restless, normal reflexes, psychological stability P- Inattention, irrability, confusion, paresthesia, weakness and tenderness of muscles, decrease or loss of ankle and knee reflexes.

gums

G- Good pink color, healthy and red, no swelling or bleeding. P- Spongy gums that bleed easily, marginal redness, inflammation, receding.

tongue

G- Good pink or deep reddish color, no swelling, smooth, presence of surface papillae, lack of lesions. P- Swelling, scarlet and raw magenta, hyperemic and hypertrophic papillae, atrophic papillae.

teeth

G- No cavaties, no pain, bright, straight, no crowding, well shaped jaw, clean with no discoloration. P- Unfilled caries, missing teeth, worn surfaces, mottled, malpositioned.

neck/glands

G- No enlargment P- Thyroid or lymph node enlargment

skeleton

G- No malformations P- Bowlegs, knock- knees, chest deformity at diaphram, prominent scapulae and ribs.

legs/feet

G- No tenderness, weakness or swelling, good color P- Edema, tender calf, tingling weakness.

Cardiovascular function

G- Normal heart rate and rhythm, lack of murmurs, normal blood pressure for age. P- Rapid heart above 100 beats/min enlarged, abnormal rhythm, evevated blood pressure.

Mouth, oral membranes

G- Reddish-pink mucous membranes in oral cavity . P- Swollen, boggy oral mucous membranes.

hair

G- Shiny, lustrous, firm, not easily plucked, healthy scalp P- Stringy, dull, brittle, dry, thin, and sparse, dipigmented, easily plucked

skin

G- Smooth and slightly moist skin with good color P- Rough, dry, scaly, pale, pigmented, irritated, bruises, subcutaneous fat loss.

lips

G- Smooth, good color, moist, not chapped or swollen. P- Dry, scaly, swollen, redness and swelling, angular lesions at corners of mouth, fissures or scars.

face/neck

G- Uniform color, smooth, pink, healthy appearance, not swollen. P- Greasy, dicolored, scaly, swollen, dark skin over cheeks and under eyes, lumpiness or flakiness of skin around nose and mouth.

Weight

G- Weight normal for height, age and body build P- Obesity 10 % above ideal body weight or underweight

muscles

G- Well-developed, firm, good tone, some fat under skin P-Flaccid, poor tone, impaired ability to walk properly

A woman comes in for her first prenatal visit. The woman tells the nurse that this is her fourth pregnancy. She had a miscarriage with her first pregnancy at five weeks. She had a 39-week viable male with her second pregnancy. She had 35-week twins with her third pregnancy, and they are all living. How would the nurse record her obstetric history using the GTPAL system? G4T2P1A1L3 G4T1P2A1L3 G4T1P1A1L3 G4T1P2A1L2

G4T1P1A1L3 G4T1P1A1L3 is correct because there is one term pregnancy, one preterm pregnancy, one abortion, and three living children according to the woman's OB history.

What is Plyhydramnios associated with?

GI and other malformations

Systems for gravidity and parity

GP system GTPAL (gravity, term, preterm, abortions/miscarriages, living)

Which factors are related to the increased occurrence of dizygotic twins? Select all that apply. Genetics Ethnicity Infertility treatment Random occurrence Timing of conception

Genetics Genetics can play a role in the increased occurrence of dizygotic twins, presumably because of an inherited tendency of women to release more than one ovum per cycle. Ethnicity Women of some heritages or country of origin, for example Africa, are more likely to have dizygotic twins. Infertility treatment Infertility therapy is associated with the increased incidence of dizygotic twin births.

Which condition is suspected by a urine dipstick screen that shows 2+ glucosuria? Dehydration Preeclampsia Preterm labor Gestational diabetes

Gestational diabetes Gestational diabetes would be a concern if the urine dipstick shows 2+ glycosuria.

Which actions would a pregnant woman be expected to take during the third trimester of pregnancy? Focus on self Quit smoking Get home ready for the baby Seek acceptance of fetus and role of mother

Get home ready for the baby Getting the home ready is an emotional response that should take place during the third trimester of pregnancy.

What can a woman do to stop Braxton Hicks contractions?

Get up and walk

What can help with nausea and vomiting?

Ginger, B6, P6 acupressure point

XX=

Girl

Which presumptive sign (felt by woman) or probable sign (observed by the examiner) of pregnancy is not matched with another possible cause(s)? Amenorrhea—stress, endocrine problems Quickening—gas, peristalsis Goodell sign—cervical polyps Chadwick sign—pelvic congestion

Goodell sign—cervical polyps Goodell sign might be the result of pelvic congestion, not polyps. Amenorrhea sometimes can be caused by stress, vigorous exercise, early menopause, or endocrine problems. Quickening can be gas or peristalsis. Chadwick sign might be the result of pelvic congestion.

Calorie Roles in relation to Pregnancy and Lactation:

Growth of fetus and maternal tissues; milk production

haploid cells

Have 1 complete set of chromosomes (half)

pyrosis

Heartburn experienced after supper

During a client's physical examination, the nurse notes that the lower uterine segment is soft on palpation. The nurse would document this finding as: Hegar sign. McDonald sign. Chadwick sign. Goodell sign.

Hegar sign. At approximately 6 weeks of gestation, softening and compressibility of the lower uterine segment occur; this is called the Hegar sign. The Chadwick sign is a blue-violet cervix caused by increased vascularity; it is seen around the fourth week of gestation. Softening of the cervical tip, which may be observed around the sixth week of pregnancy, is called the Goodell sign. (The McDonald's sign indicates a fast-food restaurant.)

physiologic anemia

Hematocrit decreased to 36% and hemoglobin to 11 g/dL

How does the ovum get to the uterine cavity?

High estrogen levels increase the motility of the uterine tubes so their cilia are able to capture the ovum and propel it through the tube toward the uterine cavity

Which finding, if present in both the male and female of a couple, could present an issue with regard to the couple's fertility? Male and female are the same age, 35. Both partners have had children in their past marriage. History of endocrine problems. History of hypertension.

History of endocrine problems. A history of endocrine problems should be investigated further because it may have an effect on the couple's fertility. Age is a relative factor but the recorded age, 35 years, does not in itself represent a significant fertility factor unless there are additional factors. The fact that both the man and the woman have already had children is a favorable sign in fertility. Although hypertension is a relevant clinical finding, it may not have a direct bearing on the couple's fertility.

Under which condition should a woman taking hormone injections use a backup method of contraception? If the injection site is still tender If the injection was given subcutaneously 14 days ago If it has been 15 weeks since the last injection was given If the injection was given three days after the start of the woman's menstrual period

If it has been 15 weeks since the last injection was given Hormone injections prevent ovulation for 14 weeks, and injections should be scheduled every 12 weeks. If 15 weeks have passed since the previous injection, the woman should use a backup method of contraception.

What is the ONLY immunoglobin that crosses the placenta?

IgG

The nurse caring for a pregnant woman knows that her health teaching regarding fetal circulation has been effective when the woman reports that she has been sleeping: In a side-lying position. On her back with a pillow under her knees. With the head of the bed elevated. On her abdomen.

In a side-lying position. Optimal circulation is achieved when the woman is lying at rest on her side. Decreased uterine circulation may lead to intrauterine growth restriction. Previously it was believed that the left lateral position promoted maternal cardiac output, thereby enhancing blood flow to the fetus. However, it is now known that either side-lying position enhances uteroplacental blood flow. If a woman lies on her back with the pressure of the uterus compressing the vena cava, blood return to the right atrium will be diminished. Although having a pillow under the knees is recommended and ideal for later in pregnancy, the woman must still maintain a lateral tilt to the pelvis to avoid compression of the vena cava. Many women will find lying on the abdomen uncomfortable as pregnancy advances.

Ductus arteriosis

In fetal circulation an anatomic shunt between the pulmonary artery and arch of the aorta. It is obliterated after birth by a rising PO2 and a change in intravascular pressures in the presence of normal pulmonary function. It normally becomes a ligament after birth but in some instances remains patent.

Ductus veneosus

In fetal circulation, a blood vessel carrying oxygenated blood between the umbilical vein and the inferior vena cava, bypassing the liver. It is obliterated and becomes a ligament after birth. .

Sexual Activity

In most cultures, sexual activity is not prohibited until the end of pregnancy.

Where does Meiosis occur in the female?

In the ovarian follicles and proceeds an ovum

Where are carbs, proteins, calcium, and iron stored?

In the placenta, read got access to meet fetal needs

Which suggestion about weight gain is not an accurate recommendation? Underweight women should gain 12.5 to 18 kg. Obese women should gain at least 7 kg. Adolescents are encouraged to strive for weight gains at the upper end of the recommended scale. In twin gestations, the weight gain recommended for a single fetus pregnancy should simply be doubled.

In twin gestations, the weight gain recommended for a single fetus pregnancy should simply be doubled. Women bearing twins need to gain more weight (usually 16 to 20 kg) but not necessarily twice as much. Underweight women need to gain the most. Obese women need to gain weight during pregnancy to equal the weight of the products of conception. Adolescents are still growing; therefore, their bodies naturally compete for nutrients with the fetus.

Sterility

Inability to conceive

what defect can occur with poor intake of folic acid

Inadequate amounts of folic acid may cause neural tube defects (brain, spine, spinal cord) . Neural tubes begin to close in the 1st month of gestation usually before female knows she is pregnant.

What is the sequential process of conception?

Includes gamete formation, ovulation, fertilization, & implantation in the uterus

The nurse correctly identifies that which physiological event may lead to a conjoined twin pregnancy? Two ova fertilized by two sperm Incomplete separation of the placenta Single ovum fertilized by two spermatozoa Incomplete separation of the inner cell mass

Incomplete separation of the inner cell mass Incomplete separation of the inner cell mass will cause the embryos to fuse, resulting in a conjoined twin pregnancy that can occur in monozygotic twins.

(Increase/decrease) in clotting factors

Increase in clotting factors → increased tendency to coagulate (clot) •Protective to prevent excessive bleeding •Makes woman more vulnerable to thrombosis, especially after c-section

Change in HR

Increase in pulse between 10-15 extra beats/min

Iodine Role in relation to Pregnancy and Lactation:

Increased Maternal Metabolic Rate

A pregnant woman reports having a lot of nasal stuffiness and congestion. Which pregnancy-associated change is likely responsible for her symptoms? Increased estrogen levels Increased oxytocin levels Increased prolactin levels Increased progesterone levels

Increased estrogen levels Increased estrogen levels are associated with increased vascularity to the mucous membranes of the upper respiratory tract during pregnancy. This increases incidence of nasal stuffiness and congestion.

A patient reports having frequent yeast infections since becoming pregnant. Which physiologic change is likely responsible for this symptom? Increased estrogen levels Increased prolactin levels Decreased progesterone levels Increased glycogen secretion within the vaginal walls

Increased glycogen secretion within the vaginal walls Increased glycogen secretion from the vaginal walls favors the growth of yeast infections during pregnancy.

How does the increase in progesterone affect the GI system?

Increased progesterone → decreased tone and motility of smooth muscles → regurgitation, slower emptying of the stomach, and reverse peristalsis

Cardiovascular system changes occur during pregnancy. Which finding would be considered normal for a woman in her second trimester? Less audible heart sounds (S1, S2) Increased pulse rate Increased blood pressure Decreased red blood cell (RBC) production

Increased pulse rate Between 14 and 20 weeks of gestation, the pulse increases about 10 to 15 beats/min, which persists to term. Splitting of S1 and S2 is more audible. In the first trimester blood pressure usually remains the same as the prepregnancy level, but it gradually decreases up to about 20 weeks of gestation. During the second trimester both the systolic and diastolic pressures decrease by about 5 to 10 mm Hg. Production of RBCs accelerates during pregnancy.

How does sexual interest change during pregnancy?

Increased vascularity → increased sensitivity → more sexual interest

Chadwick sign

Increased vascularity → violet-bluish mucosa and cervix

What causes varicose veins and hemorrhoids in pregnancy?

Increased venous pressure and reduced blood flow to the legs

Change in CO

Increases and then slightly decreases at 40 weeks (required for tissue oxygen demands) increases 30-to 50% by 32 week, declines to 20% increase at 40 weeks increased stroke volume, HR due to increased demand for oxygen

"Positive"

Indicators of pregnancy that are attributed directly to the fetus.

"Presumptive"

Indicators of pregnancy that can be caused by conditions other than gestation and are not reliable for diagnosis; they include subjective symptoms and objective signs."

An expectant couple asks the nurse about intercourse during pregnancy and whether it is safe for the baby. The nurse should tell the couple that: Intercourse should be avoided if any spotting from the vagina occurs afterward. Intercourse is safe until the third trimester. Safer-sex practices should be used once the membranes rupture. Intercourse and orgasm are often contraindicated if a history or signs of preterm labor are present.

Intercourse and orgasm are often contraindicated if a history or signs of preterm labor are present. Uterine contractions that accompany orgasm can stimulate labor and would be problematic if the woman were at risk for or had a history of preterm labor. Some spotting can normally occur as a result of the increased fragility and vascularity of the cervix and vagina during pregnancy. Intercourse can continue as long as the pregnancy is progressing normally. Safer-sex practices are always recommended; rupture of the membranes may require abstaining from intercourse.

What does the Mesoderm develop into?

Into bones & teeth, the muscles, dermis, connective tissue, CV system, spleen, and urogenital system

Where is urine excreted?

Into the amniotic fluid forming a major part of the fluid volume

An infertile woman is about to begin pharmacologic treatment. As part of the regimen, she will take purified follicle-stimulating hormone (FSH) (urofollitropin [Metrodin]). The nurse instructs her that this medication is administered in the form of a/an: Intranasal spray Vaginal suppository Intramuscular injection Tablet

Intramuscular injection Urofollitropin is given by IM injection; the dosage may vary. It cannot be given by the other routes listed

Women with inadequate weight gain during pregnancy are at higher risk of giving birth to an infant with: Spina bifida. Intrauterine growth restriction. Diabetes mellitus. Down syndrome.

Intrauterine growth restriction. Both normal-weight and underweight women with inadequate weight gain have an increased risk of giving birth to an infant with intrauterine growth restriction. Spina bifida is not associated with inadequate maternal weight gain; an adequate amount of folic acid has been shown to reduce the incidence of this condition. Diabetes mellitus is not related to inadequate weight gain. A mother with gestational diabetes is more likely to give birth to a large-for-gestational age infant. Down syndrome is the result of trisomy 21, not inadequate maternal weight gain.

Prepared sperm is placed in the uterus at ovulation

Intrauterine insemination

Vitamin D Role in relation to Pregnancy and Lactation:

Involved in Absorption of Calcium and Phosphorus, Improves Mineralization

Magnesium Role in relation to Pregnancy and Lactation:

Involved in Energy and Protein Metabolism, Tissue Growth, Muscle Action

B6 Role in relation to Pregnancy and Lactation:

Involved in Protein Metabolism

Which minerals and vitamins are usually recommended to supplement a pregnant woman's diet? Fat-soluble vitamins A and D Water-soluble vitamins C and B6 Iron and folate Calcium and zinc

Iron and folate Iron generally should be supplemented, and folic acid supplements often are needed because folate is so important. Fat-soluble vitamins should be supplemented as a medical prescription, as vitamin D might be for lactose-intolerant women. Water-soluble vitamin C sometimes is consumed in excess naturally; vitamin B6 is prescribed only if the woman has a very poor diet. Zinc is sometimes supplemented; most women get enough calcium.

Nurses should be aware that infertility: Is perceived differently by women and men. Has a relatively stable prevalence among the overall population and throughout a woman's potential reproductive years. Is more likely the result of a physical flaw in the woman than in her male partner. Is the same thing as sterility.

Is perceived differently by women and men. Women tend to be more stressed about infertility tests and to place more importance on having children. The prevalence of infertility is stable among the overall population, but it increases with a woman's age, especially after age 40. Of cases with an identifiable cause, about 40% are related to female factors, 40% to male factors, and 20% to both partners. Infertility is a state of requiring a prolonged time to conceive, or subfertility; sterility is the inability to conceive.

Why is cervical mucus important to fertility?

It is more alkaline (pH 7) compared to pH of vagina (4-5) and helps the sperm travel upwards. WBC destroy viable motile sperm Vaginal or cervical infections destroy or reduce the number of motile sperm.

A maternity nurse should be aware of which fact about the amniotic fluid? It serves as a source of oral fluid and as a repository for waste from the fetus. The volume remains about the same throughout the term of a healthy pregnancy. A volume of less than 300 ml is associated with gastrointestinal malformations. A volume of more than 2 L is associated with fetal renal abnormalities.

It serves as a source of oral fluid and as a repository for waste from the fetus. Amniotic fluid also cushions the fetus and helps maintain a constant body temperature. Its volume changes constantly; too little fluid (oligohydramnios) is associated with renal abnormalities, and too much fluid (polyhydramnios) is associated with gastrointestinal and other abnormalities.

Pruitus gravidarum

Itching of the skin that occurs during pregnancy

Where does the amnotic cavity initially derive from?

Its fluid by diffusion from the maternal blood

Which factor may negatively affect a woman's psychological adaptation to pregnancy? Job loss Exercise habits Pregnancy at age 25 Friends with multiple children

Job loss Socioeconomic status has a significant impact on psychological adaptation to pregnancy. Job loss can influence the socioeconomic status of a pregnant woman.

The nurse is explaining the duties of the support person during labor. Which duties should be included in teaching? Select all that apply. Keeping patient calm Keeping track of contractions Interpretation of fetal monitoring Administration of pain medication Ensuring physical and emotional comfort

Keeping patient calm The support person should help with breathing and relaxation techniques during labor to keep the patient calm and focused. Keeping track of contractions The support person should help time contractions during labor. This will help the woman to know when to come to the hospital for labor and birth. Ensuring physical and emotional comfort The support person should help with comfort measures during labor and assist with emotional support of the patient. This will help the woman to have less anxiety and promote relaxation during labor.

mature infant's lungs

L/S ratio 2:1

Menotropins-human Menopausal Gonadotropins (hMG) MOA

LH and FSH in 1:1 ratio direct stimulation of ovarian follicles; given sequently with human chorionic gonadotropin to induce ovulation

During the third trimester of pregnancy, the nurse would expect the woman to have which emotional response to pregnancy? Curiosity Focus on self Uncertain feelings Leaning on others for support

Leaning on others for support Vulnerability and increased dependence on others is the expected emotional response during the third trimester of pregnancy.

What is the most critical alveolar surfactant required for postnatal lung expansion?

Lecithin (l) detected after 21 weeks, increases after 24 weeks

Foods Providing 100 mcg or more per serving of Folate:

Legumes, cooked (1/2 cup) Peas: black-eyed, chickpea (garbanzo) Lentils Vegetables (1/2 cup) Asparagus Spinach, cooked Papaya (1 medium) Wheat Germ (1/4 cup)

Iron Food Sources

Liver, Meats, Whole Grain or enriched breads and cereals, Dark Green Leafy Vegetables, Legumes, Dried Fruits

Zinc Food Sources:

Liver, Shellfish, Meats, Whole Grains, Milk

Foods Providing 200 mcg or more per serving of Folate:

Liver: Lamb, Beef, Veal (100 g [3.5 of]) Rice, enriched, cooked (1 cup) Breakfast Cereals, ready-to-eat, fortified (1 cup)

Foods Providing 500 mcg or more per serving of Folate:

Liver: chicken, turkey, goose (100 g [3.5 of])

Musculoskeletal changes during pregnancy

Lordosis to maintain balance; aching, numbness, and weakness of upper extremities; waddling gait; muscles of the abdomen wall stretch and ultimately lose some tone; diastasis recti abdominis

Which characteristics allow the fetus to thrive, despite a lower than average blood PO2? Select all that apply. Blood entering the placenta has a low PCO2. Low PCO2 causes the fetal hemoglobin to carry more oxygen. Fetal blood pH becomes higher than the maternal blood pH. Fetal hemoglobin can carry more oxygen than adult hemoglobin can. The fetus has a lower oxygen-carrying capacity because of a higher average hemoglobin and hematocrit.

Low PCO2 causes the fetal hemoglobin to carry more oxygen. Hemoglobin can carry more oxygen at a low carbon dioxide partial pressure (PCO2) levels than it can at a high one (Bohr effect). Fetal blood pH becomes higher than the maternal blood pH. Fetal blood becomes more alkaline and the maternal blood becomes more acidic. This difference allows the mother's blood to give up oxygen and the fetal blood to readily combine with oxygen; therefore, fetal blood pH becomes higher than the maternal blood pH. Fetal hemoglobin can carry more oxygen than adult hemoglobin can. Fetal hemoglobin can carry 20% to 30% more oxygen than maternal hemoglobin; this allows the fetus to thrive in a low-oxygen environment.

lordosis

Lumbo sacral curve accentuated

Which developmental finding is accurate with regard to fetal growth? Heart starts beating at 12 weeks. Lungs take shape by 8 weeks. Brain configuration is complete by 8 weeks. Main blood vessels form by 8 weeks.

Main blood vessels form by 8 weeks. The heart starts beating by 4 weeks, the lungs take shape by 12 weeks, and brain configuration is complete by 12 weeks.

What is Progesterones purpose?

Maintains the endometrium, decreases contractility of the uterus, and stimulates maternal metabolism, development of breast alveoli

Gamete

Male and female germ cell, the male germ cell is a sperm and the female germ cell is an ovum (egg)

Countering Misinformation

Many myths and much of the misinformation related to sex and pregnancy are masked by seemingly unrelated issues.

With regard to protein in the diet of pregnant women, nurses should be aware that: Many protein-rich foods are also good sources of calcium, iron, and B vitamins. Many women need to increase their protein intake during pregnancy. As with carbohydrates and fat, no specific recommendations exist for the amount of protein in the diet. High-protein supplements can be used without risk by women on macrobiotic diets.

Many protein-rich foods are also good sources of calcium, iron, and B vitamins. Good protein sources such as meat, milk, eggs, and cheese have a lot of calcium and iron. Most women already eat a high-protein diet and do not need to increase their intake. Protein is sufficiently important that specific servings of meat and dairy are recommended. High-protein supplements are not recommended because they have been associated with an increased incidence of preterm births.

Chloasma

Mask of pregnancy over the cheeks, nose, and forehead; blotchy, brownish hyperpigmentation of the skin -Intensified by the sun -Seen especially in dark-complexioned pregnant women

Iron Role in relation to Pregnancy and Lactation:

Maternal hemoglobin formation, Fetal liver iron storage

What does the Yolk sack aid in transferring?

Maternal nutrients and oxygen

quickening

Maternal perception of fetal movement ("feeling life"); usually occurs between weeks 16 and 20 of gestation, but may be felt earlier by multiparous woman. .

Protein Food Sources

Meats, Eggs, Cheese, Yogurt, Legumes (dry beans and peas, peanuts), nuts and grains

B6 Food Sources:

Meats, Liver, Dark green leafy vegetables, Whole Grains

Amniotic Membranes

Membranes that surround the developing baby and the fluid: Amnion - Inner Chorion - outer

amenorrhea

Menstrual bleeding no longer occurs

B12 Food Sources:

Milk and Milk Products, Eggs, Meats, Liver, Fortified Soy Milk

Calcium Food Sources:

Milk, cheese, yogurt, Sardines or other fish eaten with bones left in, dark green leafy vegetables except spinach or Swiss chard, calcium-set tofu, baked beans, tortillas

number of division

Mitosis: 1 Meiosis: 2

type of reproduction

Mitosis: asexual Meiosis: sexual

What type of cultural concern is the most likely deterrent to many women seeking prenatal care? Religion Modesty Ignorance Belief that physicians are evil

Modesty A concern for modesty is a strong deterrent to many women seeking prenatal care; for some women, exposing body parts, especially to a man, is considered a major violation of modesty. There are other deterrents. Even if the prenatal care described is familiar to a woman, some practices may conflict with the beliefs and practices of a subculture group or religion to which she belongs. For many cultural groups a physician is deemed appropriate only in times of illness. Because pregnancy is considered a normal process and the woman is in a state of health, the services of a physician are considered inappropriate. Many cultural variations are found in prenatal care, so ignorance is not likely to be a deterrent to women seeking prenatal care.

How does MOM get rid of the fetal waste?

Moms kidneys excrete them

In understanding and guiding a woman through her acceptance of pregnancy, a maternity nurse should be aware that: Nonacceptance of the pregnancy very often equates to rejection of the child. Mood swings are most likely the result of worries about finances and a changed lifestyle, as well as profound hormonal changes. Ambivalent feelings during pregnancy are usually seen only in emotionally immature or very young mothers. Conflicts such as not wanting to be pregnant or childrearing and career-related decisions need not be addressed during pregnancy because they will resolve themselves naturally after birth.

Mood swings are most likely the result of worries about finances and a changed lifestyle, as well as profound hormonal changes. Mood swings are natural and are likely to affect every woman to some degree. A woman may dislike being pregnant, refuse to accept it, and still love and accept the child. Ambivalent feelings about pregnancy are normal for mature or immature women, young or older. Conflicts about desire to perform childrearing and career-related concerns, however, need to be resolved; the baby's arrival ends the pregnancy but not all the issues.

The vagina is (more/less) acidic during pregnancy

More (to protect against organisms)

Operculum

Mucous plug that acts as a barrier against bacterial invasion during pregnancy

Multiparous Women

Multiparous women may have never used contraceptives because of personal choice or a lack of knowledge concerning contraceptives.

Common respiratory conditions during pregnancy

Nasal and sinus stuffiness, epitaxis, changes in the voice, marked inflammatory response (may → mild upper respiratory infection), swelling of tympanic membranes and eustacian tubes → impaired hearing, earaches, sense of fullness in ears

Which physiologic change is associated with a rise in human chorionic gonadotropin (hCG) levels during pregnancy? Fatigue Nasal congestion Breast tenderness Nausea and vomiting

Nausea and vomiting Nausea and vomiting are associated with increased hCG levels and periodic hypoglycemia.

"Method used to determine the expected date of birth (EDB) by subtracting _______ and adding _________ and ______ (if appropriate) to the first day of the last normal menstrual period (LNMP). Alternatively, ____________ can be added to the first day of the LNMP and counting forward _________."

Nägele's 3 calendar months; 7 days; 1 year; 7 days; 9 calendar months" Method used to determine the expected date of birth (EDB) by subtracting 3 calendar months and adding 7 days and 1 year (if appropriate) to the first day of the last normal menstrual period (LNMP). Alternatively, 7 days can be added to the first day of the LNMP and counting forward 9 calendar months.

Why is magnesium important for pregnant women?

Needed for energy and protein metabolism, tissue growth, muscle action.

Why is zinc important for pregnant women?

Needed for enzymes involved in metabolic pathways. Deficiencies in zinc can lead to central nervous system malformations Note: folic acid inhibits zinc absorption.

Why is calcium important for pregnant women?

Needed for fetal bone growth, teeth development, maintaining maternal bone mass. Calcium intake needs same as non pregnant women. non dairy if lactose intolerant or culture doesn't like milk for adults ( puerto rico)

A patient at 34 weeks' gestation and her partner report being nervous about the upcoming delivery and baby care. Which classes should the nurse recommend? Select all that apply. Newborn care Prenatal exercise Early prenatal care Preconception care Childbirth preparation

Newborn care Newborn care class should be recommended as they can teach expecting parents how to care for the infant. Adequate preparation and education can quell anxiety and fear. Childbirth preparation Childbirth preparation class should be recommended as adequate education regarding preparation for birth can help decrease the anxiety of the couple.

A woman is seeking a long-term contraceptive method that does not require taking daily medication or the application of a device before intercourse. Which contraceptive methods would be appropriate? Select all that apply. Lea's Shield Nexplanon implant Contraceptive sponge Intrauterine device (IUD) Combined oral contraceptives (COCs)

Nexplanon implant The Nexplanon progestin implant can provide up to three years of contraception after insertion. Since it does not require daily medication or the application of a device before intercourse, it would be an appropriate method of contraception for this woman. Intrauterine device (IUD) Intrauterine devices (IUDs) provide either five years (hormonal IUD) or ten years (copper IUD) of contraception after insertion. Since they do not require daily medication or the application of a device before intercourse, this would be an appropriate method of contraception for this woman.

Protein Recommendation for Lactation:

Non pregnant needs plus +25 g

Pica

Non-food cravings for substances such as ice, Clay, and laundry starch

teratogens

Nongenetic factors that cause malformations and disorders in utero.

Parity

Number of pregnancies in which the fetus or fetuses have reached 20 weeks of gestation when they are born, not the number of fetuses born (whether the fetus is born alive or stillborn)

A woman visits the clinic for preconception counseling. Which should the nurse assess? Select all that apply. Nutrition Maternal health Fetal heart tones Alcohol/drug use Current medications Fundal height measurement

Nutrition Nutrition, especially folic acid intake, is assessed. Folic acid should be consumed at least one month prior to conception to prevent neural tube defects. Maternal health Maternal health should be assessed, as well as chronic medical problems that can affect the ability to conceive and/or the pregnancy. Alcohol/drug use Alcohol/drug use should be assessed before conception as these are harmful to the developing fetus and to the mother. Current medications Current medications should be assessed as medications can be teratogenic to a developing fetus.

Magnesium Food Sources

Nuts, Legumes, Cocoa, Meats, Whole Grains

Semen analysis is a common diagnostic procedure related to infertility. In instructing a male client regarding this test, the nurse would tell him to: Ejaculate into a sterile container. Obtain the specimen after a period of abstinence from ejaculation of 2 to 5 days. Transport specimen with container packed in ice. Ensure that the specimen arrives at the laboratory within 30 minutes of ejaculation.

Obtain the specimen after a period of abstinence from ejaculation of 2 to 5 days. An ejaculated sample should be obtained after a period of abstinence to get the best results. The male must ejaculate into a clean container or a plastic sheath that does not contain a spermicide. He should avoid exposing the specimen to extremes of temperature, either heat or cold, and the specimen should be taken to the laboratory within 2 hours of ejaculation.

Which factors can negatively affect a woman's ability to psychologically adapt to pregnancy? Select all that apply. Older age Multiparity A large family High income level Frequent exercise

Older age Age can affect a woman's ability to psychologically adapt to pregnancy. Teens and women over 35 are at a higher risk. Multiparity Women with previous children may have difficulty psychologically adapting to pregnancy. A large family Adequate social support, as would likely be offered by a large family, would help, not hinder, a woman's ability to psychologically adapt to pregnancy.

Which is characteristic of monozygotic twinning? Two ova are fertilized. Two sperm fertilize the ovum. One sperm fertilizes one ovum. Two ova are released during ovulation.

One sperm fertilizes one ovum. Monozygotic twins develop from one fertilized ovum, but later the conceptus divides into two.

Gamete intrafallopian transfer

Ooctyes are retrieved from the ovary placed into a catheter with sperm and immediately transferred into the fimbriated end of the uterine tube, fertilization takes place in the uterine tube

Gamete intrafallopian transfer (GIFT)

Oocytes are retrieved from the ovary, placed in a catheter with washed motile sperm, and immediately transferred into the fimbriated end of the uterine tube. Fertilization occurs in the uterine tube. ovulation is induced and oocytes are aspired from follicles via laparoscopy semen is collected before laparoscopy ova and sperm are transferred to one uterine tube, permitting nautal fertilization and cleavage

Which statements accurately characterize how the female body prepares for conception? Select all that apply. Oocytes remain in the ovary in the Graafian follicle. The ovum is released on the fallopian tube surface. An ovum is released 14 days before the menstrual period begins. The Graafian follicle prepares the uterine lining by producing estrogen and progesterone. The ovum matures as follicle-stimulating hormone (FSH) and luteinizing hormone (LH) are released.

Oocytes remain in the ovary in the Graafian follicle. Each mature oocyte is contained in a sac within the ovary, called the Graafian follicle. An ovum is released 14 days before the menstrual period begins. Ovulation, or the release of the ovum, does occur about 14 days before a woman's next menstrual period begins. Correct The Graafian follicle prepares the uterine lining by producing estrogen and progesterone. The Graafian follicle does produce estrogen and progesterone to prepare the endometrium (uterine lining) for a possible pregnancy.

What does gametogenesis include?

Oogenesis & Spermatogenesis

The nurse should instruct the patient to consume which food or drink to enhance the absorption of iron during pregnancy? Milk Water Cheese Orange juice

Orange juice Vitamin-C-enriched fluids/foods, such as orange juice, enhance the absorption of iron.

How does the fetus get oxygen?

Oxygen-Rich blood from the placenta flows rapidly through the umbilical vein into the fetal abdomen special circulatory pathway - ductus arteriosus

Palmer Erythema

Palms pinkish red and modeled

carpal tunnel syndrome

Paresthesia and a pain in the right hand radiating to elbow

The nurse is assessing a pregnant patient in the women's health clinic. When planning care for the patient, the nurse recognizes which information supports the need for fetal diagnostic procedures to be performed? Select all that apply. Parity Maternal age Fetal movement Nondrug allergies History of cesarean delivery

Parity Grand multiparity (>5 pregnancies) indicates the need for fetal diagnostic procedures; therefore, parity is a relevant indicator of whether fetal diagnostic procedures should be performed. Maternal age Maternal age of <16 or >35 years indicates the need for fetal diagnostic procedures; therefore, age is a relevant indicator of whether fetal diagnostic procedures should be performed. Fetal movement Decrease in or absence of fetal movement indicates the need for fetal diagnostic procedures; therefore, fetal movement is a relevant indicator of whether fetal diagnostic procedures should be performed.

What does IgG provide?

Passive immunity to specific bacterial toxins

Ballottement

Passive movement of the unengaged fetus; technique of palpating a floating structure by bouncing it gently and feeling it rebound by placing the finger in the vagina, tapping it upward to make the fetus rise, and feeling it as the fetus sinks identified 16-18 weeks

Which type of exercise can help with lower back discomfort during pregnancy? Kegel Sit-ups Push-ups Pelvic tilts

Pelvic tilts Pelvic tilt exercises strengthen the muscles of the lower back and decrease backache during pregnancy.

surfactants

Phosphoprotein necessary for normal respiratory function that prevents the alveolar collapse (atelectasis)

Linea nigra

Pigmented line extending from the symphysis pubis to the top of the fundus in the midline

linea nigra

Pigmented line extending up abdominal midline

striae gravidarum

Pink stretch marks or depressed streaks on breast in abdomen

Palmar erythema

Pinkish-red, diffuse mottling or well-defined blotches on the hands

What takes care go fetal nutrition and elimination needs?

Placenta

Which auxiliary structures play a role in the elimination of fetal waste? Select all that apply. Placenta Umbilical vein Amniotic fluid Fetal membranes Umbilical arteries

Placenta The placenta plays a key role in transferring carbon dioxide, urea, uric acid, and bilirubin from the fetus to the mother for disposal. Umbilical arteries The umbilical arteries carry blood high in carbon dioxide and other waste away from the fetus toward the placenta, where these waste products are removed through the maternal circulation.

battledore placenta

Placenta with the umbilical cord inserted into the border

operculum

Plug of mucus Phils Endo cervical canal

Measurement of the fetal heart rate would be considered which sign of pregnancy? Positive Probable Hormonal Presumptive

Positive Positive signs of pregnancy include auscultation of the fetal heart rate.

presumptive

Pregnancy related changes felt by the woman

probable

Pregnancy related changes that can be observed by an examiner

Postdate or postterm

Pregnancy that goes beyond 42 weeks of gestation

Couvade Syndrome

Pregnancy-related ritual or a cluster of symptoms experienced by some prospective fathers during pregnancy and childbirth.

During the second trimester, which role does the expectant mother assume? Focus on self Preparation for birth Preparation for motherhood Seeking safe passage for self and fetus

Preparation for motherhood The expectant mother seeks acceptance of fetus and role of mother during the second trimester.

With regard to medications, herbs, shots, and other substances normally encountered, the maternity nurse should be aware that during pregnancy: Prescription and over-the-counter (OTC) drugs that otherwise are harmless can be made hazardous by metabolic deficiencies of the fetus. The greatest danger of drug-caused developmental deficits in the fetus is seen in the final trimester. Killed-virus vaccines (e.g., tetanus) should not be given, but live-virus vaccines (e.g., measles) are permissible. No convincing evidence exists that secondhand smoke is potentially dangerous to the fetus.

Prescription and over-the-counter (OTC) drugs that otherwise are harmless can be made hazardous by metabolic deficiencies of the fetus. The statement in A is especially true for new medications and combinations of drugs. The greatest danger of drug-caused developmental defects exists in the interval from fertilization through the first trimester, when a woman may not realize that she is pregnant. Live-virus vaccines should be part of postpartum care; killed-virus vaccines may be administered during pregnancy. Secondhand smoke is associated with fetal growth restriction and increases in infant mortality.

HCG

Presence of this bio chemical marker and maternal urine or sooner I'm results in a positive pregnancy test results

Identify how circulation after birth differs from fetal circulation. Select all that apply. Pressure decreases in the left atrium. Pressure decreases in the right atrium. The three fetal shunts constrict or close. Pressure in the aorta and pulmonary artery increases. Blood flow from the pulmonary artery to the aorta reverses.

Pressure decreases in the right atrium. After birth, pressure in the right atrium decreases. Correct The three fetal shunts constrict or close. After birth, the fetal circulatory shunts are not needed because the lungs oxygenate blood; as a result, the shunts close. Blood flow from the pulmonary artery to the aorta reverses. After birth, pressure in the aorta rises as pressure in the pulmonary artery falls. This causes the direction of blood flow through the ductus arteriosus to reverse, from the aorta to the pulmonary artery.

Why are women who have used contraceptive barrier methods more likely to conceive than those who did not?

Presumably because of the protective effects against tubal damage from sexually transmitted infections (STIs), especially chlamydia.

The woman reports fetal movement. This would be documented as which sign of pregnancy? Positive Probable Intuition Presumptive

Presumptive Presumptive signs of pregnancy do include fetal movement felt by the woman, known as quickening.

Why is protein important for a pregnant woman?

Primary role is to provide amino acids for synthesis of new tissues (ex fetal growth, mammary glands, uterine growth, expansion of blood volume, secretion of milk during lactation)

A woman who has completed one pregnancy with a fetus (or fetuses) reaching the stage of fetal viability is called a: Primipara Primigravida Multipara Nulligravida

Primipara A primipara is a woman who has completed one pregnancy with a viable fetus. To remember terms, keep in mind that gravida is a pregnant woman; para comes from parity, meaning a viable fetus; primi means first; multi means many; and null means none. A primigravida is a woman pregnant for the first time. A multipara is a woman who has completed two or more pregnancies with a viable fetus. A nulligravida is a woman who has never been pregnant.

Surrogate motherhood

Process by which a woman is inseminated with the semen from the infertile woman's partner and then carries the fetus until birth

B12 Role in relation to Pregnancy and Lactation:

Production of Nucleic Acids and Proteins, esp. Important in Formation of RBCs and Neural Functioning

hematopoeisis

Production of blood cells; formation of blood.

The ovaries increase secretion of which hormone to help maintain the pregnancy? Estrogen Prolactin Progesterone Follicle-stimulating hormone (FSH)

Progesterone Increases in progesterone hormone levels during pregnancy help maintain the pregnancy.

Subfertility

Prolonged time to conceive.

Fiber Role in relation to Pregnancy and Lactation:

Promotes regular bowel elimination; Reduces long-term risk for heart disease, diverticulosis, and diabetes

Common integumentary conditions of pregnancy

Pruritus gravidarum Gum hypertrophy Epulis: red, raised nodule on the gums that bleeds easily Accelerated nail growth Oily skin and acne Hirsutism Feeling hotter from increased blood supply to the skin

Why is anemia dangerous for pregnant women?

Puts them at risk for hemorrhage at birth. Anemia in 1st trimester can lead to preterm labor.

Which measures can a pregnant woman take to prevent or relieve heartburn? Select all that apply. Drink coffee Quit smoking Avoid fried chicken Eat a snack just before bed Take a brief walk after eating Eat 5-6 small meals throughout the day

Quit smoking Smoking increases heartburn and should be avoided completely. Correct Avoid fried chicken Fried foods increase heartburn and should be avoided. Take a brief walk after eating Sitting upright (or walking) for at least one hour after eating promotes gastric emptying and decreases heartburn. Correct Eat 5-6 small meals throughout the day Eating small, frequent meals allows for gastric emptying and decreases heartburn.

Many parents-to-be have questions about multiple births. Maternity nurses should be able to tell them that: Rates of twinning and other multiple births are increasing because of the use of fertility drugs and delayed childbearing. Dizygotic twins (two fertilized ova) have the potential to be conjoined twins. Identical twins are more common in Caucasian families. Fraternal twins are same gender, usually male.

Rates of twinning and other multiple births are increasing because of the use of fertility drugs and delayed childbearing. If the parents-to-be are older and have taken fertility drugs, they would be very interested in this information. Conjoined twins are monozygotic; they are from a single fertilized ovum in which division occurred very late. Identical twins show no racial or ethnic preference; fraternal twins are more common among African-American women. Fraternal twins can be different genders or the same gender. Identical twins are the same gender.

Epulis

Red raised nodule on gums so my: bleeds after brushing teeth

What is viability?

Refers to the capability of the fetus to survive outside the uterus

What are the metabolic functions of the placenta?

Respiration, nutrition, excretion, and storage

Rest and Relaxation

Rest and Relaxation -The pregnant woman is encouraged to plan regular rest periods, particularly as pregnancy advances. -Conscious relaxation is the process of releasing tension from the mind and body through deliberate effort and practice. -The ability to relax consciously and intentionally can be beneficial. -The techniques for conscious relaxation are numerous and varied.

The nurse understands that which intervention is most appropriate for a patient with Rh-negative blood who is unsensitized and just received percutaneous umbilical blood? RhoGAM administration Transvaginal ultrasound Transabdominal ultrasound Anticoagulant administration

RhoGAM administration A patient who is Rh-negative and unsensitized should be given RhoGAM following percutaneous umbilical blood sampling (PUBS) to prevent Rh sensitization.

In which direction does neonate blood flow from the inferior vena cava? Place the structures in order.

Right atrium Right ventricle Pulmonary artery Lungs This is the circulation process of a normal human heart. Deoxygenated blood will enter the right atrium and then travels through the tricuspid valve during diastole into the right ventricle. During systole, the heart pumps blood from the right ventricle into the pulmonary artery which delivers deoxygenated blood can be to the lungs to allow for adequate oxygenation.

What does the Ectoderm give rise to?

Rise to the epidermis, the glands, nails & hair, CNS & PNS, lens of the eye, tooth enamel, and floor of the amniotic cavity

What does the Endoderm give rise to?

Rise to the epithelium lining the respiratory & digestive tract, liver & pancreas, the urethra, the bladder, and the vagina

Why should patient education focus on smoking cessation during pregnancy? Risk of low blood sugar Risk of premature birth Risk of congenital anomalies Risk of increased fetal circulation

Risk of premature birth Smoking during pregnancy is related to preterm labor and low birth weight, both of which are associated with other neonate complications. It is crucial for patients to be educated about the potential risks of smoking.

A pregnant patient should expect which of the following information to be included during a childbirth preparation class? Select all that apply. Newborn care Role of support person Comfort relief during labor The labor and birth process Proper positioning and correct latch for lactation

Role of support person The role of the support person would be included in a childbirth preparation class. This helps inform the support person how to assist the mother during labor and birth. Increased knowledge will decrease anxiety for the support person as well. Correct Comfort relief during labor Comfort relief during labor would be included in a childbirth preparation class. This would increase the woman's knowledge of different comfort relief measures to help her cope with the pain of labor and birth. Correct The labor and birth process The labor and birth process should be included in a childbirth preparation class. This information helps pregnant women understand what to expect during labor and birth. It will help decrease anxiety related to fear of the unknown as well.

A patient with a family history of spina bifida is 16 weeks pregnant. Which is the priority action for the nurse to take? Obtain an order for RhoGAM. Schedule the patient for amniocentesis. Refer the patient to genetic counseling. Schedule the patient for a transabdominal ultrasound.

Schedule the patient for amniocentesis. Amniocentesis should be scheduled between 15 and 20 weeks when there is adequate amniotic fluid present to test AFP level, which is used in screening for neural tube defects such as spina bifida. Scheduling this test should be a priority action for the nurse to take.

Match the maternal task to the correct description. Seeking safe passage Securing acceptance Learning to give of herself Committing herself to the unknown child

Seeking safe passage Seek prenatal care and adhere to guidelines. Securing acceptance Gain support from family relationships. Learning to give of herself Learn to sacrifice self for the upcoming baby. Committing herself to the unknown child Developing attachment to the unborn baby.

A pregnant woman reports that she quit smoking and started taking prenatal vitamins as soon as she found out she was pregnant. These actions are related to which task of pregnancy? Securing acceptance Seeking safe passage Learning to give of herself Committing herself to the unknown child

Seeking safe passage Seeking safe passage refers to seeking prenatal care and adhering to healthcare provider recommendations.

What is the basic test for male infertility?

Semen analysis

Diastasis recti abdominis

Separation of the rectus abdominis msucles

foramen ovale

Septal opening between the atria of the fetal heart. The opening normally closes shortly after birth, but if it remains patent, surgical repair usually is necessary.

Radioreceptor assay (RRA)

Serum tests that measures the ability of a blood sample to inhibit the binding of radiolabeled hCG to receptors

A pregnant woman reports that she is still playing tennis at 32 weeks of gestation. The nurse would be most concerned regarding what this woman consumes during and after tennis matches. Which is the most important? Several glasses of fluid Extra protein sources, such as peanut butter Salty foods to replace lost sodium Easily digested sources of carbohydrate

Several glasses of fluid If no medical or obstetric problems contraindicate physical activity, pregnant women should get 30 minutes of moderate physical exercise daily. Liberal amounts of fluid should be consumed before, during, and after exercise, because dehydration can trigger premature labor. Also the woman's calorie and carbohydrate intake should be sufficient to meet the increased needs of pregnancy and the demands of exercise. All pregnant women should consume the necessary amount of protein in their diet, regardless of level of activity. Many pregnant women of this gestation tend to retain fluid, possibly contributing to hypertension and swelling. An adequate fluid intake prior to and after exercise should be sufficient without trying to replace sodium.

Which behavior indicates that a woman is "seeking safe passage" for herself and her infant? She keeps all prenatal appointments. She "eats for two." She drives her car slowly. She wears only low-heeled shoes.

She keeps all prenatal appointments. The goal of prenatal care is to foster a safe birth for the infant and mother. Keeping all prenatal appointments is a good indication that the woman is indeed seeking "safe passage." Eating properly, driving carefully, using proper body mechanics, and wearing appropriate footwear during pregnancy are healthy measures that all pregnant women should take.

The nurse advises the woman who wants to have a nurse-midwife provide obstetric care that: She will have to give birth at home. She must see an obstetrician as well as the midwife during pregnancy. She will not be able to have epidural analgesia for labor pain. She must be having a low-risk pregnancy.

She must be having a low-risk pregnancy. Midwives usually see low-risk obstetric clients. Nurse-midwives must refer clients to physicians for complications. Most nurse-midwife births are managed in hospitals or birth centers; a few may be managed in the home. Nurse-midwives may practice with physicians or independently with an arrangement for physician backup. They must refer clients to physicians for complications, but patients are not required to see an obstetrician otherwise. Care in a midwifery model is noninterventional, and the woman and family usually are encouraged to be active participants in the care; this does not imply that medications for pain control are prohibited.

A woman taking human menopausal gonadotropins for infertility should understand which of the following regarding medications

She must report for ultrasound testing as scheduled to monitor follicular development

Positive signs of pregnancy

Signs that are attributable only to the presence of a fetus Visualization of fetus by ultrasound Fetal heart tones by ultrasound Visualization of fetus by radiographic study Fetal heart tones detected by Doppler ultrasound stethoscope Fetal heart tones detected by fetal stethoscope Fetal movements palpated Fetal movements visible

positive

Signs that can be attributed only to the presence of a fetus

Cardiovascular changes in pregnancy

Slight cardiac hypertrophy secondary to increased blood volume and CO Diaphragm is displaced upward Heart is elevated upward and rotated left; apical impulse is shifted upward and laterally 1-1.5cm More audible splitting of S1, S2, and S3 after 20 weeks Murmurs may be heard in pulmonic area (transient) Increase in pulse In twin gestation, maternal HR increases significantly in the 3rd trimester pregnant woman may experience palpitation, sinus dysrhythmias, premature atrial contractions, premature ventrical systole Cardiac rhythm may be altered, which usually does not require treatment unless there is an underlying cardiac issue

A nurse is on the phone with a woman three weeks after her intrauterine device (IUD) insertion. Which symptoms should be reported to the woman's health care provider immediately? Smelly vaginal discharge Strings can be felt during intercourse Stronger cramps during menstruation Strings are the same length as the day of insertion

Smelly vaginal discharge Unusual or malodorous vaginal discharge is a sign of infection and may indicate serious complications. Women with an IUD are at greater risk for pelvic inflammatory disease (PID), and symptoms of infection should be reported to the health care provider immediately.

Which behaviors should the nurse instruct the woman to avoid during pregnancy? Select all that apply. Exercise. Smoke marijuana to aid in sleep. Consume 8-10 glasses of water per day. Drink one glass of white wine per day for heart health. Drink 5-6 cups of coffee per day to avoid caffeine withdrawal.

Smoke marijuana to aid in sleep. Taking any illicit drugs during pregnancy should be discouraged, regardless of the reason. Marijuana is associated with neurobehavioral defects in newborns. Drink one glass of white wine per day for heart health. Drinking alcohol during pregnancy should be discouraged because it can cause serious birth defects. Even small amounts of alcohol are not recommended. Drink 5-6 cups of coffee per day to avoid caffeine withdrawal. Consuming large amounts of caffeine during pregnancy should be discouraged, regardless of the reason. It can cause spontaneous abortion and preterm labor.

Goodell sign

Softening of the cervical tip caused by increased vascularity, slight hypertrophy, and hyperplasia of the muscle and connective tissue Connective tissue becomes loose, edematous, highly elastic, and increased in volume with a velvety appearance

What is the test that uses fluid infused into teh uterus via the cervix to help define the uterine cavity adn the depth of the uterine lining via vaginal ultrasound?

Sonohysterography

funic souffle

Sound of fetal blood coursing Through the umbilical cord; it is synchronous with the fetal heart rate

Uterine souffle

Sounds made by blood in the uterine arteries that is synchronous with maternal pulse

Funic souffle

Sounds made by the blood rushing through the umbilical vessels and synchronous with fetal heart rate

Clean-catch urine

Specimen is obtained to test for levels of glucose, protein, nitrites, and leukocytes at each visit.

A couple presents for their first appointment at an infertility center. A noninvasive test done during initial diagnostic testing is: Hysterosalpingogram Endometrial biopsy Sperm analysis Laparoscopy

Sperm analysis Sperm analysis, the basic test for male infertility, is the only noninvasive procedure listed. A hysterosalpingogram is radiographic film examination that allows visualization of the uterine cavity after instillation of radiopaque contrast medium through the cervix. During an endometrial biopsy, a small cannula is introduced into the uterus and a portion of the endometrium is removed for histologic examination. Laparoscopy is useful in order to view the pelvic structures intraperitoneally.

Therapeutic donor insemination

Sperm from a person other than the male partner are used to inseminate the female partner

Waht is the immunologic test to determ sperm and cervical mucus interaction?

Sperm immobilization antibody reaction

Friability

Spotting following the cervical palpitations or intercourse

An infertile woman is given clomiphene citrate (clomid) to achieve which of the following therapeutic outcomes

Stimulate the pituitary gland to secrete FSH and LH

A pregnant patient is traveling for a family vacation. She will be driving in the car for eight hours. What should the nurse instruct the patient to do while traveling? Decrease fluid intake. Drive with minimal stops. Avoid travel by automobile. Stop frequently to walk around.

Stop frequently to walk around. The nurse should instruct the patient to stop every two hours and ambulate for ten minutes. This will help prevent the formation of clots in the lower extremities.

Striae gravidarum

Stretch marks May be caused by adrenocorticosteroids On the abdomen, thighs, breasts Usually fade after birth

How do hormones prepare the vagina?

Stretching, mucosa thickens, connective tissue loosens, smooth muscles hypertrophy, vaginal vault lengthens

Nutrition/culturally

Such a conflict commonly is not known by health care providers unless they understand the dietary beliefs and practices of the particular people for whom they are caring.

Protein Role in relation to Pregnancy and Lactation:

Synthesis of the products of conception; Growth of maternal tissue and expansion of blood volume; Secretion of milk protein during lactation.

Change in BP

Systolic BP may decrease slightly Diastolic BP decreases in 1st trimester and gradually increases to prepregnancy levels Supine hypotensive syndrome maternal position affects readings ( and cuff size)

drugs supporting ovulation in women hypothyrodism

TSH (Synthroid)

A pregnant woman reports constipation. In which ways can the woman prevent or relieve constipation? Select all that apply. Drink caffeinated coffee in the mornings Increase consumption of dairy products Take over-the-counter docustate sodium Consume at least eight glasses of water a day Walk or engage in light aerobic exercise daily Eat green leafy vegetables, whole grains, and fruit

Take over-the-counter docustate sodium Stool softeners such as docusate sodium increase water content of the stool and promote bowel movement, thus relieving constipation. Consume at least eight glasses of water a day Increasing fluid intake will increase peristalsis, soften the stool, and decrease constipation. Walk or engage in light aerobic exercise daily Walking and light aerobic exercise will increase peristalsis and decrease constipation. Eat green leafy vegetables, whole grains, and fruit Eating fiber-enriched foods helps to prevent constipation.

Zinc Recommendation for Pregnancy:

Teen- 12 mg Adult- 11 mg:

Zinc Recommendation for Lactation:

Teen- 13 mg Adult- 12 mg

Iron (mg) Recommendation for Nonpregnant Woman:

Teen- 15 mg Adult- 18 mg

Zinc (mg) Recommendation for Nonpregnant Woman:

Teen- 9 mg Adult- 8 mg

Iron Recommendation for Lactation:

Teens- 10 mg Adult- 9 mg

Azoospermia

Term for no sperm cells produced

Sterility

Term that refers to an inability to conceive

Infertility

Term that refers to subfertility, a prolonged time to conceive

Ultrasonography

Test used to visualize pelvic tissues and structures for a variety of reasons, it may be accomplished via a trans abdominal or trans vaginal approach

Radioimmunoassay (RIA)

Tests for beta subunit of hCG in serum or urine samples; usually in a lab

How does a miscarriage happen?

The Corpus Lutem stops functioning before the placenta is producing sufficient amounts of estrogen and progesterone

Age differences

The age of the childbearing couple may have a significant influence on their physical and psychosocial adaptation to pregnancy.

What forms the umbilicus?

The connecting stalk becomes compressed from both sides by the amnion

Below are four important landmarks of fetal development. Please place them in chronological order: 1. Four-chambered heart is formed. 2. Vernix caseosa is present. 3. Blastocyst development is complete. 4. Testes have descended into the scrotal sac.

The correct order is 3, 1, 2, 4. 3. The blastocyst is developed about 6 days after fertilization and before implantation in the uterus has occurred. 1. The four-chambered heart is formed during the early part of the first trimester. 2. Vernix caseosa is present during the lat ter half of pregnancy. 4. The testes descend in the scrotal sac about mid third trimester.

Using the History

The couple's sexual history provides a basis for counseling, but history taking also is an ongoing process.

Abruptio Placentae (placental separation)

The detachment of part or all of a normally implanted placenta from the uterus before the birth of the infant.

A woman at eight weeks gestation asks the nurse why she is so nauseated with her pregnancy. What should the nurse explain to the woman? Nausea is related to a decrease in progesterone. Nausea is caused by consumption of excess sugar. Nausea is not aggravated by foul odors and fatigue. The exact cause of nausea during pregnancy is unknown. It may be related to an increase in hCG.

The exact cause of nausea during pregnancy is unknown. It may be related to an increase in hCG. Nausea may be related to an increase in hCG.

With regard to the father's acceptance of the pregnancy and preparation for childbirth, the maternity nurse should know that: The father goes through three phases of acceptance of his own. The father's attachment to the fetus cannot be as strong as that of the mother because it does not start until after birth. In the last 2 months of pregnancy, most expectant fathers suddenly get very protective of their established lifestyle and resist making changes to the home. Typically men remain ambivalent about fatherhood right up to the birth of their child.

The father goes through three phases of acceptance of his own. A father typically goes through three phases of acceptance: accepting the biologic fact, adjusting to the reality, and focusing on his role. Typically, the expectant father's ambivalence ends by the first trimester, and he progresses to adjusting to the reality of the situation and focusing on his role. The father-child attachment can be as strong as the mother-child relationship and can begin during pregnancy. In the last 2 months of pregnancy, many expectant fathers work hard to improve the environment of the home for the child.

"Announcement phase; biologic fact of pregnancy; moratorium; accept the pregnancy; focusing; labor; parenthood"

The first period of paternal adaptation, during which the father accepts the_________. During the second or _______ phase, the father adjusts to the reality of the pregnancy. The developmental task is to ______. The father be- comes actively involved in the pregnancy and the relationship with his child during the third or _________ phase. The developmental task is to negotiate with his partner the role he is to play in ___________ and to prepare for _______.

A woman at 35 weeks of gestation has had an amniocentesis. The results reveal that surface-active phospholipids are present in the amniotic fluid. The nurse is aware that this finding indicates that: The fetus is at risk for Down syndrome. The woman is at high risk for developing preterm labor. The lungs are mature. Meconium is present in the amniotic fluid.

The lungs are mature. The detection of the presence of pulmonary surfactants, surface-active phospholipids, in amniotic fluid has been used to determine fetal lung maturity, or the ability of the lungs to function after birth. This occurs at approximately 35 weeks of gestation. This result is unrelated to Down syndrome and in no way indicates risk for preterm labor. Meconium should not be present in the amniotic fluid.

Which statement describes how the fertilization process in humans results in 46 chromosomes? The 23 chromosomes from the sperm and ovum are doubled. The nuclei of the gametes touch and dissolve, producing 46 chromosomes. The zona pellucida prevents sperm from entering after 46 chromosomes are restored. The fertilized ovum contains 23 unpaired chromosomes, 22 autosomes, and a Y chromosome.

The nuclei of the gametes touch and dissolve, producing 46 chromosomes. The nuclei of the gametes move toward the center of the ovum, where the membranes surrounding their nuclei touch and dissolve. As a result, the 23 chromosomes from the sperm mingle with the 23 from the ovum, restoring the diploid number to 46.

parity

The number of pregnancies in which the fetus or fetuses have reached 20 weeks of gestation. Whether the fetus is born alive or stillborn

What happens to the ovum NOT fertilized by the sperm?

The ovum degenerates and is reabsorbed

What is "capacitation"?

The physiologic change the removes the protective coating from the heads of the sperm To allow the sperm to penetrate the protective layers of the ovum before fertilization

What is Oogenesis?

The process of ovum formation

What is Meiosis?

The process when cells divide and decrease their chromosomal number by half through separation of chromosomes and then produce 2 haploid cells ( gamets - egg and sperm)

what is Spermatogenesis?

The process when male primary cells undergo meiosis and produce spermatogonia

A 22-year-old woman pregnant with a single fetus had a preconception body mass index (BMI) of 24. When she was seen in the clinic at 14 weeks of gestation, she had gained 1.8 kg (4 lbs) since conception. How would the nurse interpret this finding? This weight gain indicates possible gestational hypertension. This weight gain indicates that the woman's infant is at risk for intrauterine growth restriction (IUGR). This weight gain cannot be evaluated until the woman has been observed for several more weeks. The woman's weight gain is appropriate for this stage of pregnancy.

The woman's weight gain is appropriate for this stage of pregnancy. During the first trimester, the average total weight gain is only 1 to 2.5 kg. The desirable weight gain during pregnancy varies among women. Weight gain should take place throughout the pregnancy. The optimal rate depends on the stage of the pregnancy. The primary factor to consider in making a weight gain recommendation is the appropriateness of the prepregnancy weight for the woman's height. A commonly used method of evaluating the appropriateness of weight for height is the BMI. This woman's BMI is within the normal range, and she has gained the appropriate amount of weight for her size at this point in her pregnancy. Although the statements in A through C are accurate, they do not apply to this client.

Assisted hatching

The zona pellucida is penetrated chemically or manually to create an opening for the dividing embryo to hatch and to implant into the uterine wall

The events of conception occur in which order after fertilization of the ovum? The zygote divides in the fallopian tube. The blastocyst develops. The zygote cells divide and shrink in size. The morula forms.

The zygote divides in the fallopian tube. The zygote cells divide and shrink in size. The morula forms. The blastocyst develops.

Students are completing a clinical rotation in the prenatal care clinic. Which statements demonstrate understanding of when and how human chorionic gonadotropin (hCG) is secreted? Select all that apply. The zygote secretes hCG. A high level of hCG is a pregnancy indicator. Pregnant women have a high level of progesterone. Ovulating women have higher amounts of hCG in their system. Increased hCG production causes decreased estrogen secretion.

The zygote secretes hCG. The zygote secretes hGC to maintain the decidua. Correct A high level of hCG is a pregnancy indicator. The zygote secretes hCG to signal that a pregnancy has begun; therefore a high level of hCG is an indicator of pregnancy. Correct Pregnant women have a high level of progesterone. Implantation and survival of the conceptus are critically dependent on a continuing supply of estrogen and progesterone to maintain the decidua in the secretory phase. With continued hCG production by the conceptus, the corpus luteum continues to secrete estrogen and progesterone rather than regressing.

What is the purpose of chorionic villi?

These vascular processes obtain oxygen and nutrients from the maternal bloodstream and dispose of CO2 and waste products into the maternal blood

How are Braxton Hicks contractions different from labor?

They do not increase in intensity or frequency or cause cervical dilation

Abdominal breathing is replaced by

Thoracic/costal breathing

Vitamin C Role in relation to Pregnancy and Lactation:

Tissue Formation and Integrity, esp. connective tissue; enhancement of iron absorption

What are the goals of perinatal education classes? Select all that apply. To decrease anxiety related to childbirth To teach parents how to perform CPR for their children To provide knowledge related to the labor process and birth To increase knowledge of comfort relief measures during labor To teach the expectant families how to parent and discipline their children

To decrease anxiety related to childbirth Decreasing anxiety related to childbirth is a goal of perinatal education classes. To provide knowledge related to the labor process and birth Providing knowledge related to the labor process and birth is a goal of perinatal education classes. Correct To increase knowledge of comfort relief measures during labor Increasing knowledge of comfort relief measures during labor is a goal of perinatal education classes.

"Birth plan"

Tool that can be used by parents to explore their childbirth options and choose those that are most important to them; it serves as a tentative guide because the realities of what is feasible may change as the actual labor and birth progress.

increased prolactin level may cause amenorrhea and anovulation the same way it does during lactation.

True

Which contraception methods would be appropriate for a woman who is intolerant to hormonal therapy? Select all that apply. Tubal sterilization Emergency contraception (EC) Contraceptive vaginal ring Diaphragm with spermicide Copper intrauterine device (IUD)

Tubal sterilization Tubal sterilization is a surgical procedure that uses no hormones, and the procedure is an appropriate method of contraception for a woman who is intolerant to hormonal therapy. Diaphragm with spermicide A diaphragm does not contain hormones, nor does the spermicide used with it, and the combination of the two would be appropriate for a woman who is intolerant to hormonal therapy. Correct Copper intrauterine device (IUD) The Paragard copper intrauterine device does not contain any hormones and is appropriate for a woman who is intolerant to hormonal therapy.

Which statement about multifetal pregnancy is not accurate? The expectant mother often experiences anemia because the fetuses have a greater demand for iron. Twin pregnancies come to term with the same frequency as single pregnancies. The mother should be counseled to increase her nutritional intake and gain more weight. Backache and varicose veins are often more pronounced.

Twin pregnancies come to term with the same frequency as single pregnancies. Twin pregnancies often end in prematurity; serious efforts should be made to bring the pregnancy to term. A woman with a multifetal pregnancy often experiences anemia because of the increased demands of two fetuses; this issue should be monitored closely throughout her pregnancy. The client may need nutrition counseling to ensure that she gains more weight than what is needed for a singleton birth. The considerable uterine distention in multifetal pregnancy is likely to cause backache and leg varicosities; maternal support hose should be recommended.

monozygotic

Twins developed from a single fertilized ovum; identical twins.

dizygotic

Twins developed from two separate ova fertilized by two separate sperm at the same time; fraternal twins.

Which factor increases the risk for death or injury of monozygotic fetuses? Twins that share placentas Twins with separate morulas Twins with one amnion and one chorion Twins that develop from two inner cell masses

Twins with one amnion and one chorion Monozygotic twins that share both an amnion and a chorion are at risk for umbilical cord entanglement, which, if occurs, can result in death of one or both twins.

The nurse would expect which findings in a twin pregnancy? Select all that apply. Conjoinment Two chorions Shared amnions Identical fetuses Separate placentas

Two chorions Dizygotic twin pregnancies have two chorions because the twins develop from two zygotes. Separate placentas Dizygotic twin pregnancies have two placentas because the twins develop from two zygotes; though the placentas can fuse.

Hysteroscopy

Use of a flexible scope threaded through the cervix to view the uterine cavity directly

ovarian stimulation

Use of drugs to stimulate ovaries to develop follicles and eggs clomiphene citrate challenge test

Enzyme-linked immunosorbent assay (ELISA)

Uses specific monoclonal antibody (anti-hCG) with enzymes to bond with hCG in urine •Most common •Results in less than 5 min •Simple color-change reaction or digital reading •Most common error: performing test too early •Use first-voided morning urine •Medications can cause incorrect results

What does Estrogen stimulate?

Uteri growth and uteroplacental blood flow

Braxton Hicks

Uterine contractions that can be felt through the abdominal wall soon after the fourth month of pregnancy

hemorrhoids

Varicosities around the anus

Which placental variation presents the most risk to fetal safety? Abnormal Battledore Vasa previa Velamentous

Vasa previa The vasa previa placental variation occurs when the cord vessels branch far out on the membranes. When membranes rupture, fetal umbilical vessels may be torn, and the fetus can hemorrhage; therefore, causing risk to fetal safety.

Angiomas

Vascular spiders Star-shaped or branched, slightly raised and pulsating end-arterioles usually found on the neck, thorax, face, and arms From elevated levels of estrogen Bluish in color

Angiomas

Vascular spiders on neck and thorax

Foods Providing 50 mcg or more per serving of Folate:

Vegetables (1/2 cup) Broccoli Beans: Lima beans, baked beans, or pork and beans Greens: collards or mustard, cooked Okra, cooked Spinach, raw Fruits (1/2 cup) Avocado Orange or orange juice Pasta, cooked (1 cup)

Vitamin E Food Sources:

Vegetables Oils, Green leafy vegetables, Whole Grains, Liver, Nuts and Seeds, Cheese, Fish

A pregnant woman lies on her back for auscultation of fetal heart tones. She reports shortness of breath, chest pain, and dizziness. Which is the most likely cause of these symptoms? Vena cava syndrome Increased estrogen levels Decreased uterine growth Increased progesterone levels

Vena cava syndrome Vena cava syndrome is most likely the cause of these symptoms. If the gravid uterus partially occludes the vena cava and the aorta, this causes decreased cardiac output and maternal hypotension.

Match the pregnancy signs to the category. Visualization of fetal outline by ultrasound Softened uterus Quickening

Visualization of fetal outline by ultrasound Positive Softened uterus Probable Quickening Presumptive

Laparoscopy

Visualization of pelvic structures intraperitoneally outside the uterus by inserting a small endoscope through an incision in the anterior abdominal wall

What antioxidants and vitamins have been shown to benefit effects of male infertility.

Vitamins E and C, selenium, zinc, coenzyme Q10, and ginseng

What are the fat-soluble vitamins and what health risk can the pose to pregnant women?

Vitimins A, D, E K. These vitamins are stored in body tissues and can reach toxic level if overdose. Note: water-soluble vitamins are unlikely to cause toxicity (excreted in urine)

When does the fetus start to make respiratory movements and move/change positions in the utero?

WEEK 11 or 12

When does bile begin to form?

WEEK 12

Water Food Sources

Water and beverages made with water, milk, juices, all foods, Esp. frozen desserts, fruits, lettuce and other fresh vegetables

When does the development of the respiratory tract begin?

Week 4 and continues through Week 17

Oligohydramnios

What is having less than 300 mL of amniotic fluid called?

Polyhydramnios

What is having mote than 2 L of amniotic fluid called?

zona reaction

When a sperm successfully penetrates the membrane surrounding the ovum, both sperm and ovum are enclosed within the membrane, and the membrane becomes impenetrable to other sperm

Human chorionic gonadotropin in the urine, ballottement, Hagar sign

When assessing pregnant women during routine prenatal check ups which findings would be characterized as probable signs of pregnancy

Why can pelvic infections result in an ectopic pregnancy?

When infection and purulent discharge heals, the adhesions that form can block the tube. adhesions may permit the tiny sperm to pass through the tube, but then prevent a larger fertilized egg from completing the journey into the untrauterine cavity -> ectopic pregnancy

When does Spermatogenesis begin?

When the male reaches puberty his testes being this process

Leukorrhea

White or grayish mucoid discharge with a faint musty odor that is made of vaginal epithelial cells caused by hyperplasia of normal pregnancy

leukorrhea

White or slightly gray mucoid vaginal discharge with faint musty odor

Fiber Food Sources:

Whole Grains, bran, vegetables, fruits, nuts and seeds

The nurse discusses sexual intimacy with a pregnant couple. Which of the follow ing should be included in the teaching plan? 1. Vaginal intercourse should cease by the beginning of the third trimester. 2. Breast fondling should be discouraged because of the potential for preterm labor. 3. The couple may find it necessary to experiment with alternate positions. 4. Vaginal lubricant should be used sparingly throughout the pregnancy.

With increasing size of the uterine body, the couple may need counseling regarding alternate options for sexual intimacy.

When do most newborns void post delivery?

Within 24 hrs

Multigravida

Woman who has had two or more pregnancies

What is a the diploid number of human chromosomes?

Zygote

At which stage of development is separation of dizygotic twins apparent? Zygote Conceptus At conception End of 1st trimester

Zygote The membranes and placentas of dizygotic twins are separate because they arise from two different zygotes.

food cravings

__ __ are the urge to consume specific types of foods such as ice cream, pickles, and pizza during pregnancy

folate (folic acid); neural tube defects; .4mg

__ is a nutrient, the adequate intake of which is important for decreasing risk for __ or failures in the closure of the neural tube. An intake of __ daily is recommended for all women capable of becoming pregnant

ketonuria; perterm labor

__ is presence of ketones in the urine as a result of catabolism of fat stores; it is associated with the occurrence of __.

pica; clay; soil; laundry starch; ice/freezer frost; baking powder; baking soda; cornstarch

__ is the practice of consuming nonfood substances such as __, __, and __ __ or excessive amounts of foodstuffs that are low in nutritional value such as __/__ __, __ __, __ __, and __.

SYNTHETIC (NOT EXOGENOUS) PROGESTERONE

___________ is beneficial as adjunct therapy to improve the uterine lining after stimulated ovulation or when donor oocytes are used.

Morula

a 16 cell solid ball of cells that developes within 3 days of fertilization and is still surrounded by the protective zona pellucida

Haploid

a cell with 23 single chromosomes

Diploid

a cell with 46 chromosomes or 23 pairs

Morning sickness (n/v of pregnancy)

a discomfort most commonly experienced in the first trimester of pregnancy; it usually causes only mild to moderate nutritional problems but may be a source of substantial discomfort

late term

a pregnancy that had reached between 42 week 0 days and beyond of gestation

preterm

a pregnancy that has reached 20 weeks of gestation but ends before completion of 37 weeks of gestation

late preterm

a pregnancy that has reached between 34 weeks 0 days and 36 weeks 6 days of gestation

early term

a pregnancy that has reached between 37 weeks 0 days and 38 weeks 6 days of gestation

full term

a pregnancy that has reached between 39 weeks 0 days and 40 weeks 6 days of gestation

hysterosalpingography

a radiographic (x-ray) film examination allows visualization of the uterine cavity and tubes after the instillation of radiopaque contrast material through the cervix

Gravida 1 Para 0

a woman is pregnant for the first time (primagravida) and has not carried a pregnancy to 20 weeks (nullipara) if a woman had twins at 36 weeks with her first pregnancy, she would be gravida 1, para 1

Gravida

a woman who is pregnant

Primagravida

a woman who is pregnant for the first time

When caring for a preg women the nurse would recognize the women with what disorder has the greatest risk for giving birth to a macrosomic? a) diabetes b) anemia c)hyperthyroidism d) hypertension

a) diabetes

GnRH antagonists (ganirelix acetate, cetrorelix acetate)/ side effects

abdominal pain headache vaginal bleeding irritation at the injection site

Why do pregnant women require more nutrients in the 2nd and 3rd trimesters?

additional nutrients needed for: uterine-placental unit, total blood volume increase, mammary development, increased metabolic rates, energy needed for tissue synthesis.

subcutaneous fat

after 32 weeks

sufficient surfactant in developed alveoli

after 32 weeks

Braxton Hicks sign

after 4th month Painless around the fourth month uterine contractions that can be felt through the abdominal wall Facilitate uterine blood flow through the placenta and promote oxygen delivery to the fetus asfter 28th week contractions more definite

blood type determined

after 6th week

zygote intrafallopian transfer (ZIFT)

after IVF the ova are placed in one uterine tube during zygote stage fertilization occurs in vitro, and then the zygote is placed in the uterine tube

yolk sac function

aids in transferring maternal nutrients and oxygen, which have diffused through the chorion, to the embryo. Blood vessels form to aid transport. Blood cells and plasma are manufactured here during the second and third weeks while uteroplacental circulation is being established and is forming primitive blood cells until hematopoietic activity begins. At the end of the third week the primitive heart begins to beat and circulate the blood through the embryo, the connecting stalk, the chorion, and the yolk sac. The folding in of the embryo during the fourth week results in incorporation of part of this structure into the embryo's body as the primitive digestive system. Primordial germ cells arise in the yolk sac and move into the embryo.

amniotic fluid contains

albumin, urea, uric acid, creatinine, lecithin, sphingomyelin, bilirubin, fructose, fat, leukocytes, proteins, epithelial cells, enzymes, and lanugo hair

meiosis/process of DNA replication and cell division

allows alleles for genes to be distributed at random by each parent and then rearranged on the paired chromosomes

subjective symptoms of pregnancy

amenorrea nausea vomiting breast tenderness urinary frequency fatigue

fertilizations occurs

ampulla of the uterine tube

false positive

anticonvulsants tranquilizers

lanugo

appears first at 12 weeks on the eyebrows and upper lip by 20th week - entire body

Letrozole off-label use/ MOA

aromatase inhibitor that inhobits E2 production (estradiol), whcih causes an increase in LH:FSH ratio

Implantation

attachment process whereby the blastocyst burrows into the endometrium between 6 to 10 days after conception, the trophoblast secretes enzymes that enable it to burrow into the endometrium until the entire blastocyst is covered.

The results of an amniocentesis show that the preg women lecithin-sphingomyelin (L-S) ratio is 2;1. This results indicates that; a) the women is in her second trimester of preg b)the newborn should be able to maintain effective respiration after birth c) the fetus most likely has developed a renal problem d) an open neural tub defect is preset

b)the newborn should be able to maintain effective respiration after birth

endocervical mucus function

barrier against infection

fetus synthesizes glycogen for storage in liver

begin week 9

glycogen is stored

begin weeks 9-10 major source of energy

clomiphene citrate MOA

bids to estrogen receptors in the pituitary, blocking them from detecting estrogen

Alcohol teratogenic effects

birth defects impaired cognitive and psychomotor development emotional and behavioral problems

low birth weight

birth weight 2500 g or less

anthropometric measurements

body measurements such as height and weight

stem cells seed

bone marrow spleen thymus lymph nodes (8-11 wekks)

XY=

boy

exogenous progesterone/ side effects

breast tenderness local irritation headaches

drugs supporting ovulation in women with hyperprolactemia

bromocriptine to treat anovulation

handprints and footprints uniwue to each infant

by 16th weeks

2 more layers of cells/skin

by 7th week

genetic testing can be done

by 8th week

If a preg women intake of iron is sufficient her newborn will have enough iron stored in its liver approx how many months after birth? a) 1 month b)3 month c) 5 month d) 9 months

c) 5 month

Primary anovulation

caused by a pituitary or hypothalamic hormone disorder or an adrenal gland disorder, such as congenital adrenal hyperplasia.

Presumptive signs of pregnancy

changes felt by women - amenorrhea, fatigue, breast changes (fullness, pronounced nipples), nausea, vomiting, urinary frequency, quickening, discoloration of vaginal mucosa

headaches interventions

changing position slowly - applying cool cloth the the forehead - eating a small snack - using acetominophen only if prescribed

decidua basalis

chorionic villi tap into the maternal blood vessels

damage to nervous system is caused by

chronic poor nutrition hypoxia drugs environmetal toxins trauma disease

stage of the ovum

conception - day 14 cellular replication blastocyst formation initial development of the embryonic membrance establishment of the primary germ layers

Pre-embryonic

conception until day 14

Wharton's jelly

connective tissue that prevents compression of the blood vessels and ensures continued nourishment of the embryo/fetus

sphigomyelin

constant amount

GnRH antagonists (ganirelix acetate, cetrorelix acetate)/ indications

controlled ovarian stimulation for infertility treatment

embryo

day 15 - 8 weeks after conception 3 cm most crytical - development of the organ systems and the main external features

embryonic heart beating

day 17 maternal placental embryonic circulation in place

Oligohydramnios

decreased amount of the amniotic fluid <300ml

accelerate lung maturity

decreased maternal placental blood flow maternal hypertension' placental dysfunction infecition corticosteroid use due to fetal hypoxia, stresses the fetus, increases the blood level of corticosteroids and accelerate alveolar and surfactant development

Conception

defined as the union of a single egg and sperm, marks the beginning of a pregnancy.

uterine enlargement

determined by measuring fundal height and used to estimate the weeks of gestation

amnion

develops from the interior cells of the blastocyst becomes covering of the umbelical cord and covers the chorion on the fetal surface of the placenta

chorion

develops from tophoblasts and contains the chorionic villi on its surface covering of the featl side of placenta contains major umbelical blood vessels

mitosis has 2

diploid cells

Follitropins (purified FSH)/MOA

direct action on ovarian follicle

human chorionic gonadotropin (hCG)/ MOA

direct action on ovarian follicle to stimulate meiosis and rapture of the follicle

exogenous progesterone/ MOA

direct stimulation of endometrium

constipation

discomfort of pregnancy characterized by infrequent and difficult passage of hard, dry stool

pyrosis (heartburn)

discomfort of pregnancy that is usually caused by reflux of gastric contents into the esophagus

gallbladder

distended hypercholesterolemia, development of gallstones ( from increased progesterone)

false negative

diuretics promethazine

Embryo donation

donated embryo is transferred to the uterus of an infwertile women at the appropriate time ( normal or induced) of the menstrual cycle

Preimplantation Genetic Diagnosis (PGD)

done on a single cell removed from each embryo after 3-4 days it is designed to screen for inherited diseases developing embryos without the gene associated disease are transferred to the uterus

Therapeutic donor insemination (TDI)

donor sperm are used to inseminate the female partner

ovum released

during ovulation from the ruptured ovarian follicle

sexual dysfunction can manifest as

dyspareunia decreased libido unrealistic or rigid routines decreased body image depression ambivalence

constipation interventiosn

eating high fiber food - drinking no less than 2000 ml - exercising regularly - cunsulting about the use of stool softeners

Heartburn interventions

eating small frequent meals stitting up right fro 30 min after meal drinking milk between meals avoiding fatty and spicy food performing tailor sitting exercises consult about using antacids

Donor oocyte

eggs are donated by an IVF procedure, and the donated eggs are inseminated. The embryos are transfered into the recipient's uterus, which is hormonally prepared with estrogen/progesterone therapy

objective symptoms of pregnancy

elevated basal temperature breast and abdominal enlargement changes in the uterus and vagina melasma linea nigra

ankle edema intervention

elevating the legs at least twice a day and when resting sleeping in a side-lying position wearing supporting stockings avoiding sitting or standing in 1 position for long periods

flat bones and skull develop

embryonic period

dicidua

endometrium after implantation

iron

enough for 5 months after birth from mother

Blastocyst

erm used to refer to the solid cell ball when a cavity becomes recognizable within it.

factors increasing flow

estrogen

Fertility data for the woman include...

evaluation of the cervix, uterus, tubes, and peritoneum; detection of ovulation; assessment of immunologic compatibility; and evaluation of psychogenic factors.

Hydramnios

excessive amount of the amniotic fluid >2L.

ptyalism

excessive salivation

assisted reproductive technology (ART)

fertility treatments in which both egg and sperm are handled involves removing egg from the woman, fertilizing the egg in the lab, and returning embryo or enmbryos to the woman or srrogate carrier

meconium

fetal waste products as green to black, tarry meconium passes through within first 24 hrs of birth

Chorionic villi

finger-like projections, develop out of the trophoblast and extend into the blood-filled spaces of the endometrium.

prenatal visits schedule

first trimester - initial visit weeks 16-28 - monthly 29-36 - biweekly 36-birth - weekly

Amniotic fluid

fluid that surrounds the developing baby in utero comes from diffusion from the maternal blood. 700 to 1000ml.

Fatigue interventions

frequent rest periods throughout the day use correct posture and mechanics obtaining regular exercise performing muscle relaxation strengthening exercise for the legs and hip joints avoiding eating and drinking food containing stimulants

Allele

genes on corresponding loci that code for variations of the same trait

inhibit lung maturity

gestational diabetes chronic glomerulonephritis

Gestational carrier ( embryo host)/ surrogate mother

gestationla carrier 1. a couple undertakes an IVF cycle and the embryo is yransferred to the uterus of another woman ( the carrier) who has contracted with the couple to carry the baby to term. The carrier has not genetic investment in the child. 2. surrogate motherhood is a process by which a woman is inseminated with semen and then carries the fetus until birth

embryoblast

gives rise to the embryo part of blastomere

trophoblast

gives rise to the plasenta part of blastomere

GTPAL

gravida, term births, preterm births, abortions, living children

intestines displaced

growing uterus touches the anterior abdominal wall and displaces the intestines to either side of the abdomen

fetal alcohol syndrome

growth restrictions CNS abnormalities facial dysmprphia

MyPlate

guide that can be used to make daily food choices during pregnancy and lactation, just as during other stages of the life cycle.

Earliest biochemical marker

hCG (human chorionic gonadotropin)

What 4 hormones do MOMs endocrine glands produce?

hCG, hCS, progesterone, and estrogen

meiosis has 4

haploid cells

Mitosis

he process by which body (somatic) cells replicate to yield two cells each with the same genetic makeup as the parent cell.

Nutritional assessment

health history (chronic illnesses) usual maternal diet physical examination ( weight and height minimum) lab tests - Hg Hct ( risk of anemia)

What functions does amniotic fluid serve?

helps maintain a constant body temp, a source of oral fluid & repository for waste, assist in the maintenance of fluid & electrolyte balance, allows freedom of movement for musculoskeletal development, cushions the fetus from trauma, acts as a barrier to infection, and allows for fetal lung development

complimentary and alternative measures

herbals have not been proven to promite fertility - relaxation, osteopathy, stress management ( aromatherapy, yoga, acupuncture) - nutritionla and exercise counseling - antioxidants - vitamins E and C, selenium, zinc, coenzymes Q10, ginseg

ovarian failure

high follicle stimulating hormone and low estrodiol

uterus changes in size, shape, position

high level of estrogen and progesterone lead to uterine growth in first trimester ( 7th week- large hen egg, 10th week - orange; 12 weeks - grapefruit at conception - upside down pear; second trimester - spheric or globular; later becomes ovoid palpable above symphesis pubis ( between 12-15 weeks) level of umbelicus ( 22-24 weeks of gestation xiphoid - at term

birth setting choices

hospital birth center home birth

Letrozole off-label use/ side effects

hot flashes headaches breast tenderness may increase risk of cengenital anomalies

monozygotic twins

identical twins develop from one fertilized ovum which then divides they are the same sex and have the same genotype ifdicision soon after fertiolization - 2 embryos, 2 amnjons, 2 chorions, 2 placentas division between 4-8 days - 2 embryos, 2 amnions, one chorion, one placenta

pregnant women tubular absorption of glucose

impaired 1-10 g of glucose in pregnant vs 100 mg in non pregnant

cervical changes

in normal unscarred cervix softening of the cervical tip can be observed about the beginning of the 6th week

sperm remain viable

in the female reproductive truct - 48 hr ormore but few retain fertilization potential for more than 24 hrs

total blood volume

increaes by 30-45% by 12-1500 ml

Hyperpigmentation

increase in melanotropin Darkening of nipples, areolae, axillae, and vulva; chloasma; linea nigra

BMR

increased by 10-20% by 3rd trimester returns to nonpregnant level by 5-6 days after birth leads to heat intolerance, greater need for sleep

Changes in urine

increased GFR by 50% and renal plasma flow (RPF) in early pregnancy, glucosuria and wasted glucose and other nutrients, proteinuria does not usually occur in normal pregnancy except during labor or after birth increased createnine clearance reduction of serum createnine, BUN, uric acid levels increase excretion of albumin ( abnormal > 30 mg/24 hrs)

uterine blood flow

increases 10 fold through uterus 450-650 ml min

side lying position

increases renal perfusion -> increases urine output and decreases edema

Metformin off label use/ MOA

induces ovulation through reducing insulin resistance and thus affecting gonadotropins and andrigens stimulates the ovary

listeriosis

infection that can occur as a result of consuming certain foods such as unpasteurized milk or products made with unpasteurized milk, raw fish or seafood, raw dough or batter, and unpasteurized juice; it increases the risk for miscarriage, premature birth, and stillbirth

Embryo donation/ indication

infertility not resolved by less aggressive forms of therapy absence of ovaries male partner is azoospermic or is severely compromised

endometriosis

inflammatory peritoneal damage caused by endometrial tissue that has been migrated out of the uterus and implanted on pelvic organs or connective tissue

hormonal analysis

is performed to assess endocrine function of the hypothalamic-pituitary-ovarian axis when menstrual cycles are absent or irregular to assess women's ovarian reserve blood level of prolactin and thyroid hormones follicle stimulating hormone and astrodial are drawn on day 3 of the cycle

inability to digest lactose because of the absence of the lactase enzyme in the small intestine

lactose intolerance

renal function most effeicient

lateral recumbent position

elevated estrogen

ligaments of rib cage relax permitting chest expansion

human chorionic gonadotropin (hCG)/side effects

local irritation at injection site headaches irritability edema depression' faitgue

oligospermia

low sperm count - spermatic fluid is ejaculated backwards, or retrograde, into the bladder

Hg

lowest limit 11 g - 1st trimester, 3rd 10.5 g - second trimester 12 g non pregnant

intracytoplasmic sperm injection/ indication

male partner is azoospermic or has a very low sperm count; couple has a genetic defect male partner has antisperm antibodies

indications for intrauterine insemination

male subfetility cervical factor vaginal factor

Pinocytosis

mechanism used for transferring large milecules- albumin, gamma globulins across the placental membranes

BMI; underweight; normal weight; overweight (high); obese

method used to evalueate the appropriateness of weight for height. If the calculated value is less than 18.5, the person is considered to be __, 18.5-24.9 is __, 25-29..9 is __, and >30 is __.

drugs supporting ovulation in women with polycyctic ovary disease

metrofmin ( insulin sensitizing agent) dexamethasone ( steroid) - potentiate clomiphene to treat anovulation

assisted hatching

micromanipulation technique if zona pellucida is thick and tough and the embryo cannot break through or "hatch", through this coating in the blastocystic phase of development infrared laser is used to create a hole in the zona pellucida so that the embryo can break through and implant the zona pellucida is penetrated chemically or manually to create an opening for the dividing embryo to hatch and implant into uterine wall

intracytoplasmic sperm injection

micromanipulation technique introduces sperm beneath the zona pellucida directly into the egg selection of one sperm cell that is injected directly into the egg to achieve fertilization

where it occurs

mitosis - all organisms miosis - Humans, animals, plants, & fungi

Cleavage

mitotic cellular replication zygote travels the length of the uterine tube into the uterus 3-4 days

conjoined

monozygotic twins in which cleavage is incomplete and occurs late (13-15 days postconception)

capacity of kidney to excrete water

more efficient earlier than later -> more thristy at the beginning

medical therapy for infertility/ ovarial stimulation

most common - selective estrogen receptor modulator ( clomiphene) or gonadotropin ( FSH, LH)

It is common for women to be ambivalent about the pregnancy during which trimester of pregnancy? In the first trimester In the third trimester In the fourth trimester In the second trimester

n the first trimester In the first trimester, it is common for women to be ambivalent about the pregnancy.

risks of assisted reproductive technology

nausea - fluid retention - ovarian hyperstimulation -invasive procedures - psychological stress - general anesthesia - congenital malformations - placental problems - miscarriages - multiple gestations - ectopic pregnancy

Metformin off label use/ side effects

nausea vomiting diarrhea lactic acidosis liver dysfunction

Poor ovarian reserve

need donor eggs

Recommended weight gain pattern for normal weight preg women?

normal wt- 0.4kg wkly. (During whole preg 25 to 35 lbs)

IgA

not produced by fetus colostrum contains large amount and provide passive immunity to the neonate

cervix

nullipara - rounded after birth - more oval in the horizontal plane and the transversal os appears as a transverse slit

other factors affecting femal fetility

nutritional deficiencies ( anemoa) - obesity - substance abuse - thyroid dysfunction - genetic disorders ( turner syndrome) - anxiety

Recommended weight gain pattern for obese preg women?

obese wt- 0.2kg wkly

backache interventions

obtaining rest - using correct posture - wearing low-healed comfortable shoes - performing pelvic tilt exercises - sleeping on a firm matress

Follitropins (purified FSH)/side effects

ovarian enlargement ovarian hyperstimulation local irritation at injection site multifetal gestation

Menotropins-human Menopausal Gonadotropins (hMG) side effects

ovarian enlargement ovarian hyperstimulation local irritation at injection site multifetal gestation

Menotropins-human Menopausal Gonadotropins (hMG) indication

ovarian follicular growth and maturation

intrauterine insemination

ovarian stimulation therapy floowed with times intercourse if sperm quality is low or female has factors such as unfavourable cervical mucus, semen allergy, or endometriosis, the sperm can be introduced directly into the uterus using intrauterine insemination (IUI)

Recommended weight gain pattern for overweight preg women?

over wt- 0.3kg wkly

Letrozole off-label use/ indication

ovulation induction

human chorionic gonadotropin (hCG)/ indication

ovulation induction

Clomiphene citrate indication

ovulation induction treatment of luteal phase inadequacy

Causes of hyperprolactinemia

phenothiazine opiates - diazepam - reserpine - methyldopa - TCA - benighn pituitary adenoma

prenatal care choices

physicians midwives certified nurse midwives direct entry midwives traditional or lay midwives doulas

Normal adaptation that occurs during pregnancy when the plasma volume increases more rapidly than red blood cell mass

physiologic anemia

other blood changes

plasma increase more than RBC decrease in HG and Htc WBC increases

decidua capsularis

portion covering the blastocyst

decidua vera

portion lining the rest of the uterus

Pica

practice of consuming nonfood substances (clay, starch, soil, laundry starch). about 40% of women experience pica leads to low Hg could lead to consumption of lead, dietary imbalance, contaminated with heavy metals, toxic substances associated with iron and calcium deficiency

obesity is a risk for

preeclampsia gestational diabetes macrosomia cephalopelvic disproportion operative vaginal birth emergency cesarean birth postpartum hemorrhage wound, UTI, genital tract infection birth trauma late fetal death miscarriage congenital anomalies growth abnormalities stillbirth obese child

Gravidity

pregnancy

hCG level greater than 25

pregnancy

lung maturity determined

presence of pulmonary surfactants

What is hCGs purpose?

preservers the function for the Corpus Lutem, ensuring the continued supply of estrogen and progesterone needed for pregenancy

Ketonuria is associated with

preterm labour

meiotic divisionspermatocytes

primary spermatocyte goes through 1st meiotic division and has copies of DNA 1st division - 2 haploid secondary spermatocytes are formed 2nd meiotic division - male produces 2 gametes with X chromosomes and 2 with Y chromosomes

Fertilization

process of successful penetration of the membrane surrounding the ovum by a sperm. takes place in the ampulla (outer third) of the uterine tube. takes 3 to 4 days for the egg to travel the length of the tube into the uterus

Gametogenesis

process whereby gamestes are formed and mature. for the male = speratogenesis for the female = oogenesis

IgM

produced by the end of first trimesteer

increased vaginal discharge interventons

proper cleansing and hygiene wearing cotton underwear avoiding douching consulting HCP if infection is suspected

Dietary Reference Intakes

recommendations for daily nutritional intakes tha tmeet the needs of almost all healthy members of the population

assisted hatching indications

recurrent miscarriages - to improve implantation rate in women with previously unsuccessful IVF attempts advanced age

halving of genetic material accomplished

replicating DNA once and then dividing twice

Metformin off label use/ indication

restores cyclic ovulation and menses in many women with PCOS

intrahepatic cholestasis

retantion and accumulation of bile in the liver caused by factors within liver

hyperemesis gravidarum

severe and persistent vomiting during pregnancy causing weight loss, dehydration, and electrolyte abnormalities

epidermis begins

single layer of cells derived from the ectoderm at 4 weeks

hemorroids interventions

soaking in warm sitz bath - sitting on a soft pillow - eating high fiber food - drinking sufficient fluid - increasing exercise - applying ointments, suppositories

Hegar's sign

softening and compressibility of lower uterus

Goddell's sign

softening of the cervix

Morula

solid ball of cells surrounded by the protective zona pellucida

fluid from lungs

squzzed during birth remaining absorbed into blood stream durign first 2 hr not squeezed during Cs, child migth have problems

fingernails

start at 10th weeksn- thickening from epidermis reach fingertip by 32 week toenails - 36

placenta

structure composed of 15-20 lobes/areas called cotyledons. Serves as a means of metabolic exchange and produces several hormones including human chornic gonadtropin (hCG)

umbilical cord

structure that connects the developing baby to the placenta. it contains 3 vessels: 2 arteries and 1 vein.

least efficient

supine position

breast tenderness interventions

supportive bra - avoid using soap on nipples

GnRH antagonists (ganirelix acetate, cetrorelix acetate)/MOA

suress gonadotropin secretion; inhibit premature LH surges on women undergoing ovarian hyperstimulation

Physiologic/dependent edema

swelling of ankles and feet at the end of the day

shortness of breath interventions

taking frequent rest - sitting and sleeping with the head elevated or on the side - avoiding overexerction

Fetus

term used to refer to the developing baby from 9 weeks of gestation (when fetus becomes recognizable as a human being) until the pregnancy ends.

Embryo

term used to refer to the developing baby from day 15 until 8 weeks after conception when embryo measures 3 cm from crown to rump at 4 weeks arm buds and the beginning of eyes are forming

healthful diet for conception

the best way to ensure that adequate nutrients are available for the developing fetus

Wharton's jelly

the connective tissue that prevents compression of the blood vessels to ensure continued nourishment of the developing baby.

Decidua

the endometrium after implantation

Zygote

the first cell of the new individual

Trophoblast

the outer layer of cells surrounding the bastocyst cavity

Decidua basalis

the portion directly under the blastocyst, where the chorionic villi tap into the maternal blood vessels

Meiosis

the process by which germ cells divide and decrease their chromosomal number by half, produces gametes (eggs and sperm) When the female gamete (egg or ovum) and the male gamete (spermatozoon) unite to form the zygote, the diploid number of human chromosomes (46, or 23 pairs) is restored.

Conception

the union of a single egg and sperm?

ossification continues

through childhood throughout life with the growth and remodeling of bones

exogenous progesterone or human gonadotropin is given

to support early pregnancy until placental production is sufficient

exogenous progesterone/ indications

treatment of luteal phase inadequacy

Follitropins (purified FSH)/ indication

treatment of polycystic ovary syndrome follicle stimulation for assisted reproductive techniques

stress can cause infertility

true

gestational mother is the legal mother in oocyte donor cases

true egg donot has no rights or responsibilities in relation to the offspring

in vitro fertilization- embryo transfer/ indications

tubal disease or blockage severe male infertility endometriosis unexlained infertility cerfical factor immunologic infectility

Gamete intrafallopian transfer (GIFT)/ indications

tubal disease or blockage severe male infertility endometriosis unexlained infertility cerfical factor immunologic infectility except there must be normal tubal anatomy, patency, and absence of previous tubal disease in at least one uterine tube

zygote intrafallopian transfer (ZIFT)/ indications

tubal disease or blockage severe male infertility endometriosis unexlained infertility cerfical factor immunologic infectility except there must be normal tubal anatomy, patency, and absence of previous tubal disease in at least one uterine tube

fetal HR at 17 days

ultrasound can detect

Recommended weight gain pattern for underweight preg women?

under wt- 0.5kg wkly

What are the values for the follow BMI classifications: underweight, normal, overweight, obese

underweight- less than 18.5 normal- 18.5 to 24.9 overweight- 25 to 29.9 obese- 30 or more

nasal stuffiness interventions

use of humidifier avoid using nasal sprays or antihistamines

hysteroscopy

uses a flexible scope threaded through the cervix to directly view the uterine cavity. This is the definite method for evaluation of lyomyomas and adhesion

sonohysterography

uses fluid infused into the uterus through the cervix to help define the uterine cavity and the depth of the uterine lining, using vaginal ultrasound

Secondary anovulation

usually seen in young to midlife women --> is relatively common and is caused by the disruption of the hypothalamic-pituitary-ovarian axis

diminished blood flow to fetus

vasoconstriction ( hypertension, cocaine) woman on her back ( compress vena cava) excessive maternal exercise ( diverts blood fromuterus to muscles)

clomiphene citrate side effects

vasomotor flushes abdominal discomfort nausea and vomiting breast tenderness ovarian enlargement

laparascopy

visual examination of the abdominal cavity for pelvic structures

when is the diploid number restored

when the female gamete and male gamete unite to form the zygote

in vitro fertilization- embryo transfer

woman's eggs are collected from her ovaries, fertilized in the lab with sperm, and transferred to her uterus after normal embryo development has occured ovarian follicales are monitored via ultrasound and removed at maturity through intravaginal needle aspiration or laparascopic procedure sperm retreived via ejaculation or via needle

Where does hematopoiesis occur?

yolk sac, begin 3rd week begins during 6th week


Kaugnay na mga set ng pag-aaral

Chapter 21: Respiratory Care Modalities

View Set

Final Exam: Human nutrition chapter 19

View Set

840. David Goggins # Kids Explain What Is Love

View Set

PA/ID 90-1 ROSTER STAFFING SCHEDULED OVERTIME

View Set

Chapter 1-5 Practice Quiz : Economics

View Set